Anda di halaman 1dari 447

See discussions, stats, and author profiles for this publication at: https://www.researchgate.

net/publication/329031752

Quantitative Methods For Business Management

Book · November 2018

CITATIONS READS
0 3,229

1 author:

Dominic Ngugi
Africa Nazarene University
2 PUBLICATIONS   0 CITATIONS   

SEE PROFILE

Some of the authors of this publication are also working on these related projects:

INFLUENCE OF BEHAVIORAL CHARACTERISTICS ON INDIVIDUAL PERCEPTION OF A PORTFOLIO RISK IN NAIROBI SECURITIES EXCHANGE View project

Financial modelling View project

All content following this page was uploaded by Dominic Ngugi on 19 November 2018.

The user has requested enhancement of the downloaded file.


Quantitative Methods
For Business
Management
Quantitative Methods

John Kimani Gichuhi


Lecturer
Africa Nazarene University

Dominic Ngugi Ndungu


Africa Nazarene University
THE AUTHORS
SUMMARY CONTENTS
CHAPTER 1 SET THEORY AND COUNTING NUMBERS 1

CHAPTER 2 LINEAR EQUATIONS 15

CHAPTER 3 SIMULTANEOUS LINEAR EQUATIONS 48

CHAPTER 4 ALGEBRA 63

CHAPTER 5 BINOMIAL THEOREM 75

CHAPTER 6 PERMUTATIONS & COMBINATIONS 81

CHAPTER 7 MATRICES ALGEBRA 84

CHAPTER 8 DECISION ANALYSIS 155

CHAPTER 9 CALCULUS I: DIFFERENTIATION 167

CHAPTER 10 CALCULUS II: INTEGRATION 218

CHAPTER 11 NON – LINEAR EQUATIONS 228

CHAPTER 12 REVENUE, COST AND PROFIT 241

CHAPTER 13 DECISION TREES 250

CHAPTER 14 MATHEMATICS OF FINANCE 258


CHAPTER 15 LINEAR PROGRAMMING 274

CHAPTER 16 INTRO TO QUANTITATIVE TECHNIQUES 293

CHAPTER 17 L P: SIMPLEX METHOD 295

CHAPTER 18 NETWORK ANALYSIS AND SCHEDULING 313

CHAPTER 19 TRANSPORTATION MODEL 343

CHAPTER 20 ASSIGNMENT MODEL 378

CHAPTER 21 INVENTORY PLANNING & CONTROL 406


CONTENTS
CHAPTER 1 SET THEORY AND COUNTING

Introduction 1

Standard Symbols used in set 2

Intersection of Sets 3

Venn diagram 4

Principle of counting 6

Application of Venn 8

Practice Problem 11

CHAPTER 2 LINEAR EQUATIONS

Introduction 15

Number line 15

Integers 16

Multiplication and Division 19

Points on x and y Plane 20

Gradients 21

Practice Problem 23

Equation of Straight 25

Practice Problem 30

Sketching a Graph 31

Points of intersection 33

Practice Problem 36
CHAPTER 3 SIMULTANEOUS LINEAR

Elimination method 38

Practice Problem 39

Solving simultaneous equations with three unknowns 40

Practice Problem 41

Solving Simultaneous Equations more than three unknown 42

Practice Problem 45

Application of Linear Equation to Business 46

Demand Function 47

Supply Function 49

Practice Problem 52

Equilibrium 55

Practice Problem 62

CHAPTER 4 ALGEBRA

Introduction 63

Practice Problem 63

Simple Linear inequalities 64

Practice Problem 64

Compound Linear inequalities 64

Practice Problem 64

Inequalities and number line 65

Practice Problem 65
Solving linear inequalities 66

Practice Problem 66

Solving simultaneous linear 67

Practice Problem 68

Intersecting Regions 69

Application of linear inequalities 70

Practice Problem 72

CHAPTER 5 BINOMIAL THEOREM

Introduction 75

Operations of problem 76

Expanding 78

Practice Problem 79

CHAPTER 6 PERMUTATIONS & COMBINATIONS

Introduction 81

Evaluating permutations 82

Practice Problem 82

Combination 83

Practice Problem 83

CHAPTER 7 MATRICES ALGEBRA

Introduction 84

Types of matrices 84
Square matrix 84

Identity matrix 85

Row vector matrix 85

Column Vector 85

Null vectors 85

Practice Problem 86

Operation of Matrices 87

Additional of matrix 87

Subtraction of matrix 88

Scalar multiplication 88

Matrix transportation 89

Multiplication of matrix 90

Practice Problem 90

Application in Business 92

Practice Problem 94

Inversion of Matrices 95

Practice Problem 99, 109

Application of Matrices 121

Practice Problem 138

Markov Process model141

Characteristics or Assumptions of Markov 141

Practice Problem 151

CHAPTER 8 DECISION ANALYSIS


Introduction 155

Essential elements for decision 155

Maxmin Minimax Approach 158

Minimax Regret Approach 158

Decision making with probabilities 160

Practice Problem 168

CHAPTER 9 CALCULUS I: DIFFERENTIATION

Introduction 167

The gradient of a curve 170

Practice Problem 177

Rules of Differentiation 178

Constant Rule 178

Practice Problem 179

The Sum Rule 180

Practice Problem 179

The Difference rule 181

The Sum Rule 182

The Combination Sum rule and Difference rule 183

Practice Problem 184

The Product rule or Multiplication rule 185

Practice Problem 186

The Quotient Rule or Division Rule 187

The chain rule 190


Practice Problem 192

Maximization and Minimization of Function 193

Stationary Points 193

Application of differentiation 199

Practice Problem 206

Partial Differentiation 208

Practice Problem 212

Constrained Optimization 214

Practice Problem 216

CHAPTER 10 CALCULUS II: INTEGRATION

The anti – derivative 218

Definite Integral 220

Application of Definite Integral 222

Practice Problem 225

CHAPTER 11 NON – LINEAR EQUATIONS

Quadratic functions 228

Discriminant 229

Practice Problem 232

Application of Quadratic equation 235

Practice Problem 239

CHAPTER 12 REVENUE, COST AND PROFIT

Introduction 241
Total Revenue 241

Total Cost 244

Profit 244

Break even analysis 245

Practice Problem 247

CHAPTER 13 DECISION TREES

Introduction 250

Solving decision tree 252

Practice Problem 255

CHAPTER 14 MATHEMATICS OF FINANCE

Simple Interest 258

Compound Interest 259

Investments Analysis 261

Payback Period Method 262

Net Present Value 264

Practice Problem 270

CHAPTER 15 LINEAR PROGRAMMING

Introduction 274

Steps involved in linear programming 275

Problem Formulation programming 275

Solving linear program using graphical 278

Essential functions in linear 282


Formulating a linear programming 282

Application of linear program 283

Practice Problem 288

CHAPTER 16 INTRODUCTION TO QUANTITATIVE

Introduction 293

Stages in Quantitative study 293

Example of Model 294

CHAPTER 17 LINEAR PROGRAMMING:

Solving linear program using simplex 295

Definition of Terms 295

Solving LP using simplex algorithm 296

Shadow Prices 306

Slack variables 306

Practice Problem 306

Dual Program/ Minimizing Simplex Method 311

CHAPTER 18 NETWORK ANALYSIS AND

Introduction 313

Basic Network Terminologies 313

Rules for drawing networks 314

Using the Standard Normal Table 318

Float time 336

Practice Problem 339


CHAPTER 19 TRANSPORTATION MODEL

Introduction 343

Stages of Transportation Algorithm 343

Solving a transportation problem using minimum cost 344

Alternative allocation 350

Vogel method 351

Testing for optimality 357

Testing for optimality using Module 357

Degeracy 369

Practice Problem 374

CHAPTER 20 ASSIGNMENT MODEL

Introduction 378

Hungarian Algorithm 378

Alternative assignment 383

Practice Problem 402

CHAPTER 21 INVENTORY PLANNING & CONTROL

Introduction 406

Basic stock model 407

Assumptions of EOQ 407

Annual cost of ordering 408


Annual cost of holding the stock 408

Total cost in EOQ Model 409

The optimum order quantity 409

Economic Batch Quantity 412

The planned shortage model 417

Practice Problem 420


SET THEORY AND
CHAPTER 1 COUNTING NUMBERS
Learning Objectives
At the end of this section the students should be able to:
• Drawing of the Venn diagram.
• Identify elements of sets
• Identify the different symbols used in the sets
• Apply Venn diagram to the world of business today

A set is a collection of objects or elements. A set of student may contain students who are male
and female and also students registered for different degree programs. The set brace { } are
used enclose numbers belonging to the set.
A set containing of numbers1, 2,3, 4, 5, 6, 7, 8, 9 and 10may be written as follow
{1, 2, 3, 4, 5, 6, 7,8, 9,10}

Elements are numbers contained in a set. The above is a set of number 1 up to 10.
When writing down the set of multiples of 4
The listings of the multiples can be done as follows.
{4, 8, 12, 16, 20, 24 …}
Notice that there has been a pattern established and the last three dots indicate the patterns
continue to the last term which could be the nth term.
Consider listing the multiples of 4 which are less than 100

{4, 8,12, 16, 20, 24,...,100}


Instead of listing all the elements the three dots can be used to show where there is a pattern.
When we wish to show that a particular number belong to a particular set we use the following
symbol. Thus if we have a set of prime numbers less than ten
{2, 3, 5, 7}
CHAPTER 1 Set theory and counting number 2

Thus to show that an element belong to a set of prime numbers less than ten {2, 3, 5, 7}
We can write as follows
5 ∈ {2, 3, 5, 7}
5 is an element of the set containing 2, 3, 5 and 7. Also 3∈{2, 3, 5, 7}
If we want to show that an element is not an element of the above set we place slash through
the symbol as follows
11 ∉{2, 3, 5, 7}
When naming sets we use capital letter for example let the above set be A, written as follows
A={2, 3, 5, 7}
then 3 ∈ A and 11∉ A
Two sets are equal if they contain the same elements for example A= {1, 2, 3} and B= {3, 2, 1}all
contain exactly the same elements and are equal.
In symbols it is written as follows
{1, 2, 3}={3, 2, 1}
Sets that do not contain exactly the same elements are not equal. For example set C = {4, 5, 6}
and D = {7, 8, 9} are not equal.
In symbols it can be written as follows
{4, 5, 6} ≠ {7, 8, 9}

Standard Symbols used in set

¥= {0,1, 2, 3, 4, 5,...}
- used to represent set of natural numbers ,
- represent positive and negative integer, = {+4, +3, +2, +1, -1, -2, -3, -4}
- represent a set of rational numbers, = {1.2, 1.3, 1.4, 1.5}
- represent set of real numbers- ={
CHAPTER 1 Set theory and counting number 3

Intersection of Sets

Intersection of sets is represented by the symbols ∩


We say that the intersection of two set A and B is written as follows
A ∩ B which means the elements that are contained in both A and B. For example if set A
contain {1, 2, 3, 4, 5} and set B contains the following elements { 2, 4, 6, 8, 10}
A = {1, 2, 3, 4, 5}
B = {2, 4, 6, 8, 10}
Then the intersection is written as follows,
A ∩ B = {2, 4}

Example 1.1

Given the following sets write down their intersection


G = {4, 5, 6, 7, 8. 9} and H = { 1, 3, 5, 7}
the intersection is given by
G ∩ H = {5, 7}
The set containing all elements in A and B is called Union of set A and B and it is represented by
the following symbol A ∪ B given as follows A ∪ B = {1, 2, 3, 4, 5, 6, 8, 10}
Consider set C which is a set containing odd numbers {3, 5, 7, 9} and a set D which contain
multiple of 2 {2, 4, 6, 8.10}
Then there is no intersection C ∩ D Such a set with no members, is called an empty set , it is
represented by ∅
Thus the above example is written as C ∩ D = ∅
In most case there are some elements which are contained in a particular question or topic.
Such sets are represented by the symbol ∪ it is called the universal set.
On the same not all elements of the universal set that are not contained in set A are called
compliment of set A which are written as A' (complement set of A)
For example given the following universal set = { 3, 4, 5, 6, 7, 8, 9} and that A = {5, 6, 7}
the compliment of A is the set of
A' = {3, 4, 8, 9}
CHAPTER 1 Set theory and counting number 4

Note that A ∩ A' = ∅ and A ∪ A' = ∪


Consider set E is also a member of set F, then E is called a subset of F.
For example E = {4, 5, 6, 7} is a subset of { 3, 4, 5, 6, 7, 8, 9, 10} the symbol is written as follows E
⊂F
In set the notation n(A) is used to represent the number of elements in set A

Venn Diagram

A Venn diagram is diagrammatic representation of the relation of relationships of sets


A Venn diagram contains of a rectangle which encloses all the elements contained in the
universal sets and circles which represent the sets and subsets.

Example 1.2

Set A contains {1, 2, 3, 4, 6, 7} and B = {2, 6, 8, 9, 10}


Representing this on Venn diagram is as follows
The intersection will be A ∩ B = {2, 6} Point of
intersection

A∩B
A 1, 3, 4, 7 2, 6 8, 9, 10 B
CHAPTER 1 Set theory and counting number 5

Example1.3

Consider the following Set C which is {3, 5, 7, 9} and set C is a subset of set D { 1, 2, 3, 4, 5, 6, 7,
8. 9} then drawing the Venn diagram is draw as follows
C ∪ D = {1, 2, 3, 4, 5, 6, 7, 8, 9}

U = {1, 2, 3, 4, 5, 6, 7, 8, 9}

D 2, 4, 6, 8 1, 3, 5
C

7, 8

Example 1.4
Given that E = {1, 2, 3, 4, 5, 6, 7, } and 8, 9 and 10 are not contained are compliments of E that is
E' ={8, 9, 10} this can be drawn as shown in the Venn diagram.

E
E'

Placing the numbers we obtain the following Venn representation.

E 1,2, 3, 4, 5. 6,7
s E'
8, 9, 10

Two sets which have no elements in common are said to be disjoined sets. For example set
K = { 12, 13, 14, 17} and H = {11, 15, 16, 18, 19 }
CHAPTER 1 Set theory and counting number 6

This can be drawn as follows in the following Venn diagram.

H 11,15,16,18,19 K 12,13,14,17

Principle of counting

Consider set A if we denote the number of elements in set A as discussed earlier we can
symbolically write it as n(A)
If set A is given by A = {1, 2, 3, 4} the we denote the number of elements contained in set A to
be n(A) = 4 and if another set B is given by B = {a, b, c, d, e, f, g, h, h, I, j, k} then n(B) = 11
If set C is an empty set or null set then C = ∅ and can be denoted by n(C) = 0.
Principle of Inclusion-Exclusion
Let consider two set A and B
A = {a, b, c, d, e, f, g, h}
B = {b, d, f, k, l, m}
Then the universal set will be
A = {a, b, c, d, e, f, g, h, k, l, m} then n(A) = 11
A ∩ B = {d, d, f} then A ∩ B = 3
A = {a, b, c, d, e, f, g, h} then n (A) = 8
B = {b, d, f, k, l, m} then n (B) = 6
In general we can use the Inclusion-Exclusion principle which states that given two sets the
number of the union is equal to the number of the first set plus the number of the second set
minus the intersection of the two sets
n(A ∪ B) = n(A) + n(B) – n(A ∩ B)
To explain this further lets consider the above example
n(A ∪ B) = 11 n(A) = 8 n(B) = 6 n(A ∩ B) = 3
n(A ∪ B) = n(A) + n(B) – n(A ∩ B)
11 = 8 + 6 – 3
CHAPTER 1 Set theory and counting number 7

Example 1.5

In the year 2009, five hundred students graduated in Africa Nazarene University. Of these 500
students, 310 had degrees in business of any sort, 238 had undergraduate degrees in business
and 184 had master’s degree in business.
Required
How many students had both undergraduate and master degree in business?

Solution
Let
A = {students with an undergraduate degree in business}
B = {students with masters degree in business}
Then
A ∪ B = {students with at least on degree in business}
A ∩ B = {students with both undergraduate and masters in business}
So n(A) = 238 n(B) = 184 n(A ∪ B) = 310

The question is asking as to check for the n(A ∩ B). By Inclusion-Exclusive principle we have
n(A ∪ B) = n(A) + n(B) – n(A ∩ B)
310 = 238 +184 - n(A ∩ B).
n(A ∩ B) = 112

Sets Operation
A ∪B = B ∪A = Commutative law for Union
A ∩B = B∩A = Commutative law for intersection
A ∪ (B ∪ C) = (A ∪ B) ∪ C = Associative law for union
A ∪ (B ∪ C) = (A ∪ B) ∪ C = Associative law for intersection
A ∪ (B ∩ C) = (A ∪ B) ∩ (A ∪ C) = Distributive law for union
A ∩ (B ∪ C) = (A ∩ B) ∪ (A ∩ C) = Distributive law for intersection

There are two additional rules called De Morgan Laws


( A ∪ B)C =AC ∩ B C
CHAPTER 1 Set theory and counting number 8

Application of Venn
Example 1.7
A survey conducted of 600 students at Africa Nazarene University produced the following
results.
190 of the students read Daily Mirror;
180 of the students read Telegraph newspaper;
500 read Magazines;
130 read Daily Mirror and Telegraph newspaper;
110 read the Telegraph newspaper and Magazines;
130 read Daily Mirror and Magazines:
90 read all three.

Use the above information to answer the following questions


a. How many students read none of the publication?
b. How many read magazines
c. How many read Daily Mirror and the Telegraph newspaper , but not Magazine.

Solution
Let letter D represent the Daily Mirror
Let letter T represent Telegraph Newspaper
Let letter M represent the Magazines
Thus
The number of sets in Daily mirror are n(D) = 190
The number of sets in Telegraph Newspaper are n(s) = 180
The number of sets in Magazines are n(M) = 500
The number of elements of intersection of Daily Mirror and Telegraph Newspaper
n(D ∩ S) = 130
The number of elements of intersection of telegraph Newspaper and Magazines
n(S ∩ M) = 110
CHAPTER 1 Set theory and counting number 9

The number of elements of intersection of Daily mirror and Magazines n(D ∩ M) = 130
Intersection of the three n(D ∩ S ∩ M) = 90
Those who read Daily Mirror and Magazines only are 130- 90 = 40
Those who read Daily nation and telegraph Newspaper 130 – 90 = 40
Those that read Telegraph Newspaper and Magazines only = 110 – 90 =20

Those who read Daily Mirror only are


190 – 90 – 40 – 40 = 20
Those students who read telegraph Newspaper are
180 – 90 – 40 – 20 = 30
Those students that read Magazines only
500 – 90 – 40 – 2- = 350
Total number of students who read the three publications are
90 + 40 + 40 +20 +30 + 20 + 350 = 590

Those students who didn’t read any publication


600 – 590 = 10 students
D ∪S∪ M =600
M

350

40
20
D 20 90 S

40 30

Answers
a. None of the publication 600 – 590 =10
b. Magazine only 350
c. Daily mirror and Telegraph but not Magazine 40
CHAPTER 1 Set theory and counting number 10

Example1.8

Infotell a research firm collected the following information about particular buying behavior of
certain families and gave the following information;
150 families bought food, clothes and luxuries;
250 families bought food and luxuries;
420 families bought food;
200 families bought clothes and food;
60 families bought luxuries and clothes, but not food;
40 families bought luxuries, but neither food nor clothes;
100 families bought none of the three;
180 families bought clothes but not luxuries.

Using the Venn diagram answer the following question;


How many families were surveyed?
How man families bought clothes?
How many families bought luxuries but not clothes?
How many families did not bring luxuries?
How many families bought food or costumes?

Solution
Let letter F represent food
let letter C represent clothes
Let letter L represent luxuries

Thus
The intersection of the three n(F ∩ C ∩ L) = 150
The number of set in food is n(F) = 420
The intersection of Food and Luxuries n(F ∩ L) = 250
The intersection of food and clothes n(F ∩ C) = 200
We have no intersection of clothes and luxuries n(L ∩ C)
CHAPTER 1 Set theory and counting number 11
U = 750
Clothes

130

50
60
150
Food 120 Luxuries
100 40

Family Surveyed = 120 + 100 + 150 + 50 + 40 + 60 + 130 + 100 = 750


50 + 150 + 60 + 130 = 390
100 + 40 = 140
750 – 100 – 120 – 130 = 400
750 – 100 – 40 = 610

Practice Problem

1. In certain college club of Abel, Ang, Ban, Charles and Debal are doing a accounting
majors with Ang and Debal being student representative. Gor, Batt, Hesh, Ila and Joy are
choir members with Ila and Joy doing degree courses.
Use set A to represent club members, set B choir members, set C student
Representatives, set D accounting major students.
a) Specify element of the element of the four sets A, B, C and D
b) Draw a Venn diagram representing sets A, B, C and D
c) Identify members of the following sets and state in words the implication of
d) A ∩ C, D ∪ C and D ∩ C
e) Establish a universal set of the student’s membership.

2. In a study on the pass rate of CPA sec IV class, 60 candidates passed QT, 80 passed FA III
and 50 passed law II. 20 passed QT and FAIII, 15 passed Law II and FAIII, 25 passed QT
and Law II while 10 passed all the three subjects.
a) What is the total number of candidates in the class?
b) If a candidate who fails one subject is referred, how many candidates were
referred?
CHAPTER 1 Set theory and counting number 12
3. The main daily newspapers in Town K are: A, B and C. The management of one dailies is
concerned about low sales volume of their paper. In a recent survey of 1000 families in
the town K the numbers that the various newspapers were found to be as follows:-
Newspaper No. of Readers
The people 280
The people and East Africa Standard 80
The East Africa Standard 300
The People and Daily Nation 100
The Daily Nation 420
The East Africa Standard and Daily Nation 50
All the three newspaper 30

a) Present this information in a Venn Diagram


b) Determine the number of families who did not read any of the three
c) Advice the manager of a car dealing business which newspaper to advertise in
order to reach the largest audience.
4. The manager of Karibu Café kept a record of his customers’ breakfast orders for one
week. He discovered the following:

200 ordered Samosa


180 ordered Hot dog
150 ordered Hamburger
100 ordered Samosa and Hot dog
80 ordered Samosa and Hamburger
60 ordered Hot dog and Hamburger
30 ordered Samosa , Hot dog and Hamburger.

Determine the customers;


a) Who ordered Samosa only,
b) Who ordered Hot dog only,
c) Who ordered Hamburger only,
d) Who ordered Samosa and Hot dog only,
e) Who ordered Samosa and Hamburger only,
f) Who ordered Hot dog and Hamburger only.

5. Some enrolment totals Africa Nazarene University for the 1st trimester 2011/2012 are
the following;

150 enrolled in Business Statistics


640 enrolled in Management Mathematics I
310 enrolled in Christian Beliefs
90 enrolled in both Business Statistics and Management Mathematics I
CHAPTER 1 Set theory and counting number 13
60 enrolled in both Business Statistics and Christian Beliefs
200 enrolled in both Management Mathematics I and Christian Beliefs
50 enrolled in all three courses

Determine the number of students enrolled in;


a) Business Statistics only
b) Management mathematics I only,
c) Business Statistics and Management Mathematics I and not Christian Beliefs,
d) Christian Beliefs only,
e) Christian Beliefs and Management Mathematics I but not Business Statistics,
f) Christian Beliefs and Business Statistics, but not Management Mathematics I.

6. Suppose set A has 8 elements, set B has 12 elements and set C has 10 elements.

Furthermore, suppose that the numbers of elements in A n B, A n C, B n C and


A n B n C are 4, 2, 3, and 1 respectively.
Determine the number of elements in A n B’ n C’, B n A’ n C’ and C n A’ n B’.

7. Explain the purpose of a Venn diagram in modern business world.

8. A study by Strategic Research Consultants on the cause of business failure of 1500 micro
enterprise firms gave the following results:

600 failed due to lack of capital only


110 failed due to lack of capital and inexperienced management
140 failed due to lack of capital and a poor location
230 failed due to inexperienced management only
150 failed due to inexperienced management and a poor location
60 failed due to all the three
80 failed due to other causes but not any of the three.

a) Present the information above in a Venn diagram and determine the number of
micro enterprise firms that failed due to:
b) A poor location only
c) At least two of the causes mentioned above
d) Exactly one of the causes

9. A milling company ran an advertising campaign to popularize its products. After a month
the marketing department conducted a survey to gauge the success of the campaign.
They interviewed 250 people. Out of the 250, 70 had not seen the adverts. 130 of those
who saw the adverts had bought their products. In total 160 people bought the
CHAPTER 1 Set theory and counting number 14
company’s products. How many of the interviewees neither saw the adverts nor bought
the company’s products?

10. A certain publisher is interested in the reading habits of the people in a certain small
city. A survey indicates that the number of subscribers ( in thousands) to the Time
magazine is 25, the number of subscribers Economist is 18 and the number of
subscribers to the East African is 12. The number of subscribers to both Time and
Economist is 10, the number subscribing to both Time and the East African is 1, and the
number subscribing to both Economist and the East African is 2. The number of people
with subscriptions to all three magazines is 1.

Determine the number subscribing to exactly one of the magazines.

11. The following represent 450 workers and the mode of transport that the use regularly

Mode of transport Workers


Train 290
Public transport 350
Boda Boda 290
All three 200
Both train and Boda Boda 210
Both train and public transport 250
Boda boda and Public transport 280

Determine:

a) Represent the information in Venn diagram


b) How many used at least two type of transport
c) How many used boda boda only

12. In Mombasa city, three newspapers A, B and the C. A survey shows that 30% of the
population of the city read A. B is read by355 read C and 155 read both the A and B. 5%
read both A and the C, 6% read both the A and the C and 2% read all the three.
a) Draw a Venn diagram to represent the information
b) What percentage of the city dwellers read none of the papers
c) How many read at least two of the papers
d) Africa Nazarene wants to advertise in two newspapers. Which two newspaper
would give them the best coverage
CHAPTER 2 LINEAR EQUATIONS
Learning Objectives

At the end of this chapter the student should be able to do the following:
• Simple addition, subtraction, multiplication and division of positive and
negative numbers;
• Perform simple operation on a number line;
• Identifying and plotting points on Cartesian plane;
• Getting gradient of a line;
• Drawing a line on Cartesian plane using gradient and its intercept;
• Solving simultaneous linear equations.

Number line

A number line is a horizontal line with both negative and positive numbers. Generally as
student of mathematics if you cant understand the number line you miss the point of
mathematics.
A number line consist of integers.
{ 1, 2, 3, 4, 5, 6, 7, } These numbers are called counting numbers or natural numbers.
In general between two integers are rational numbers.
For instance between 1 and 2 there are 1.2, 1.5, 1.8, .1.9.
These numbers can be written in ratios as follows respectively.
6 3 9 19
, , ,
5 2 5 10
Thus rational numbers are numbers that can be expressed in ratio of integers.
Numbers that are not rational are called irrational numbers. For instance irrational numbers
are.

π , 2, 6, 7, 10, 12
CHAPTER 2 Linear equations 16
Integers

When you think of integer it better to think of zero, whole numbers which are positive
and negative. Always when you think of a number line consider it as made of integers.
For instance 23, 1 ,457 -128, -2367 are all integers, but numbers like3/5,0.0001, 2.23004,
and -4.3 are not.
In general we that an integer is a set of {-5,-4,-3,-2,-1, 0, 1, 2, 3, 4, 5, } the three dots
means you keep on going both directions.

Consider two integers x and y


The relationships between these two numbers can be expressed as follows.
x< y
In general this means x is on the left of y in the number line. We read this as x is less
than y
Sometime x and y is written as
x≤ y

Which means x is less than or equal to y .It coincides on the number line.
Again x and y can be written as follows
x> y
This means that x is greater than y and that x is on right of y on the number line.

This means that x is greater than y or equal to y and that it coincides on the number line
x≥ y
Addition and Subtraction.
To add or subtract numbers in a number line we think of walking forward and backwards
f h h k f d

Example 2.1

Evaluate
i) -3 – 4
ii) -7 + 10
iii) -5 + 4
Solution
To evaluate
-3 - 4
It will be x (+y)
You at point (-3) you move 4 units or steps on your left or backwards
CHAPTER 2 Linear equations 17

Figure 1.1
Count 4 units or steps on left

-8 -7 -6 -5 -4 -3 -2 -1 0 1 2 3

Thus -3 - 4 = -7

To evaluate

-7 + 10
In terms of x and y it x +y
At point (-7) you move 1steps forward. Picture this that you own your friend 7 sweets and now
you have just brought 10 sweets if you give your friend 7 sweets you are left with 3 sweets.

Figure 1.2 Count 10 units or steps toward right

-8 -7 -6 -5 -4 -3 -2 -1 0 1 2 3 5 6

Thus -7 + 10 = 3
CHAPTER 2 Linear equations 18

To evaluate

-5 + 4
In general x + y
At point (-5) you move 4 steps forward.

Figure 1.3 Count 10 units or steps toward right

-8 -7 -6 -5 -4 -3 -2 -1 0 1 2 3 5 6

Thus -5 + 4 = -1

Practice Problems

Use a number line to evaluate the following.


-1+ 6
-2 - 3
4-8
–9+3
CHAPTER 2 Linear equations 19

Multiplication and Division

Multiplication of numbers involving negatives and positives one must note following order
of multiplication.
( − × − = + ) negative × negative = positive
( − × + = − ) negative × positive = negative
( + × − = − ) positive × negative = negative
( + × + = + ) positive x positive = positive
From the above rule we evaluate the following respectively.

3 × -5=-15
-4 × 4=-16
-5 × -6=30
7 × 9=63

In division these rules also apply.

( − ÷ − = + ) negative ÷ negative = positive


( − ÷ + = − ) negative ÷ positive = negative
( + ÷ − = − ) positive ÷ negative = negative
( + ÷ + = + ) positive ÷ positive = positive

We can solve the following using the above rules

Gradients

Evaluate

−3 × 6 × −9 × 2
−18
CHAPTER 2 Linear equations 20

Solution

We multiply the first part called the numerator.

−3 × 6 × −9 × 2 = −324

324
= −18
−18

Points on x and y Plane

Any given line has two of the points which one can use to plot a line. These are called the
coordinates. Coordinates are inform of x and y. Consider line PQ, the line has two points P (x, y)
and Q (x, y).
This line can plotted in graph on x on the horizontal axis and y on the y axis this is called as the
Cartesian plane.
O is the intersection of the x-axis and y- axis and is called the origin that is the point with the
coordinates (0, 0)

Figure 1.4

Vertical axis
or y- axis

O Horizontal axis or x- axis


CHAPTER 2 Linear equations 21

Gradients

The change on the y-axis over the change of the x-axis is called gradient.
We can think of the gradient of a line is the measure of its steepness or angle of slope.
If we have two points the gradient of the line is defined as the vertical change divided by the
horizontal change.
In general if we have point A ( x1, y1 ) and B ( x2 , y2 ) the gradient of the line segment AB is given
as follows.
From the line AB we can express this as:

Change in y (y distance) ∆y
Change in x (x distance) =
∆x

∆( y2 − y1 )
(Where ∆ is the Greek capital D ‘Delta’ which is used to mean
∆( x2 − x1 )
change)
Diagrammatically the line AB can be represented as bellow on Cartesian plane.

Figure 1.5

Change in y (∆y )
y-axis

A( x1 y1 ) Change in x (∆x)

x-axis
CHAPTER 2 Linear equations 22

Example 2.2
Consider three points A, B, and C .A(0,3), B ( 2, 4) and C (10, 8)
Find the gradient of AB and BC.
What can you say about the points.

Solution

Change in y (vertical distance) ∆y


=
Change in x (horizontal distance) ∆x

Figure 1.6
9 C (10, 8)
8
7
6
5 B (2, 4)
4
3
A (0, 3)
2
1

1 2 3 4 5 6 7 8 9 10

Taking point AB. A (0, 3) and B (2, 4 ) we get the slope as by

∆y 4 − 3 1
= =
∆x 2 − 0 2

Taking point AB, A(0,3),and B (2,4) we can also obtain the gradient

∆y 8 − 4 4 1
= = =
∆x 10 − 2 8 2
CHAPTER 2 Linear equations 23

Taking point BC, B (2,4) and C (10,8)

We can get the slope by

∆y 8 − 4 4 1
= = =
∆x 10 − 2 8 2

What we can see is that the gradient is the same .Thus A, B, and C are points on a straight line.
In general any two pairs of a point selected they give the same gradient

Figure 1.7
9 C (10, 8)
8
7
B (2, 4)
6
5
4
3 A (0, 3)
2
1

1 2 3 4 5 6 7 8 9 10

Practice Problem

1 Find the gradient of the following coordinates


(2, 3) and (7, 6)
(-2, 7 ) and (-1, -1 )
(4, 4)and (5, 7 )

2. If point (1, 1 ) and (4, 7) and (5, y) are point on a straight line find the value of y.
CHAPTER 2 Linear equations 24

3. The line from (1, 1) to (x, 3x) has a gradient 2 .Find x.

4. The four corners of quadrilateral are (1, 1), (2, 2), (4, 3), and (5, 3). Show that the
quadrilateral is trapezium.

5.Let points A, B and C have coordinates respectively .If A ( x1, y1 ) , B ( x2 , y2 ) and C ( x3 , y3 ) are on
straight line show that
0
y1 x2 − y2 x1 + y2 x3 + y3 x2 + y3 x1 − y1 x3 =
CHAPTER 2 Linear equations 25

Equation of Straight
The gradient is the steepness or the slope of line, curve, or arc. When we have the gradient of a
straight line we can be able to find the equation given any particular point that it passes
through.
The standard form of the equation of a straight line is give by

y mx + c
=

Where m is the gradient and c is constant which represents the y intercept.


In general given point ( x, y ) and the gradient is m then taking another point of the line ( x1, y1 )
we can express m in terms of y

∆( y − y1 )
=m
∆( x − x1 )

Multiplying all through by ( x − x1 )

y − y1
( x − x1 ) m( x x1 )
=−
x − x1

This leads to the standard equation of a straight line.

The equation can be expressed as follows using the points (x, y) and (4, 2)
y−4
= −2
x−2

y − y1 = m( x − x1 )
y mx + c
=
y mx + c
=

Where c is a constant and m the gradient


CHAPTER 2 Linear equations 26

Example 1

Find the gradient of the line through the points (4, 7), (5, 9)

Solution

∆y 9 − 7 2
= = = 2
∆x 5 − 4 1

The equation must be inform of = y mx + c


Taking any point for instance along the line (4, 7)

y−7
=2
x−4

Multiplying by x − 4 on both side

y−7
( x − 4) 2( x 4)
=−
x−4

y − 7 = 2x − 8

y = 2x − 8 + 7

y 2x −1
=

This is an equation of a positive gradient when plotting the graph the line moves from left to
right upwards.
CHAPTER 2 Linear equations 27

Figure 1.8
9
8
7 Positive gradient
6
5
4
3
2 y 2x −1
=
1

-1 1 2 3 4 5 6 7 8 9 10

Find the equation of line passing through the following points (2, 4) and (1, 6)
The gradient is given as

Solution

Change in y (vertical distance) ∆y


=
Change in x (horizontal distance) ∆x

Example 2

∆y 6 − 4 2
= = = −2
∆x 1 − 2 −1

Multiply by on both sides x − 2


CHAPTER 2 Linear equations 28

y − 4 =−2( x − 2)
y − 4 =−2 x + 4
y =−2 x + 4 + 4
y= −2 x + 8
Plotting the graph the line moves (slopes) from top left to right downwards. This kind of line is
called negative gradient.
9

Figure 1.9

8
7
6
5 Negative gradient
4
3
2 y=−2 x + 8
1

1 2 3 4 5 6 7 8 9 10

Example 2.3

Consider points A(4, 2) and B(7, 2)


The gradient of the line passing through these points
The equation we obtain is supposed to be inform of = y mx + c
∆y 2 − 2 0
= = = 0
∆x 7 − 4 3

The gradient is zero


Thus taking any point along the line we obtain the following equation
y−2
= 0 taking point (4, 2)
x−4
CHAPTER 2 Linear equations 29

Thus taking any point on the line (x, y) and taking point (4, 2)
We get the following

y − 2= 0( x − 4)
y−2= 0
y=2

When you plot the graph it is a horizontal line.

Figure 2.0
9
8
7 Zero or horizontal
6 gradient

5
4
3
2
y=2
1

1 2 3 4 5 6 7 8 9 10

Suppose you have two points (4, 2) and (4, 6) the gradient is

∆y 6 − 2 4
= = = ∞
∆x 4 − 4 0

The equation is either inform of

x = 1, 2,3, 4...........

The graph can be plotted as follows it is a vertical line.


CHAPTER 2 Linear equations 30

Figure 2.1
98 Vertical gradient

7
6
5
4
x = 1, 2,3, 4,5...n
3
2
1

1 2 3 4 5 6 7 8 9 10

Practice Problem

Find the equation of following straight lines.


Gradient 2 through (5, 6)
Gradient -3 through (1, 2)
Gradient 0 through (2, 5)
Gradient ∞ through (1, 1)
1
Gradient through (3, 3)
2
Through (2, 4) and (6, 8)
Through (1, 1) and (0, 9)
Through (1, 3) and (2, 3)
Through (1, 2) and (1, 5)
Find the gradient of the following lines
a) y = −2 x + 4
b) 5 =
y 15 + 10
c) 3 x + 6 y + 3 =0
d) 3 y − x − 1 =4
e) 2=x 6y + 7
3. A straight line goes through the points (a, 0) and (0, b) show that the equation
x y
written as + = 1
a b
CHAPTER 2 Linear equations 31

Sketching a Graph

When sketching a graph of a straight line there are two ways which can be used.

Method 1

Consider a straight line 2 x + y = 8


Given a range of −2 ≤ x ≤ 4
In general you consider −2 ≤ x ≤ 4 as the distance of the x-axis.
First make sure that the line is in the format =
y mx + c
So we get
y= −2 x + 8

With the points of x and y we plot these points into a graph


For instance point (-2, 12), (-1, 10) and so on the Cartesian plane

Figure 2.2

8
7
6
5
4
3
2 y=−2 x + 8
1

1 2 3 4 5 6 7 8 9 10
CHAPTER 2 Linear equations 32

Method 2
This an alternative method which student consider to be the simple
Using the same equation of 2 x + y = 8 plot the graph using method 2
Solution
Setting x = 0 you get
2(0) + y =8
0+ y = 8
y =8

Hence the coordinate of the first point of the line is (0, 8)


Also setting y = 0 we can obtain the value of x as follows

2x + 0 =8
2x = 8
x=4

Hence the second point has the following coordinates (4, 0)

Using these coordinates (0, 8) which give the y-intercept that is where the line cuts on intersect
with the y axis on the Cartesian plane.
The coordinates of (4, 0) this is where the line will intersects with the x-axis called the x-
intercepts.

Figure 2.3
y − Intercept
8
7
6
5
4 y=−2 x + 8

3
2
x − Intercept
1

1 2 3 4 5 6 7 8 9 10
CHAPTER 2 Linear equations 33

This is the easiest method in algebra but remember if you are looking for accurate point it is
necessary to use a graph paper or solve the simultaneous equation.

Practice Problem

Find the coordinate of the two points of the following lines and hence sketch their graphs.

i) x − 4y =
8

ii) 3x − 4 y =
7

iii) − x + 2y =6

Points of intersection

A point of intersection is where two or more straight line cut each other or meets.
We will be dealing with simultaneous equation with two or more unknowns. In general a point
of intersection gives the solution of any given simultaneous equations.

Example 2.4

Consider the following

3x + 4 y =
18.............................(i )
6x − 3y =3...............................(ii )
Required
CHAPTER 2 Linear equations 34

Find the point of intersection of the two linear equations.


Plot the point on a graph.

Solution
From the previous topic we are already familiar with how to get the points
Take the first equation
3x + 4 y =
18.............................(i )
It passes through the point (0, 4.5) and (6, 0)
6x − 3y =3...............................(ii )
The second equation passes through points (0, -1) and (0.5, 0)
These two line sketched on a graph the point of intersection is (2, 3)
Always when you have gotten the solution it is better to verify whether you have gotten the
right solution by substituting the values.
By doing this you are assured of minimizing common mistakes that students make not that they
didn’t know but they overlooked. I call these mathematical remedy.

The point of intersection is correct because it satisfies our equations

Figure 2.4

8
7
6x − 3y =
3
6
5
4 Point of
intersection (2,3)
3
2
3x + 4 y =
18
1

-1 1 2 3 4 5 6 7 8 9 10
CHAPTER 2 Linear equations 35

However some simultaneous equation that has no point of intersection that means that they
have no solution.

Example 2.5

Find the point of intersection and comment on the answer of


2x + 4 y = 11................................(i )
6 x + 12 y =33..............................(ii )

Solution

Using either of the methods discussed earlier for our case let use the second method.
Taking equation
2x + 4 y =
11................................(i )
When x = 0 we get

2(0) + 4 y + 11
4 y = 11
y = 2.75
So we have points (0, 2.75)
Also when y = 0
2 x + 4(0) =11
2 x = 11
x = 5.5
The second point on the line is (5.5, 0)
Considering the second equation
6 x + 12 y =33..............................(ii )
When x = 0
6(0) + 12 y =33
12 y = 33
y = 2.75
The first point on the second line is (0, 2.75)
When we set y = 0
6 x + 12(0) =33
6 x = 33
x = 5.5
CHAPTER 2 Linear equations 36

The second point is (5.5, 0)


With the above points we can draw a graph

Figure 2.5

6 x + 12 y =
633
5 Coincidental lines
4
3
2
1

1 2 3 4 5 6 7 8 9 10 2x + 4 y =
11

The above lines on the same line and are called coincident line

Practice Problem

Find the point of intersection of

5 x + 10 y =25 5x − 7 y =9
a) b)
15 x + 30 y =75 x− y = 3

7x + 9 y =
3
b)
2x − 5 y =
16
SIMULTANEOUS LINEAR
CHAPTER 3 EQUATIONS
Learning Objectives

At the end of this section the student should be able to:


• Solving a system or two equation using substitution and elimination
method;
• Solve a system of 3 unknown s using elimination method;
• Solve a system of simultaneous equation of more than 3 unknowns
using Gauss method;
• Detect when the system of the above equation has no solution;
• Detect when the system of the above equation has more solution that is
infinite;

Example 3.1

Consider the two equation we had earlier on the other section

3x + 4 y =18.............................(i )
6x − 3y = 3...............................(ii )
The coefficient of the first equation (i) of x is 3 and the coefficient for the second equation for
the second equation (ii) of x is 6
We can multiply the equation (i) by 2 so that we can eliminate x by subtracting equation (ii)
from the result of equation (i)
= 3 x + 4 y 18................................(i )..... × 2
= 6 x − 3 y 3..................................(ii ).... ×1
The result of equation (i) we subtract equation (ii)

x cancel when
subtracted
6x + 8 y =
36 Subtract equation (ii) from (i)
6x − 3y =3
CHAPTER 3 Simultaneous Linear equations 38

11 y = 33
y=3

With y = 3 we can deduce x from either of our two original equation.


For instance let’s take equation (i)

3x + 4 y =
18.............................(i )

We substitute y = 3

3 x + 4(3) = 18
3 x + 12 = 18
3=x 18 − 12
3x = 6
x=2

Remember the mathematical remedy check whether the value satisfy the equation

The above method is called the elimination method

There are four stages we are supposed to remember when using the elimination method.

Stage 1
Add or subtract a multiple of one equation to from the other to eliminate x or y depending on
your choice of elimination.

Stage 2
Solve the resulting equation for x or y

Stage 3
Substitute the value of x or y into one of the original equation to deduce x or y.

Stage 4
Mathematical remedy ,check the values whether they satisfy the equations.

Lets consider the above system of equation


CHAPTER 3 Simultaneous Linear equations 39

3x + 4 y =
18.............................(i )
6x − 3y = 3...............................(ii )
We can choose which unknown value to substitute
For instance let make x the subject of the equation (i)
3x + 4 y =18.............................(i )

4
Use 6 − y as your value of x to substitute from equation (ii)
3
6x − 3y = 3.............................(ii )
 4 
6 6 − y  − 3y = 3
 3 
36 − 8 y − 3 y =
3
36 − 11 y = 3
−11 y =− 3 36
−11 y = −33
y=3
With the value of y = 3 our x = 2
The above method is called substitution method.

Practice Problem

Using elimination method solve the following system of equation


6x − 4 y =
8
a)
2x − 4 y =
4

6 x + 10 y =
38
b)
4 y − 4 y + 22 =
0

3x + 2 y =
10
c)
−x + 2 y =8
CHAPTER 3 Simultaneous Linear equations 40

Solving simultaneous equations with three


unknowns

When solving a simultaneous equation with three unknowns it is important to understand that
you must derive other two equations from the three equations to get equation (iv) and (v)
Example
Consider the following systems

2 x + 6 y − 2 z = 8..................................................(i )
4 x + 2 y + 2 z = 20................................................(ii )
6 x − 2 y + 2 z = 8...................................................(iii )

Solve for x, y and z


Take the first two equations (i) and (ii)

2 x + 6 y − 2 z = 8......................................................(i )
4 x + 2 y + 2 z = 20.....................................................(ii )

To eliminate x multiply the first equation with 2


To get
2 x + 6 y − 2 z = 8......................................................(i ).......... × 2
4 x + 2 y + 2 z = 20.....................................................(ii )

Subtract the second equation from the result

4 x + 12 y − 4 z = 16
x cancel
out 4 x + 2 y + 2 z = 20
10 y − 6 z = −4.......................................................(iv)

Using the (i) and the (iii) equation to obtain the (v) equation

2 x + 6 y − 2 z = 8.........................................................(i )
6 x − 2 y + 2 z = 20........................................................(iii )

To eliminate x we multiply the first equation by 3

2 x + 6 y − 2 z = 8.........................................................(i )......... × 3
6 x − 2 y + 2 z = 20........................................................(iii )
CHAPTER 3 Simultaneous Linear equations 41

The result of equation (i) we subtract equation (iii)

6 x + 18 y − 6 z = 24
Y 6 x − 2 y + 2 z = 20
cancels 20 y − 8 z = 4.......................................................(v)

Now taking equation (iv) and (v) we can use either of elimination on substitution method to
solve for y and z

10 y − 6 z = −4.......................................................(iv)
20 y − 8 z = 4.......................................................(v)

Multiply equation (iv) with 2

20 y − 12 z = −8
20 y − 8 z = 4

We get the result and get the difference

20 y − 12 z = −8
20 y − 8 z = 4
− 4 z = −12
z=3

We obtain y through any of equation (iv) or (v)

20 y − 8(3) = 4
20 y = 4 + 24
20 y = 28
28
y=
20
y = 1.4

When you have the values of y and z we can substitute our original three equation the value
of x by taking any of the equation.
Taking equation (i)
CHAPTER 3 Simultaneous Linear equations 42

2 x + 6(1.4) − 2(3) = 8
2 x + 8.4 − 6 = 8
2 x = 8 − 8.4 + 6
2 x = 5.6
x = 2.8

Practice Problem

Find the value of the x, y and z


x + y + z =1
a) 2 x + 3 y + 3 z = 4
3x + 2 y + 5 z = 6

8q − 3s + 2t = 20
b) 6q + 3s + 12t = 35
4q + 11 y + 11 =33

2a − 3b + 10c =3
c) −a + 4b + 2c =20
5a + 2b + c =−12

Solving Simultaneous Equations more than


three unknown
Example 3.2

x1 + x2 + 3 x3 − x4 =
2.......................................(i )
2 x1 + x2 − 3 x3 + x =0.....................................(ii )
3 x1 + x2 + x3 + x4 =−2...................................(ii
CHAPTER 3 Simultaneous Linear equations 43

Solution
Replace equation (ii) with equation (i) time 2

x1 + x2 + 3 x3 − x4 2.......................................(i )..... × 2
=
2 x1 + x2 − 3 x3 + x =0.....................................(ii )

We obtain the following systems

2 x1 + 2 x2 + 6 x3 − 2 x4 =
4.......................................(i )
2 x1 + x2 − 3 x3 + x =0.............................................(ii )

Getting the difference we obtain the following result.

2 x1 + 2 x2 + 6 x3 − 2 x4 =
4...........................................(i )
2 x1 + x2 − 3 x3 + x = 0.................................................(ii )
3 x2 − 9 x3 + 3 x4 = −4..........................................(iv)

Multiply equation (i) by 3 and minus equation (iii)

x1 + x2 + 3 x3 − x4 2...........................................(i )........ × 3
=
3 x1 + x2 + x3 + x4 =−2.........................................(iii )

The result from the above is as follows

3 x1 + 3 x2 + 9 x3 − 3 x4 =
6...........................................(i )
3 x1 + x2 + x3 + x4 = −2...............................................(iii )
4 x2 − 8 x3 + 4 x4 = −8..........................................(v)

We can take equation (i), (iv) and (v) to replace equation (ii) and (iii) respectively. Now instead
of writing (iv) and (v) we are going to use (ii) and (iii)

3 x1 + 3 x2 + 9 x3 − 3 x4 =6...........................................(i )
3 x2 − 9 x3 + 3 x4 =−4..........................................(ii )
4 x2 − 8 x3 + 4 x4 =
−8..........................................(iii )

1
To eliminate x 2 we divide equation (ii) by 3 or multiply by and equation (iii) by 4
4
CHAPTER 3 Simultaneous Linear equations 44

1
3 x2 − 9 x3 + 3 x4 =
−4..........................................(ii ) ……… ×
3
We obtain the following
4
x 2 − 3 x3 + x 4 = −
3
1
In the (iii) equation when we divide both sides by
4
1
4 x 2 − 8 x3 + 4 x 4 = −8..........................................(iii )............. ×
4

After dividing we get the following result

x 2 − 2 x3 + x 4 = −2........................................................(iii )

Replace the (iii) equation with the (ii) by getting the difference

x 4 cancels

x 2 cancels
4
− 3 x3 − −2 x3 = − − −2
3
2
x3 = −
3

Using equation (iii) substitute for x3 using equation (ii)

2 4
x 2 − 3(− ) + x 4 = −
3 3
10
x2 = − − x4
3

Where x 4 is an arbitrary figure in other word it is based on subject to individual judgment or


preference.
Substitute for x3 and x 2 in the first equation
CHAPTER 3 Simultaneous Linear equations 45

 10   2
x1 −  − − x 4  + 3 −  − x 4 = 2
 3   3
10
x1 + + x 4 − 2 − x 4 = 2
3
10
x1 + − 2 = 2
3
10
x1 + − 2 − 2 = 0
3
10
x1 + − 4 = 0
3
10 − 12
x1 + =0
3
2 2
x1 − = 0 ⇒ x1 = 0 +
3 3
2
x1 =
3
2 10
Using equation (i) can get the value of x 4 having the values of x1 = , x 2 = − − x 4 and
3 3
2
x3 −
3
x1 − x 2 + 3 x3 − x 4 = 2

With the above figures we substitute.

x1 − x 2 + 3 x3 − x 4 = 2
2  10   2
−  − − x 4  − 3 −  − x 4 = 2
3  3   3
2 10
+ +x
3 3

The method we have used above is called Gaussian elimination process

Practice Problem
Find the values of x1, x 2 , x3 and x 4 from the following linear systems
x1 − x 2 − x3 + x 4 = 1
x1 + 2 x 2 + x3 = 1
2 x1 + x 2 − x 4 = 3
− x1 + x 2 − x3 + x 4 = 0
APPLICATION OF LINEAR
EQUATION TO BUSINESS
Learning Objectives

At the end of this section the student should be able to:


• Identify and draw a linear demand equation.
• Identify and draw a linear supply equation.
• Determine the equilibrium price and quantity for and quantity for single
product or good
• Determine the equilibrium price and quantity for two ,three or more
than three products

The application of linear equations in business is also called demand and supply analysis
Demand Functions
Demand function is an equation that shows the relationships between the quantity demanded
and other factors like tastes, presence of substitute and price
We are going to concentrate on price as a factor that affect demand.
Remember the formulae we had of y = mx + c
The value of y depends on the value of x thus y is called the dependent variable.
The value of x on the other hand does not depend on y but determines y thus in general it is
called independent variable.
When we replace y by P which represent the price and x replaced by Q which is the price.
We can introduce parameter or constants that is a and b
In general the equation will be as follows.

P = aQ + b

a and b are counting numbers for instance 1,2,3,4................................n


The demand function has a negative gradient
a is the gradient and b is the y intercept.
Plotting the graph it slopes from bottom left to top right upwards .The demand curve has
negative slope coefficient that is − a
CHAPTER 3 Simultaneous Linear equations: Application Linear equation to business 47

Figure 2.6

P=
−aQ + b
Price ∆y Change in x
∆y
= −a
∆x

Change in x (∆x)

Quantity

Demand Function

The marketing management of Beta shoe company noticed that on a certain month the safari
boots cost KES 1000 ,the customers bought 600 pairs and the cost was KES 1600, 300 pairs were
bought.
Assuming the demand function is a linear function calculate
The line that passes through the points
Sketch the graph
Find the change in
a) P when Q = 700
b) Q when P = 2000

Solution

We first get the gradient


CHAPTER 3 Simultaneous Linear equations: Application Linear equation to business 48

Taking the points (600, 1000) and (300, 2000)

∆(1600 − 1000) 600


=
− −2
=
∆(300 − 600) 300

Let get the line that passes through the points


Taking any point lets take point (300, 1600)

P − 1600
= −2
Q − 300

Multiply both sides by Q − 300

P − 1600
(Q − 300) −2(Q − 300)
=
Q − 300

P − 1600 =2Q + 600

−2Q + 600 + 1600


P=

−2Q + 2200
P=

Sketching the graph we are going to use the following coordinate. Remember the previous
method we were using to get the coordinates.
When P = 0 the value of Q is 1100 so the coordinates are (1100, 0)
On the other hand when Q = 0 , P is equal to 2200 hence the coordinates are (0, 2200).
We plot the graph as follows.
CHAPTER 3 Simultaneous Linear equations: Application Linear equation to business 49

Figure 2.7

2200

Price −2Q + 2200


P=

Quantity 1100

Supply Function

The equation of supply consists of the ratio or relationships between the quantity supplied and
factors that affect supply such as price. In this course we are going to look at the price as our
major factor that affects supply.

Consider the following function P = aQ + b


CHAPTER 3 Simultaneous Linear equations: Application Linear equation to business 50

Figure 2.8

P aQ + b
=

Price ∆y Change in y
∆y
=a
∆x

Change in x (∆x)

Quantity

The supply curve move from bottom left to upwards right as shown by the figure 2.8 above. The
supply curve has a positive slope coefficient that is a

Example 3.6

In a Datacom offer the sales manager observed that when the price of a flash disk 1GB was fixed
at KES 400 the quantity supplied was 250 and when the price changed to KES 250 the quantity
supplied was 200. Assuming that price was the only factors that affected supply and other
factors remains constants.

Calculate
the slope
Obtain the equation that passes through these points.
Sketch the graph of the equation obtained.
Hence find the value of
i) P when Q = 400
CHAPTER 3 Simultaneous Linear equations: Application Linear equation to business 51

ii) Q when P = 650

Solution

We get the gradient by

∆(400 − 250) 150


= =3
∆(250 − 200) 30

We obtain the equation passing through points (400, 250) and (250, 200) by taking (Q, P) and
any point as follows.
∆( P − 250)
=3
∆(Q − 200)

Multiply both sides by Q − 200

P − 250 = 3(Q − 200)

P − 250 =3Q − 200

Collect the like terms together by adding on both sides 250

P =3Q − 200 + 250

P 3Q + 50
=

Hence using the equation P = 3Q + 50 we can be able to find the value of P when Q = 400

P 3Q + 50
=

Q = 400

Substitute Q by 400
CHAPTER 3 Simultaneous Linear equations: Application Linear equation to business 52

=P 3(400) + 50

=P 1200 + 50

P = 1250

Also given the value of P = 650 we can obtain the value of Q as follows using the same
equation. P = 3Q + 50
Substitute P with the value given of 650

P 3Q + 50
=

P = 650

650
= 3Q + 50

Subtract 50 on both sides

650 − 50 = 3Q + 50 − 50

600 = 3Q

200 = Q

200 = Q can also be written as Q = 200

Practice Problem

1. Suppose a certain commodity has the following linear demand and supply. When the
price is 7500 the quantity is 100uniits, when the price is 4625 the quantity is 750 units and
when the price is 2525 the quantity is 100 units, when the price is 1525 the quantity is 200
units

a) Obtain the linear equation that go through the points given above, clearly explain
which is the demand and supply curve.
b) Equilibrium price and quantity.
CHAPTER 3 Simultaneous Linear equations: Application Linear equation to business 53

2. At a unit price of sh. 16,000, the demand of a product is 300 units and at a unit price of
sh.48,000 the demand is 100 units. At a unit price of sh. 30,000 the supply is 550 units
and at a unit price of sh. 50,000, the supply is 650 units.

Determine the market equilibrium point

3. The quantity demanded per month of product is 250 units when the unit price KES 1400.
The quantity demanded per month is 1000 units when the unit price is KES 1100. The
suppliers will market 750 units if the price is KES 600 or lower but will supply 2250 units
if the price is KES 800. Both the demand and supply functions are known to be linear

a) Find the demand equation


b) Find the supply equation
c) Find the equilibrium price and quantity
d) Illustrate the demand and equation on a graph paper

4. Recently the government liberalized the consumer prices of maize. For the last six
months, the Economic Institute have been keeping the average monthly price and
quantity of maize consumed and supplied per bag in Kenya, The table shows this data;

Price (KES) Quantity Demanded (Kg) Quantity supplies (Kg)


15 6 20
14 8 18
13 10 16
12 12 14
11 14 12
10 16 10

a) Obtain the demand and supply equation


b) Determine the equilibrium price and quantity

5. The sales manager of Akamba Wood Carvings Ltd finds that he can sell 45 pieces per
week at a price of KES 80 per item, but if he changes the price to KES 100 per item he
can sell only 40 pieces per week. The production manager determines that the total
production cost of 45 items per week is KES 3,000 and that of 40 items per week is KES
2800
CHAPTER 3 Simultaneous Linear equations: Application Linear equation to business 54

a) Assuming that the unit price is linear function of the quantity sold determine the
profit function.
b) Assuming the total production cost per week is a linear function of the quantity
produced, determine the cost function.

c) Calculate the level of sales that will maximize the weekly revenue.

d) Calculate the level of sales that will maximize the weekly profit

e) Determine the corresponding profit per item sold the weekly profit maximized.
CHAPTER 3 Simultaneous Linear equations: Application Linear equation to business 55

Equilibrium

This is the state where the supply and demand are equal in other words it occurs when quantity
demanded is equal to the quantity supplied.
To get the equilibrium price and quantity we just equate the two linear functions of supply and
demand.
A graphical representation would look like one below.

Figure 2.9

Supply equation

Price Equilibrium
P

Demand equation

Quantity

Example 3.7

Suppose a commodity A has the following linear functions.


Given the price is KES 9500 the quantity is 3000 units also when the price is KES 6625 the
quantity is 2750 units.
When the price is KES 4525 the quantity is 2100 units. Also when the price KES 2525 the
quantity is 2200 units.
CHAPTER 3 Simultaneous Linear equations: Application Linear equation to business 56

Required
a) Obtain the linear functions that passes through the points
b) Identify which of the two equations is demand and supply functions.
c) Calculate the equilibrium price and quantity
d) Sketch the graph of the equations.

Solution

We first obtain the change in price and change in quantity.

9500 − 6625 2875


= = 11.5
3000 − 2750 250

Taking any of the point we get

P − 9500
= 11.5
Q − 3000

Multiply both sides by Q − 3000 to obtain

P − 9500 = 11.5(Q − 3000)

P − 9500 = 11.5Q − 34500

Add 9500 both sides to get the following equation

P = 11.5Q − 34500 + 9500

=P 11.5Q − 25000

The above is supply function since the Q coefficient 11.5 is positive thus we can write this one as

P = 11.5QS − 25000

Taking the other points in part (ii) we can obtain the slope just as we did in the first part.
CHAPTER 3 Simultaneous Linear equations: Application Linear equation to business 57

∆(4525 − 2525) 2000


= = −20
∆(2100 − 2200) − 100

Taking any point we can obtain the line as follows

P − 4525
= −20
Q − 2100

Multiply both sides by Q − 2100 to obtain the following equation

P − 4525 =
−20(Q − 2100)

P − 4525 =
−20Q + 42000

P − 4525 + 4525 =
−20Q + 42000 + 4525

−20Q + 46525
P=

The equation is a demand function since the coefficient of Q is negative that means it slopes
from top left to bottom right and can be expressed as follows.

P = −20QD + 46525

With our equations we can get the equilibrium price and quantity by equating them both.

P = −20QD + 46525 = P = 11.5QS − 25000

Collect the like terms together

P − P − 20Q − 11.5Q =−25000 − 46525

−31.5Q =
−71525

−71525
Q=
−31.5

Q = 2271
CHAPTER 3 Simultaneous Linear equations: Application Linear equation to business 58

By substituting to any of equation of your choice we get the value of P

P = 11.5QS − 25000

P =11.5 × 2271 − 25000

P = 1116.5

The price is KES 1116.50 and the quantity is 2271 units,


When plot the graph of the two linear equation it appears as follows.

Example 3.8

Suppose a certain commodity has the following linear demand and supply
When the price is 7500 the quantity is 100uniits, when the price is 4625 the quantity is 750 units
and when the price is 2525 the quantity is 100 units, when the price is 1525 the quantity is 200
units
CHAPTER 3 Simultaneous Linear equations: Application Linear equation to business 59

Obtain the linear equation that go through the points given above, clearly explain which is the
demand and supply curve.

Equilibrium price and quantity.

Solution

Change in price ∆P
a) =
Change in quantity ∆Q

∆P 7500 − 4625 2875


= = = 11.5
∆Q 1000 − 750 280

P − 7500
= 11.5
Q − 1000

P − 7500= 11.5 ( Q − 1000 )

P − 7500 = 11.5Q − 11500

P =11.5Q − 11500 + 7500

=P 11.5Q − 4000

The above is supply function because of the positive co-efficient of Q which is 11.5

Change in price ∆P
=
Change in quantity ∆Q

∆P 2525 − 1525 1000


= = = −10
∆Q 100 − 200 −100

P − 1525
= −10
Q − 200

−10 ( Q − 200 )
P − 1525 =
CHAPTER 3 Linear equations: Application Linear equation to business 60

P − 1525 =
−10Q + 2000

−10Q + 2000 + 1525


P=

−10Q + 3525
P=

This is demand function because of a negative coefficient of -10

b).At equilibrium the demand is equal supply

11.5Q − 4000 =
−10Q + 3525

11.5Q + 10Q = 3525 + 4000

21.5Q = 7525

Q = 350

=P 11.5Q − 4000

=P 11.5 ( 350 ) − 4000

=P 4025 − 4000

P = 25

The equilibrium price is 25 and the quantity is 350 units

Example 3.9

At a unit price of KES 16,000, the demand of a product is 300 units and at a unit price of KES
48,000 the demand is 100 units. At a unit price of KES 30,000 the supply is 550 units and at a
unit price of KES 50,000, the supply is 650 units.
Determine the market equilibrium point
CHAPTER 4 Linear equations: Application Linear equation to business 61

Solution
Demand function

∆P 48000 − 16000 32000


= = = −160
∆Q 100 − 300 −200

P − 48000
= −160
Q − 100

−160 ( Q − 100 )
P − 48000 =

P − 48000 =
−160Q + 16000

P=
−160Q + 64000

Supply demand

∆P 50,000 − 30,000 20,000


= = = 200
∆Q 650 − 550 100

P − 30,000
= 200
Q − 550

P − 30,000 =200 ( Q − 550 )

P − 30,000 =200Q − 110,000

P=
200Q − 110,000 + 30,000

=P 200Q − 80,000

At equilibrium the supply is equal to demand


CHAPTER 3 Simultaneous Linear equations: Application Linear equation to business 62

200Q − 80,000 =
−160Q + 64000

200Q + 160Q = 64000 + 80000

360Q = 144000

Q = 400

=P 200 ( 400 ) − 80,000

=P 80000 − 80000
P=0

Practice Problem

The demand and supply equations of a particular goods are given by

−12QD + 360
P=

P QD + 87
=

Where P, QS and QD represent the price, quantity supplied and quantity demanded
respectively.
Calculate the equilibrium price and quantity.
Find the value of P when the quantity of supply changes to 300
ALGEBRA
CHAPTER 4
Learning Objectives

At the end of this section the student should be able to:


• Identify the symbols <,>, ≤, and ≥
• Operating of inequalities.
• Multiplying of brackets.
• Operations of fractions that is multiplying and dividing , adding and
subtracting.
• Operation of algebraic fractions

Introduction

A number line which we met at the beginning of the chapter can be used to decide whether a
number is greater than or less than.
The symbols
< means it is less than
> means it is greater than
≤ means it is less than or equal to
≥ it means it is greater than or equal to
Consider the number line below

Practice Problem

Classify each of the statement as true or false

8 ≥ -3 d) -2 ≥ -5 g) 7 > 0 > 12

15 ≤ 13 e) -7 ≤ 6 ≤ 10 h) -5 ≥ 0
CHAPTER 4 Algebra 64

-15 < -21 f) 4 > -4 > -8 i) 0 ≤ -12

Simple Linear inequalities


Practice Problem

x>5 c) x < −3 e) y ≥ 16

y ≤3 d) y > −32 f) y ≤8

Compound Linear inequalities


Practice Problem

9 < 10 < 15 b) 3 > −2 > −4 c) 7 < x ≤ 12

Note
One can convert two simple linear inequalities into a compound inequality

Example 4.1

x < 6 and x ≥ 4 ⇒ 4≤ x <6

x ≤ 3 and x ≥ −7 ⇒ −7≤ x ≤3
CHAPTER 4 Algebra 65

Practice Problem

Write as single compound inequality


2
6≥x and x ≥ −4 b) x< and x ≥ −5
3

c) x + 1 > 2 and 2x ≤ 7 d) x − 4 ≥ −6 and x + 1 < 10

Inequalities and number line

Note
For < and > circle the number

For ≤ and ≥ shade the circle

Practice Problem
Illustrate the following sets on a number line

a) x < 7 b) − x > −6 c) x ≥ −10

d) − x ≤ −6 e) x ≥ 2x − 6 f) x > −12

Illustrate the following sets on a number line

1≤ x ≤7 b) 0 < x ≤ 8 c) −3 < x ≤ 8

d) − 7 < x ≤ −1 e) 3≤ x ≤9 f) 0 < x < 14

g) −3< y <7 h) 0≤y ≤6 i) 1< y <3


CHAPTER 4 Algebra 65

Solving linear inequalities


Example 4.2
Solve
2x − 4 < 2

1
3− x ≥6
5

Solution
2x – 4 < 2

2x < 6
X <3

1
3− x ≥6
5

1
− x ≥9
5
− x ≥ 45
x ≤ −45

Note
Dividing the unknown by a negative value, the inequality sign changes.

Practice Problem

Solve the following inequalities and illustrate the solution on a number line

a)8 – 4x ≤ 2 b) 5 + 3x ≤ 8 + 4x
CHAPTER 4 Algebra 67

c) 3x – 7 < 2 d) 3x – 2 ≥ x – 6

3(2x-5) < 4x f) 2(5x – 3 ) ≥ 3(4x – 7)

1 1
x +1≥ 5 h) 3 − x ≤ 5
2 2

Solving simultaneous linear


Example 4.3

Solve

3x + 1 > 13
2x – 4 ≤ 8

Solution

3x + 1 > 13 ⇒ 3x > 12, x>4

Again 2x – 4 ≤ 8 ⇒ 2x ≤ 12

⇒4 < x ≤ 6

Solve

-6 < 1 – 2x ≤ 8
CHAPTER 4 Algebra 68

Solution
The inequality can be reduced to two inequalities that is – 6 < 1 – 2x and 1 – 2x ≤ 8 and then
solved separately.

– 6 < 1 – 2x
-7 < -2x
3.5 > x
Again
1 – 2x ≤ 8
-2x ≤ 7
x≥ 3.5
⇒ -3.5 ≤ x 3.5

Practice Problem

Solve the following simultaneous inequalities

x + 7 > 53 x+5>3
a) b)
x − 10 < 3 x −1 ≤ 5

1
x −3≤1 x +6>3
c) 2 d)
2x − 3 ≤ 5
x +7>3

1 1
e) 4 x + 2 ≤ x + 5 < 2 x + 9 f) x +1< x +2 ≤ x +2
2 8

Representing the following inequalities on a Cartesian Plane


Note
For < and > use broken line

For ≤ and ≥ use continuous line

Always shade the unwanted region


CHAPTER 4 Algebra 69

Practice Problem

Represent the following linear inequalities on a Cartesian plane;

a) x≥4 b) 3 < x < 7

c) y > −3 d) 0 ≤ y < 6

e) x ≥ 12 f) −1 ≤ x ≤ 4

g) x ≥ 4, y < 2 h) y < 0

Represent the following linear inequalities on Cartesian plane

a) 2 x + y > 2 b) 5x + 2y ≤ 4

c) x + y ≥ 0 d) 8=
x 2y ≥ 6

e) 3=
x 4y ≤ 6 f) 3y − x ,5

g) y − x ≤ 2 g) 12=
x 3y > 9

Intersecting Regions

Represent the following on a Cartesian plane

x= y > 0 y+x<4
x <3 b) y > 2
y>0 x +2 > 0
CHAPTER 4 Algebra 70

2x + y < 5 2x + y < 6
c) y < x + 3 d) x ≤ 2
y>0 y≤5

4 x − 3y < 6
y−x<0
e) x < 3 f)
y<5
y ≤6

Application of linear inequalities

The manager of caribou café has sufficient money to buy 100 crates of soft drinks of type x and
y. He wants to buy at least twice as many crates of type x as type y. he wants to buy maximum
80 crates of type x and at least 10 crates of type y.
Write down all the inequalities based on these facts.
Represent them on a graph.
The profit on a crate of type x is KES 120 and that of type y is KES 100. Find the number of crates
of each type that he should buy to make maximum profit and calculate this maximum profit.

Solution

The inequalities obtained are;

x + y ≤ 100................................................................... ( i )
x ≥ 2y.......................................................................... ( ii )
x ≤ 80.......................................................................... ( iii )
y ≥ 10........................................................................... ( iv )
We need to get the points for equation (i) and (ii) because the rest can be drawn directly
CHAPTER 4 Algebra: Application of linear inequalities 71

x + y ≤ 100................................................................... ( i )

points x y
0 100
100 0

x ≥ 2y.......................................................................... ( ii )

points x y
20 10
40 20

100 x ≤ 80

80

60
x ≥ 2y
40

20
y ≥ 10

x + y ≤ 100
20 40 60 80 100

Profit
= 120 x + 100y
A = 120 ( 20 ) + 100 (10 ) = 3400
B = 120 ( 80 ) + 100 (10 ) = 10600
C = 120 ( 80 ) + 100 ( 20 ) = 11600
D = 120 ( 66 ) + 100 ( 33) = 11220

He should buy 80 crates of x and 20 crates of y to give a maximum profit 11600


CHAPTER 4 Algebra: Application of linear inequalities 72

Note always take the highest profit

Practice Problem

1. For an in-service course in business management, at least four but not more than 9

workers are to be chosen. The ratio of the number of male workers to the number of

female workers must be less than 2:1 and there must be more males than females

2. A car park is to be laid for x matatus and y buses. Matatus are allowed 10 sq. meters of

space and buses 20 sq. metres, and there is only 500 sq meters available. Not more than

40 vehicles are allowed at a time. There are always both types of vehicles parked and at

the most 15 buses are allowed at a time. Write down all the inequalities based on these

facts. The parking charge for the matatus is KES 150 and that for the bus is KES 200 per

day. Find how many vehicles of each type should be parked on the lot in order to obtain

maximum income and calculate this maximum income

3. The manager of a company wishes to buy two types of machines A and B. Machine A

costs 1000 and B costs 750.he must buy at least one of each type. The total expenditure

on the machine is not to exceed 7500.machine A helps him cut down hi production cost

by KES 15 a week and B by KES 10 a week. How many of each type must he buy to make

a maximum savings?

4. Suppose you are the sport manager and you have been requested to build a rectangular

playing field x metres long and y metres wide. The perimeter of the field must be more

than 500 m but less than 700m. The length of the field must be less than twice its width
CHAPTER 4 Algebra: Application of linear inequalities 73

and the area of the field must not be more than 2500 square metres. Write down

inequalities to represent this information.

5. A farmer has 70 acres of land available for growing maize and beans. The cost per

hectare for growing maize is KES 3000 and that for beans is KES 2000 and he has only

KES 18000. The labour per ha is 2 man-days for the maize and 4 man-days for the beans

and a total of 240 man-days of labour is available.

6. The manager of a theatre wishes to divide the seats available into two classes A and B.

There are not more than 120 seats available; there must be at least twice as many B

class seats as there are A class seats; Class A seats are priced at KES 300 each and class B

at KES 200 each and at least KES 20000 should be collected at each show to meet the

expenses.

a) Write down inequalities and represent them on a graph


b) Find the number of seats for each class that will give maximum profit
c) Calculate maximum profit
d) Find the least number of seats that must be sold in order to incur no loss.
7. A factory operates on two types of machines as shown below;

Machine X Machine Y Maximum


available
Floor space required 2 3 18
Number of men needed to 4 3 24
operate it
There are more machines of type X than those of type Y. if the profit from using machine
X is KES 400 per hour and that from using machine Y is KES 800 per hour, find graphically,
CHAPTER 4 Algebra: Application of linear inequalities 74

the number of machines of each type that should be in sue to give maximum profit per
hour and find this maximum profit.

8. A farmer has 50 acres of land which he plans to plant maize and beans. Each acre of
maize requires 6 people and each acre of beans require 2 people to work on it. He has at
least 60 people available. He must plant at least 10 acres of beans. The profit on maize is
KES 2000 per acre. If he plants x acre of maize and y acres of beans.
a) Write down 4 inequalities of x and y to describe his plan
b) Represent the 4 inequalities graphically
c) Determine the acreage for each crop for maximum profit.
BINOMIAL
CHAPTER 5 THEOREM
Learning Objectives
At the end of this section the students should be able to:

 Use Pascal Train

 Multiplication of the variables

 Use binomial formulae

Given the following expression ( x + y ) n n can take various numbers such as


n = 1, 2,3, 4,5, 6, 7,...n we can expand this as listed below.

By taking the products of expansion

By taking the coefficient of x and y we obtain Pascal's triangle


CHAPTER 5 Binomial theorem 76

1 1
1 2 1
1 3 3 1
1 4 6 4 1
1 5 10 10 5 1

This can be obtained by adding

1 1
1 2 1
1 3 3 1
1 4 6 4 1
1 5 10 10 5 1
1 6 15 20 15 6 1

Obtain the six pattern by adding as below.

1 5 10 10 5 1
1 5 10 10 5 1
1 6 15 20 15 6 1

The above is called the Binomial Theorem

The Binomial theorem is expressed in the following terms;

There are forms of binomial expansion that is positive integral values of the form ( x + y ) n and
negative integral values of the format ( x − y ) n
Expanding using the positive integer
CHAPTER 5 Binomial theorem 77

Example 5.1

Expand using binomial theorem


Getting the coefficient from the Passcal's triangle as follows

Multiply the corresponding coefficient with corresponding values of x and y

Example 5.2

Expanding ( p − q )5 a negative integer using Binomial theorem

Solution

Get the coefficients from the Pascal's triangle of power 5


CHAPTER 5 Binomial theorem 78

Multiplying

Remember every where q has an odd power it supposed to be negative

Example 5.3

Using Binomial expand (4 + 6 x)5

Solution

When we are expanding the form ( x + y ) n with x = 4 , y = 6 x and n = 5 thus we can expand
as follows;

By adding the following

We get the answer as follows with the pattern continuing

1 - 7 q + 21q2 -36 q3 + ...


CHAPTER 5 Binomial theorem 79

Example 5.4

Expand (1 + x)3 hence solve the following


(1.2)3
(0.9)3

Solution

Obtaining the coefficients of power 3 from the Pascal's triangle 1 3 3 1


(1 + x) 3 =+
1 3x + 3x 2 + x 3

With (1.2)3 it can be written in the following form (1 + 0.2)3

This means that thus everywhere there is x in our expression we substitute with
0.2
(1 + 0.2)3 =+
1 3 x + 3 x2 + x3
x = 0.2
1 + 3(0.2) + 3(0.2) 2 + (0.2) 3
1 + 0.6 + 0.12 + 0.008
= 1.728

With (0.9)3 this can be written (1 − 0.1)3


This means that x = −0.1
(1 - 0.1) 3 =+
1 3x + 3x2 + x3
x = -0.1
1 + 3( -0.1) + 3(-0.1) 2 + (- 0.1)3
1 - 0.3 + 0.03 - 0.001
= 0.729
CHAPTER 5 Binomial theorem 80

Practice Problem

Expand the following


(a) (3 x + y )7
(b) (2 p − 3q )10
2 6
(c) (m − )
m
Using Binomial expansion expand the following (1 + 4 x)5 . Hence evaluate the following (1.004)5
PERMUTATIONS
CHAPTER 6
& COMBINATIONS
Learning Objectives
At the end of this section the students should be able to:
 Use the n! (factorial)
 Multiplication of permutation
 Express in factorial notation
 Express in factors
 Simplification of permutation

Let think of numbers 1, 2, and 3 if these numbers were written in a vertical way or column way
we would obtain six different arrangements as listed below.
123
132
213
231
312
321
From the above arrangement, each arrangement is a possible or likely permutation of the
numbers 1. 2 and 3 and thus we can say that there are six permutations that we can obtain from
the three numbers.
In general there three operations that can be done as follows
This can be written in as for any natural number n ;
n ! = n (n -1)(n - 2)...(3)(2)(1)

n ! is read as n factorial
Thus given 7! = 7 × 6 × 5 × 4 × 3 × 2 × 1 the same case to 4! = 4 × 3 × 2 × 1
CHAPTER 6 Permutation & Combinations 82

Example 6.1

Evaluate
8!× 4
6!
9!
3!× 3
10!
2!× 5!

Solution

8!× 4 8 × 7 × 6 × 5 × 4 × 3 × 2 × 1 × (4)
= 224
6! 6 × 5 × 4 × 3 × 2 ×1

10! 9 × 8 × 7 × 6 × 5 × 4 × 3 × 2 ×1
= 20160
2!× 3 3 × 2 × 1 × (3)

10! 9 × 8 × 7 × 6 × 5 × 4 × 3 × 2 ×1
= 15120
2!× 5! 2 ×1× 5 × 4 × 3 × 2 ×1

Note that we can use dot instead of multiplication sign for example in number (1) above
8!.4 8.7.6.5.4.3.2.1.(4)
= = 224
6! 6.5.4.3.2.1

Practice Problem
1 Evaluate:
5! ii) 8! iii) 16! iv) 19!
CHAPTER 6 Permutation & Combinations 83

3!6! 14! 7!
v) vi) (3!) 2 vii) viii)
9! 5!4! 3!(2!) 2
2 Express in factorial notation
6×3
i) 5.4.3.2.1 ii) 7.9 iii) 11.7.9.2 iv)
2 ×1
7× 4×3
v)
3 × 2 ×1
Express in factors

5!.7! ii) 12! + 2! iii) 9! - 3! iv) 19! - 3!(5!)

Combination

n n n!
If   is the number of combinations of n elements taken r at a time then  =
r   r  (n − r )r !

Example 6.2

A club in certain university wants to form committees of 3 people from a group of 8 people.
How many committees can be formed?

Solution

8 8! 8.7.6.5.4.3.2.1
=  = = 56
 3  5!3! 5.4.3.2.1.3.2.1

Practice Problem
In a drama club in certain university wants to form committees of 5 people from a group of
12people. How many committees can be formed?
MATRICES
CHAPTER 7 ALGEBRA
Learning Objectives
At the end of this section the students should be able to:
 Define a matrix order
 Identify types of matrices
 Identify elements in matrix
 Determine the column of a matrix
 Determine the row of matrix

A matrix is a rectangular array of elements. Elements are always real valued numbers.
Example of matrices.

a a12  1 0 
a)  11  b)  
 a21 a22  0 1 

Types of matrices
a) Square matrix

This is the matrix of the order n × n where n represent the rows and n represents the number
of column.

2 4
8 0 
 
CHAPTER 7 Matrices Algebra 85

b) Identity matrix

Where all elements on principle diagonal are one in unity while others are zero.

1 0 0 
1 0  0 1 0 
0 1  and  
  0 0 1 

c) Row vector matrix

Matrix of the order 1× n for example [1 2 4]

d) Column Vector

Matrix of the order n ×1

4
5
 
7 

e) Null vectors

All elements are zero be it in any order of the matrix for our case we use 2 × 2 matrix

0 0 0 
0 0   
0 0  and also 3 × 3 matrix 0 0 0 
  0 0 0 
CHAPTER 7 Matrices Algebra 86

Practice Problem
Identify the following types of matrices

 1 3 −2 0
 3 −1 −7 1 
[2 8 0 3] c) 
 1 −2 −2 5
 
 −3 −4 1 2

1 0 0 
0 0 
0 0  d) 0 1 0 
  0 0 1 
OPERATION OF MATRICES

Learning Objectives
At the end of this section the students should be able to:
 Understand the different types of matrices.
 Add and subtract matrices.
 Use scalar to multiply matrices.
 Operation of matrices multiplication.

1) Additional of matrix

a b  e f
Taking two matrices A   and B g
c d   h 

a + e b + f 
We add A + B =
c + g d + h 
 

Example 7.1

3 3  2 3 3 + 2 3 + 3   5 6 
A  and B   the we add
= A+ B  =   
1 5  6 0 1 + 6 5 + 0   7 5 
CHAPTER 7 Matrices Algebra: Operation of Matrices 88

2) Subtraction of matrix

a b e f
Again if we have two matrices C   and D   we subtract as follows
c d  g h
a−e b− f 
C−D=
 
c − g d − h

 3 3  2 3
Example C   and D   then C-D is given as follows
1 5  6 0

3− 2 3− 3  1 0
C−D  =
=   
 1 − 6 5 − 0   −5 5 

3) Scalar multiplication

When multiplying a matrix by a real number it is referred as to scalar multiplication .The


result is matrix in which the element is multiplied by real numbers
 3 2
 
A =  1 1  And multiplying with a scalar of 2
2 0
 
 2×3 2× 2  6 4
   
2 A =  2 ×1 2 ×1  =  2 2 
 2× 2 2× 0  4 0
   

Example 7.2

Suppose the supply below announces a 10% increase on both the purchase and delivery of
these item what is the new unit cost matrix.
CHAPTER 7 Matrices Algebra: Operation of Matrices 89

6 4 2 
1 1 0.5
 

An 10% increase means that the new unit cost are the former cost plus 0.1 the former cost that
is the new cost are 0.1 former cost matrix .
New cost matrix 1.10A

1.1× 6 1.1× 4 1.1× 2 


1.10 A =  
1.1×1 1.1×1 1.1× 0.5

6.6 4.4 2.2 


= 
1.1 1.1 0.55

4) Matrix transportation

Given a matrix A when column and row of a matrix are interchanged to create a new matrix B
we say that A and B are transpose of each other.
2 1 4 2
A =  2 2 0 2 

 4 1 0 6 
A transpose is written as AT
2 2 4
1 2 1 
A =
T
4 0 0
 
2 2 6

Example 7.3

1 3 2 
X = 
4 1 2
CHAPTER 7 Matrices Algebra: Operation of Matrices 90

X transpose is written as X T

1 4 
X =  3 1 
T

 2 2 

5) Multiplication of matrix

This is a product of two matrix given A is m × n and matrix B is n × p . The matrix product AB is a
m× p
In multiplication of matrices, the number of column of matrix A must be equal to the number of
rows of matrix B.
Example
3 4 
a b c d
Given that matrix X =  2 5  and C = 
e f g h 
 6 10 
Find XC

Before multiplying the matrices we must know the order of each matrix .
Matrix X = 3 × 2 and C= 2 × 4
The two matrixes must be compatible that is the number of column of the
first matrix must be equal to the number of row of the second matrix, the
product of the two matrices will be order of 3 × 4
CHAPTER 7 Matrices Algebra: Operation of Matrices 91

3 4 
2 5  a b c d
  e f g h 
 6 10  

Practice Problem

Given that A, B, C, D and E represent the following matrices:

2 3 1
A =  1 1 1  B = [1 0 6]
 −3 0 −2 

2 3
c=  D=[ −1 −1]
3 2

0 2 1 1
3 0 1 6 
E=
1 1 0 7
 
1 2 2 0

Evaluate and comment on each

a) AT , BT , C T , DT , E T

b) A − B, B − C , D − E

c) A + B, C + D

d) BA, DC
CHAPTER 7 Matrices Algebra: Operation of Matrices 92

Application in Business
Example 7.4

ABC Ltd produces toy at two locations. They have an order to make stuffed animals. The
quantities of each type of animal to be produced at each plant may be expressed as 2 × 3
production p thus,

Pandas Dolls Birds


Nairobi 500 800 1300 
P=
Mombasa 400 400 700 

The amount and type of material (in cubic feet) required to manufacture each of animal may be
represented by 3 × 3 activity matrix A . thus

Plush Stuffing Trim


Pandas  1.5 30 5 
A = Dolls  2 35 8 
 
Birds  2.5 25 15

Determine:
Find the amount of each type of material required for each plant

Solution

Pundas Dolls Birds


1.5 30 5 
Nairobi 500 800 1300  
   2 35 8 
Mombasa  400 400 700 
 2.5 25 15
Nairobi
CHAPTER 7 Matrices Algebra: Operation of Matrices 93

1.5 
[500 800 1300]  2 = ( 500 ×1.5) + (800 × 2 ) + (1300 × 2.5)= 5600
 2.5
30 
[500 800 1300] 35 =

( 500 × 30 ) + (800 × 35) + (1300 × 25) = 75,500
 25
5
[500 800 1300]  8 = ( 500 × 5) + (800 × 8) + (1300 ×15)= 28, 400
15

Mombasa

1.5 
[ 400 400 700]  2 = ( 400 ×1.5) + ( 400 × 2 ) + ( 700 × 2.5)= 3250
 2.5

30 
[ 400 400 700] 35 = ( 400 × 30 ) + ( 400 × 35) + ( 700 × 25) = 43500
 25
5
[ 400 400 700]  8 = ( 400 × 5) + ( 400 × 8) + ( 700 ×15)= 15700
15

A combination the Nairobi and Mombasa matrix we get

plush Stuffing trim


Nairobi 5600 75500 28400 
=
Mombasa 3150 43500 15700 
CHAPTER 7 Matrices Algebra: Operation of Matrices 94

Practice Problem

1. Purity fashion makes trousers at two locations. They have an order to make ladies jeans
trousers.

Pencil Hipstar Tight


Athi River  600 1000 1200 
Industrial area 1400 700 1100 

The amount and type of material in cubic meter required to make each type of trouser may be
represented by 3 by 3 activity matrix A as below.

 Jean fabric Cotton thread 


pencil  1.5 0.3 10 
hipstar  2.5 0.35 5 
 
 3.0 0.25 20 

Find the amount of each type of material required for each plant.

A group operates a chain of filling stations in each of which are employed cashiers, attendants
and mechanics as show.

Types of fillings tation


 L arg e Medium Small 
Cashier  
4 2 1
Attendants  
 12 6 3 
Mechanics  
 6 4 2 

The numbers of filling stations are


INVERSION OF MATRICES

Learning Objectives

At the end of this section the students should be able to:


• Determine where there a matrix is singular or non singular;

• Determine the two by two and three by three identity matrices;

• Calculate the inverse of 2 × 2 matrix;

• Calculate the inverse of 3 × 3 matrix by using cofactors.

• Use the inverse of matrix to solve problems in business world.

• Determine the determinant of a matrix.

• Identify singular or non singular matrix.

Let consider a square matrix which have the same number or rows and columns. When we
multiply
AI = A and IA = A

Where I is given below

1 0 
I=  
0 1 

The matrix is called identity matrix and it is equal to number 1 in ordinary mathematics.
Consider another matrix represented by letter A
Where
CHAPTER 7 Matrices Algebra: Inversion of Matrices 96

a b 
A=  
c d 
To determine the matrix A −1 which is the inverse of A we can perform the following operation.

1 a b 
A −1 =
ad − bc  c d 

In the above we can say that

AA −1 = I and A −1A = I

When we say A −1 is an inverse of A it is as same as the reciprocal of A, but sometimes the


students look at it as very difficult but here is how we obtain the inverse of A given

a b 
A=  
c d 
First we interchange the numbers in the diagonal that is formed by the first number in the
matrix as follows

Interchanging “a”
d b
A=  and “d”
c a 
Second change the signs of the remaining diagonal that is made of the second number. If the
number are positive change signs to negative and vice versa

 d −b  Change signs to
A=  
 −c a  negative

To obtain the determinant we multiply numbers in the first diagonal subtract the numbers in
the second diagonal
ad − bc
Then we are supposed to get the reciprocal of the determinant
1
ad − bc
CHAPTER 7 Matrices Algebra: Inversion of Matrices 97

Here we have obtained a scalar which we are supposed to multiply the matrix that we
interchanged and changed sign
To obtain
1  d −b 
ad − bc  −c a 

When determinant of any matrix is zero that is A = 0 the is said to be singular matrix and when
the matrix has a determinant that of a number or non zero determinant it is said to be non-
singular matrix

Example 7.5

Determine the inverse of the following matrices and comment whether they are singular or non
singular.

2 3
A=  
3 2

6 4 
B=  
3 2

3 7
C=  
2 5

Solution
Let begin by calculating the determinant of matrix A
2 3
det(A) or A = =( 2 × 2 ) − ( 3 × 3) =−
4 9=−5
3 2
We can see that A ≠ 0 thus matrix A has a inverse and is a non-singular matrix
1
Since we have the determinant we need to get the reciprocal of -5 as −
5
Following the steps we need to interchange the first diagonal which is made of numbers 2 and 2
CHAPTER 7 Matrices Algebra: Inversion of Matrices 98

Interchanging 2
and 2
2 3
A=  
3 2

Change the signs in the second diagonal which is made up of 3 and 3

 2 −3 Change signs to


A=  
 −3 2  negative

Finally multiply the above matrix with the scalar matrix obtain

1  2 −3  − 2 5 3 5 
− =
5  −3 2   3 5 2 5

When we multiply A −1A = I that we are supposed to obtain an identity matrix

 − 2 5 3 5   2 3  1 0 
 3 5 2 5  3 2  =  0 1 
    

6 4 
B=  
3 2

Let start by obtaining the determinant

6 4
B= = ( 6 × 2 ) − ( 4 × 3) = 12 − 12 = 0
3 2

From the above operation we can see that B = 0 hence it has no inverse and is
Singular matrix

3 7
C=  
2 5
CHAPTER 7 Matrices Algebra: Inversion of Matrices 99

Calculating the determinant

3 7
C= = ( 3 × 5 ) − ( 7 × 2 ) = 15 − 14 = 1
2 5
Thus this is a non-singular matrix and we can obtain its inverse.
First get the reciprocal of the determinant which is on 1
Second interchange the first diagonal of 3 and 5

5 7 
C=   Interchanging 3
2 3 and 5

Change the signs in the second diagonal of 2 and 5

 5 −7  Change signs to
C=   negative
 −2 3 

Finally multiply the above with the inverse of the determinant

1  5 −7   5 −7 
=
1  −2 3   −2 3 

Practice Problem
Find the determinant of the following and hence the inverse of the following matrices

9 3 
K=  
6 2 

 7 5
L=  
 4 3
CHAPTER 7 Matrices Algebra: Inversion of Matrices 100

 5 5
M=  
 4 3

We have been dealing with square matrix that is 2× 2 now we need to consider a 3× 3 matrix.
The same concept of identity matrix applies but need to consider some technicality that applies.
Considering a matrix A which is 3 × 3 with the following elements

 m11 m12 m13 


A =  m 21 m 22 m 23 
 m31 m32 m33 

Since there are nine elements we can obtain nine cofactors from the matrix. We can define
cofactor as the determinant of 2× 2 which is obtained a row and a column as shown below.
To obtain the cofactor of m11 we need to delete the first row and the first column and the
multiplication of the first diagonal minus second diagonal of the remainder gives the cofactor of
m11

 m11 m12 m13 


A =  m 21 m 22 m 23  Delete the first
row and column
 m31 m32 m33 

m 22 m 23
M11 = = ( m 22 )( m33 ) - ( m 23 )( m32 )
m32 m33

We repeat operation until we achieve all the nine cofactors

 m11 m12 m13 


A =  m 21 m 22 m 23  Delete the first row
 m31 m32 m33  and second column

m 21 m 23
=M12 = ( m 21 )( m31 ) − ( m 23 )( m31 )
m31 m31
CHAPTER 7 Matrices Algebra: Inversion of Matrices 101

The row will remain deleted and then move to the line to the last column which is deleted

 m11 m12 m13 


A =  m 21 m 22 m 23  Delete the first row
and third column
 m31 m32 m33 

m 21 m 22
M13 = = ( m 21 )( m32 ) - ( m 22 )( m31 )
m31 m32

To get the cofactors in the second row we delete the second row and the first column.

 m11 m12 m13 


A =  m 21 m 22 m 23  Delete the second
 m31 m32 m33  row and first column

m12 m13
M 21 = = ( m12 )( m33 ) - ( m13 )( m32 )
m32 m33

The second row remains deleted and now we need also to delete the second column.

 m11 m12 m13 


A =  m 21 m 22 m 23  Delete the second row
 m31 m32 m33  and second column

m11 m13
M 22 = = ( m11 )( m33 ) - ( m13 )( m31 )
m31 m33

The second row is still deleted and now we need to delete the third column so as to obtain the
cofactor of M 23
CHAPTER 7 Matrices Algebra: Inversion of Matrices 102

 m11 m12 m13 


A =  m 21 m 22 m 23  Delete the second row
and third column
 m31 m32 m33 

m11 m12
M 23 = = ( m11 )( m32 ) - ( m12 )( m31 )
m31 m32

Finally we perform the same operation in the third row and delete it and delete also the first
column to obtain M 31

 m11 m12 m13 


Delete the third row
A =  m 21 m 22 m 23  and first column
 m31 m32 m33 

m12 m13
M 31 = = ( m12 )( m 23 ) - ( m13 )( m 22 )
m 22 m 23

To obtain M 32 we need to delete the third row and the second column.

 m11 m12 m13 


Delete the third row
A =  m 21 m 22 m 23  and second column
 m31 m32 m33 

m11 m13
M 32 = = ( m11 )( m 23 ) - ( m13 )( m 21 )
m 21 m 23

To obtain M 33 we need to delete the third row and the third column

 m11 m12 m13 


A =  m 21 m 22 m 23  Delete the third row
and third column
 m31 m32 m33 
CHAPTER 7 Matrices Algebra: Inversion of Matrices 103

m11 m12
M 33 = = ( m11 )( m 22 ) - ( m12 )( m 21 )
m 21 m 22

After obtaining the cofactors we are supposed to multiply by the signs

+ − + 
− + −
 
 + − + 

This just as we were doing in a square matrix where we were changing the signs were simply
multiply or adding a prefix of the signs below in the matrix

+ − 
− +
 

Example 7.6

Determine the cofactors of the following matrix

1 1 1 
B =  4 1 2 
 5 3 4 

Solution

Let have the following matrix represent the cofactor matrix

 B11 B12 B13 


B B22 B23 
 21
 B31 B32 B33 
CHAPTER 7 Matrices Algebra: Inversion of Matrices 104

We start by getting the cofactors of B11 by deleting the first row and column and multiply the
diagonals

1 1 1  Delete the first


B =  4 1 2  row and column
 5 3 1 

1 2
B11 = =1(1) − 2 ( 3) =−
1 6 =−5
3 1

To get B12 we delete the first row and the second column

1 1 1  Delete the first row


B =  4 1 2  and second column
 5 3 1 

4 2
B12 = 4 (1) − 2 ( 5 ) =−
= 4 10 =
−6
5 1

B13
is obtained by deleting the first row and the third column and multiply the remainder
diagonally.

1 1 1 
B =  4 1 2  Delete the first row
and third column
 5 3 1 

4 1
B13 = = 4 ( 3) − 1( 5 ) = 12 − 5 = 7
5 3
CHAPTER 7 Matrices Algebra: Inversion of Matrices 105

We now can move to the second row in which to obtain B21 we need to delete the second row
and first column
1 1 1 
B =  4 1 2  Delete the second
row and first column
 5 3 1 

1 1
B21 = =1(1) − 1( 3) =1 − 3 =−2
3 1

Similarly we delete the second row and the second column to get B22

1 1 1 
B =  4 1 2  Delete the second row
 5 3 1  and second column

1 1
B22 = =1(1) − 1( 5 ) =−
1 5 =−4
5 1

To obtain B23 we need to delete the second row and the third column

1 1 1 
B =  4 1 2  Delete the second row
 5 3 1  and third column

1 1
B23 = =1( 3) − 1( 5 ) =3 − 5 =−2
5 3

Finally we move to the third row where first we need to obtain B31

1 1 1 
B =  4 1 2 
 5 3 1  Delete the third row
and first column
CHAPTER 7 Matrices Algebra: Inversion of Matrices 106

1 1
B31 = = 1( 2 ) − 1(1) = 2 − 1 = 1
1 2

B32 is obtained by deleting the third row and the second column

1 1 1  Delete the third row


B =  4 1 2  and second column
 5 3 1 

1 1
B32 = =1( 2 ) − 1( 4 ) =2 − 4 =−2
4 2

B33 is the last cofactor and can be obtained by deleting the third row and column

Delete the third row


1 1 1 
and third column
B =  4 1 2 
 5 3 1 

1 1
B33 = =1(1) − 1( 4 ) =−
1 4 =−3
4 1

 B11 B12 B13 


The cofactor was represented by  B21 B22 B23  and we can replace with the figures we
 B31 B32 B33 
obtained as

 −5 −6 7 
 −2 −4 −2 
 
 1 −2 −3

+ − + 
We need to place a prefix of the following signs  − + − 
 + − + 
CHAPTER 7 Matrices Algebra: Inversion of Matrices 107

 + − +   −5 −6 7   −5 6 7 
 − + −   −2 −4 −2  =  2 −4 2 
    
 + − +   1 −2 −3  1 2 −3

Example 7.7

Determine the cofactors of the following matrix C

2 6 3
C =  −5 1 −2 
 7 −3 1 

Solution

Let have the following matrix represent the cofactor matrix

 C11 C 12 C13 
C = C21 C22 C23 
 C31 C32 C33 

We can combine the finding the cofactors and putting the prefix of the signs

2 6 3 + − + 
 
C =  −5 1 −2  and  − + − 
 7 −3 1   + − + 
The fist sign is +

1 −2
C11 = =1(1) − ( −2 )( −3) =1 − 6 =−5
−3 1
CHAPTER 7 Matrices Algebra: Inversion of Matrices 108

−5 −2
C12 = = − 5 (1) − ( −2 ) 7 =−5 + 14 =9
7 1

−5 1
C13 = =−5 ( −3) − 1( 7 ) =15 − 7 =8
7 −3

6 3
C21 = = 6 (1) − 3 ( −3) = 6 + 9 = 15
−3 1

2 3
C22 = = 2 (1) − 3 ( 7 ) = 2 − 21 =
−19
7 1

2 6
C23 = = 2 ( −3) − 6 ( 7 ) = − 6 − 42 =−48
7 −3

6 3
C31 = = 6 ( −2 ) − 3 (1) = − 12 − 3 =−15
1 −2
2 3
C32 = =2 ( −2 ) − 3 ( −5 ) =−4 + 15 =11
−5 −2

2 6
C33 = = 2 (1) − 6 ( −5 ) = 2 + 30 =32
−5 1

The cofactors are as given below

 −5 9 8 
 15 −19 −48
 
 −15 −11 32 
 + − +   −5 9 8 
 − + −   15 −19 −48
  
 + − +   −15 −11 32 
CHAPTER 7 Matrices Algebra: Inversion of Matrices 109

 −5 −9 8 
=  −15 −19 48
 −15 11 32 

Practice Problem
Find the cofactors of the following matrices

1 2 3
D =  4 5 6 
7 8 9 

 3 −2 −3
E =  3 −3 3 
 4 7 −8

Example 7.8

Now that we have learned how to determine the cofactors we need to get the determinant and
the inverse of a 3 × 3 by
Taking the cofactors of the example we had previously

 −5 6 7 
 2 −4 2 
 
 1 2 −3

We can obtain the determinant in two ways

Method 1

Multiply the corresponding row or column of the original matrix and the cofactor matrix
CHAPTER 7 Matrices Algebra: Inversion of Matrices 110

1 1 1   −5 6 7 
The original matrix B =  4 1 2  and the cofactor matrix  2 −4 2 
 
 
 5 3 1   1 2 −3

When we take the first row of the original matrix, we should also take the first row of the
cofactors matrix and multiply

B =1( −5 ) + 1( 6 ) + 1( 7 ) =−5 + 6 + 7 =8

When we pick every corresponding row or column we are supposed to get the same
determinant of 8.
Let take the first column

B =1( −5 ) + 4 ( 2 ) + 1( 5 ) =−5 + 8 + 5 =8

Method 2

1 1 1 
Taking the same matrix B =  4 1 2  we can use any row or column for our case let use the
 5 3 1 
third row.

1 1 1 1 1 1
B =5 −3 +1
1 2 4 2 4 1

Remember here we must consider the signs since we took the third row we take the signs in the
third row
CHAPTER 7 Matrices Algebra: Inversion of Matrices 111

+ − + 
− + −
 
 + − + 

B = 5 (1) − 3 ( −2 ) + 1( −3) = 5 + 6 − 3 = 8

Example 7.9

Get the determinant of the following matrix

2 6 3
C =  −5 1 −2 
 7 −3 1 
Given the cofactors are

 −5 −9 8 
 −15 −19 48
 
 −15 11 32 

Taking any row or column we are supposed to multiply with the corresponding row or column.
When we take the first row in the original matrix we should also take the first in the cofactor
matrix

C=2 ( −5 ) + 6 ( −9 ) + 3 ( 8 ) =−10 − 54 + 24 =−40

Let check with the remaining rows

C =−5 ( −15 ) + 1( −19 ) + ( −2 ) 48 =75 − 19 − 96 =−40

7 ( 15 ) + ( 3)11 + 32 =
C =− −105 + 33 + 32 =
−40
CHAPTER 7 Matrices Algebra: Inversion of Matrices 112

Similarly we can check the columns

C =2 ( −5 ) + ( −5 )( −15 ) + 7 ( −15 ) =−10 + 75 − 105 =−40

C =6 ( −9 ) + 1( −19 ) + ( −3)11 =−54 − 19 + 33 =−40

3 ( 8 ) + ( −2 ) 48 + 32 =
C= 24 − 96 + 32 =
−40

Method 3

We can get the determinant by the third method as below

2 6 3
C =  −5 1 −2 
 7 −3 1 

Add

Example 7.10

We can use the matrix method to solve a linear system


4x + 5 y =
60
3x + 3 y =
39

Solution
Let first arrange it into matrix form

4 5 1  3 −5  x  1  3 −5 60 


3 =
 3 −3  −3 4   y  −3  −3 4  39 
1 0   x   1 −5 3 60 
0 =
 1   y   −1 4 3  39 
CHAPTER 7 Matrices Algebra: Inversion of Matrices 113

First get the determinant

Det 4 ( 3) − 5 ( 3) =
12 − 15 =
−3

Interchange the first diagonal

3 5 
3 4 
 

Change the signs

 3 −5
 −3 4 
 

 4 5 1  3 −5  x  1  3 −5 60 


 3 3 −3  −3 4   y  = −3  −3 4  39 
       

1 0   x   −1 5 3  60 
0 1   y  =  1 −4 3 39 
     

 x  ( −1× 60 ) + ( 5 3 × 39 ) 
 y  =  1× 60 + −4 3 × 39 
  ( ) ( )

 x   −60 + 65
 y  =  60 − 52 
   

 x  5 
 y  = 8 
   

x=5

y=8
CHAPTER 7 Matrices Algebra: Inversion of Matrices 114

Example 7.11

When solving for 3× 3 matrix we need the cofactors

x+ y+z = 6
4x + y + 2z = 12
5x + 3 y + z =14

Solution

Let first write it in a matrix form

1 1 1   x   6 
 4 1 2   y  = 12 
    
 5 3 1   z  14 

The second step is getting the inverse where we need to find the cofactor matrix
The cofactor of this matrix had already been calculated as

 −5 −6 7 
 −2 −4 −2 
 
 1 −2 −3
When we take the cofactor and place the prefix of the signs + or – the matrix is referred as
adjugate matrix

 −5 −6 7   + − + 
 −2 −4 −2   − + − 
  
 1 −2 −3  + − + 

To get
CHAPTER 7 Matrices Algebra: Inversion of Matrices 115

 −5 6 7 
 2 −4 2 
 
 1 2 −3
To solve for system we need to transpose the adjugate matrix. Transposing is changing rows
into columns or column into rows as seen earlier in the introduction of the matrices

 −5 6 7 
 2 −4 2 
 
 1 2 −3

When transposed we get

 −5 2 1 
 6 −4 2 
 
 7 2 −3

After transposing a matrix we get a matrix called an adjoint matrix


Now that we had the determinant is 8

 −5 2 1  1 1 1   x   −5 2 1   6 
1      1
 6 −4 2   4 1 2   y  =  6 −4 2  12 
8 8
 7 2 −3  5 3 1   z   7 2 −3 14 

1 0 0   x   −5 2 1   6 
0 1 =    1
 0  y   6 −4 2  12 
8
0 0 1   z   7 2 −3 14 

 x  −5 6 7   6 
 y 1   
  8  2 −4 2  12 
=
 z   1 2 −3 14 
CHAPTER 7 Matrices Algebra: Inversion of Matrices 116

 x  ( −5 × 6 ) + ( 6 × 2 ) + (1×14 ) 
 y=  1  6 × 6 + −4 ×12 + 2 ×14 
  8 ( ) ( ) ( )
 z  ( 7 × 6 ) + ( 2 ×12 ) + ( −3 ×14 ) 

 x  −30 + 12 + 14 
 y=  1  36 − 48 + 28 
  8 
 z   42 + 24 + 42 

 x 8
 y  = 1 16 
  8 
 z   24 

 x 88 1 
 y =  2
=
  16 8   
 z   24 8  3 

=x 1=y 2=z 3

Example 7.12

Using the matrix method solve for the linear system


5 x − 10 y − 5 z =
−25
−2 x + 3 y − 4 z =60
−6 x − 7 y − 8 z =−20

Solution

Arrange it in matrix form


CHAPTER 7 Matrices Algebra: Inversion of Matrices 117

 5 −10 −5  x   −25


 −2 3 −4   y  =  
     60 
 −6 −7 −8  z   −20 

Then let’s get the cofactors for the matrix

 5 −10 −5
 −2 3 −4 
 
 −6 −7 −8

3 −4
A11 = = −52
−7 −8

−2 −4
A12 = = −8
−6 −8

−2 3
=A13 = 32
−6 −7

−10 −5
=A 21 = 45
−7 −8
5 −5
A 22 = = −70
−6 −8

5 −10
A 23 = = −95
−6 −7

−10 −5
=A 31 = 55
3 −4
CHAPTER 7 Matrices Algebra: Inversion of Matrices 118

5 −5
A 32 = = −30
−2 −4

5 −10
=A 33 = 5
−2 3

Now we can write the cofactor matrix


 −52 −8 32 
 
 45 −70 −95
 55 −30 5 

We apply the prefix of the signs

 −52 −8 32  + − +   −52 8 32 
  − + − =  
 45 −70 −95    −45 −70 95
 55 −30 5   + − +   55 30 5 

Let get the determinant using the first row of the original matrix and cofactor matrix

5 ( −52 ) + ( −10 )( 8 ) + 32 ( −5 ) =−500

Transpose the cofactor matrix

 −52 −45 55


 8 −70 30 
 
 32 95 5 

 −52 −45 55  5 −10 −5  x   −52 −45 55  −25


1       1 
 8 −70 30   −2 3 −4 =  y  8 −70 30   60 
−500 −500
 32 95 5   −6 −7 8   z   32 95 5   −20 
CHAPTER 7 Matrices Algebra: Inversion of Matrices 119

1 0 0   x   −52 −45 55  −25


0 =   1   
 1 0   y  −500  8 −70 30   60 
0 0 1   z   32 95 5   −20 

 x  −52 −45 55  −25


  1   
= y  −500  8 −70 30   60 
 z   32 95 5   −20 

 x ( −52 × −25 ) + ( −45 × 60 ) + ( 55 × −25 ) 


  1  
= y  −500  ( 8 × −25 ) + ( −70 × 60 ) + ( 30 × −20 ) 
 z   ( 32 × −25 ) + ( 95 × 60 ) + ( 5 × −20 ) 

x  −2500 
  1  
 y  −500  −5000 
=
 z   5000 

 x   −2500 −500 
 y =  
   −5000 −500 
 z   5000 −500 

 x  5 
 y  =  10 
   
 z   −10 

x= 5 y = 10 z = −10
CHAPTER 7 Matrices Algebra: Inversion of Matrices 120

Practice Problem
Solve for x using matrix method

3x + 3 y + z =7
−2 x + 10 y − 5 z = 5
x − 11 y + 2 y = 12

x + 2 y + 3z = 7
2 x + 5 y + 3z =19
2 x + 6 y + 8z = 24
APPLICATION OF MATRICES

Learning Objectives

At the end of this section the students should be able to:


Understand what is meant by input-output analysis through;
• Determining the technical coefficient matrix
• Determine the final demand given the total output matrix
• Determine the total output matrix given final demand
• Use Leontief model to solve practical business problem
• Understand the transition matrices
• Use Markov model to solve problem in business world

Input-output model is an application of matrix that has been successfully used to describe
mathematics model used to analyze how industries depend on each other in an economic
system. Here when we talk about the industry we are referring to the industries and consumers
or households.
Industries depend on each other by some industry using industries product as raw materials as
well as for external use by the consumers.
The output produced by the industries for consumers is called the final or external demand
Industry use each others output and we refer this as the intermediate output
Input-output analysis can be seen as the flow of industries that produce and use goods. Some of
the goods produced are used in the industry processes that are consumed internally and some
are used by the outsiders that is externally.
The input-output model most popular because it can handle large number of variables as
required to describe an economic situation.
CHAPTER 7 Matrices Algebra: Application of Matrices 122

Example 7.13

Let’s consider two industries that is industry 1 and Industry 2 and that one unit worth of output
of industry 1 requires 20% worth of industry 1 and 30% of industry 2. One unit worth of output
of industry 2 requires 60% industry 1 and 40% of industry 2
This can be presented by the figure 1.1 below

Figure 1.1
Output
Industry 1 Industry 2
Input Industry 1 0.2 0.5
Industry 2 0.3 0.4

Solution

The above information can be represented inform of a matrix A by just writing it the way it is as
given below
Technology matrix or
0.2 0.5 technical coefficient matrix
A=  
 0.3 0.4 

The above matrix A is called the technology matrix or technical coefficient matrix
In general if we assume that 1200 units is required in industry 1 rather than one unit then we
can express the above as follows.

0.2 ×1200 =
240 Units of industry 1
0.3 ×1200 =
360 Units of industry 2

When we look at the output some has been left out for internal demand which amount to 600
units.
CHAPTER 7 Matrices Algebra: Application of Matrices 123

Again if we consider 1000 units are required in industry 2 then we can calculate the need
internal demand.
0.5 ×1000 =
500 Units of industry 1
0.4 ×1000 =
400 Units of industry 2
The remainder which is 100 units is used by the industry 2 for their internal demand.

Example 7.14

In general given the following matrix A

a a12 
A =  11
 a13 a14 

Let assume that this represent the technical matrix for two industries

Solution

Then let have d1 and d 2 represent the final demand for the two industries
The final demand for the first industry can be expressed as follows

a11 + a12 + d 2

When we use x1 is the total output that meets the requirement then we express

x1 = a11 + a12 + d 2

When we have x1
x1 = a21 + a22 + d 2
CHAPTER 7 Matrices Algebra: Application of Matrices 124

When we express this in matrix notation then we can express the above as follows

 x1   a11 a12   x1   d1 
= x  a +
 2   21 a22   x2   d 2 

The above illustration can be used to demonstrate the equation

x Ax + d
=

Where x is the total output vector and d is the final demand vector
In the introduction we noted that the input-output can be used to represent various variables
In general if we had three industries as given below with the output of x

ai1 x1 Used by industry 1


ai 2 x1 Used by industry 2
ai 3 x1 Used by industry 3

Now when we have nth number of industry we will nave

ain xn Used by the nth number of the industry

When we have a combination of these industries it can be represented in the matrix notation as
Where i = 1, 2,3,...n

 x1   a11 a12 a13 . . . a1n   x1   d1 


x  a  x  d 
 2  21 a22 a23 . . . a2 n   2   2
 x3  =a31 a32 a33 . . . a3n   x3   d3 
.   . . . . .  . 
 ..   .. .
.
.
.
.
. . 
.  .. 
 xn   an1 an 2 an 3 . . . ann   xn   d n 

Example 7.15
Suppose we consider a two sector economy Agriculture and Transport and assume that their
production in capacity. Agriculture total production is used as below
400 units is used by Agriculture, 900 units by transport and 300 units is left for external
consumers
Transport production total output is 400 units is used by Agriculture, 300 units by transport
while 500 units is left foe external consumers
CHAPTER 7 Matrices Algebra: Application of Matrices 125

Determine the technical coefficient matrix and total internal consumption


If the final demand is 100 units for Agriculture and 80 units for transport sector

Solution
We can represent the above information into a table form

Figure 1.1

Output
Agriculture Transport Final Total
demand output
Agriculture 800 400 400 1600
Input

Transport 400 600 200 1200

The technological matrix can be obtained by dividing the first column by 1600 and the second
column by 1200

800 1600 400 1200 


A= 
 400 1600 600 1200 

The technical coefficient matrix will be obtained as

 0.5 0.25
A= 
0.25 0.5 

The internal consumption can be obtained


Ax = Total internal consumption

 0.5 0.25
A= 
0.25 0.5 
CHAPTER 7 Matrices Algebra: Application of Matrices 126
1600 
x= 
1200 

 0.5 0.25 1600  1100 


0.25 0.5  1200  = 1000 
    

The internal consumption is 1100 units for Agriculture and 1000 units by Transport
If the final demand is written in matrix notation

100 
d= 
 80 

Let’s get the Leontief inverse from what we call the Leontief Model
I−A

1 0   0.5 0.25  0.5 −0.25


0 1  − 0.25 0.5  =
 
     −0.25 0.5 

Det (A) = (0.5 × 0.5) − (0.25 × 0.25) = 0.1875

−1
Using x= ( I − A ) d

1  0.5 −0.25 100  30 


=
0.1875  −0.25 0.5   80  15 

1 30  30 0.1875 160 


= =
0.1875 15  15 0.1875   80 

160 
=   The final demand for Agriculture is 160 units and 80 units for Transport
 80 
CHAPTER 7 Matrices Algebra: Application of Matrices 127

Example 7.16
An economy has two sectors A and B, each unit of A requires 0.3 units of A and 0.5 units of B.
Each unit of B output require input of 0.3 units of A and 0.2 units B. Determine the level of
output that will satisfy the final demand of 780 units of A and 520 units of B

Solution

A B
A 0.3 0.5
B 0.3 0.2 

Let’s calculate I − A

1 0  0.3 0.5  0.7 −0.5


0 1  − 0.3 0.2  =
 
     −0.3 0.8 

Det
= ( 0.7 × 0.8) − ( −0.3 × −0.2
= ) 0.5

The Leontief inverse ( I − A )


−1

1  0.7 −0.5
0.5  −0.3 0.8 

1  0.7 −0.5  286 


0.5  −0.3 0.8  182 

1  286  572 
=
0.5 182  364 
CHAPTER 7 Matrices Algebra: Application of Matrices 128

A = 572 units
B = 364 units

Example 7.17

Given the following information

Figure 1.1

Output
X Y Final demand
X 400 400 200
Input

Y 100 150 250

Calculate the technology matrix, assuming that total output is just sufficient to meet the total
internal consumption and final demand

Solution

Total output = Total internal consumption + Final demand


X = AX + D
That is

x Ax + d
=

X; XX = 400 + 400 + 200 = 1000


Y;XY = 100 + 150 + 250 = 500

Technology matrix
 400 1000 400 500 
100 1000 150 500 
 
CHAPTER 7 Matrices Algebra: Application of Matrices 129

0.4 08 
A= 
 0.1 0.3

Example 7.18

Given that an economy of certain country consists of three sectors; Mining (M), Textiles (T) and
Chemicals (C). The input-output matrix for the economy is given by

Figure 1.2

Output
Mining Chemicals Textile
Mining 0.4 0.1 0.1
Textile 0.1 0.4 0.3
Input

Chemicals 0.2 0.2 0.2

Determine the amount of mining goods are consumed in the production of KES 50 million worth
of chemical goods
Determine the total input required to produce one unit of mining products
Determine the amount in shillings required to produce KES 100 millions of chemical products
Among the sectors which one consumes the greatest mining products and which consumes the
least
What is the final output given that the sectors are required to meet an external demand of KES
672 million, KES 224 million and KES 448 from the mining , textile and chemicals sector
respectively
Calculate the total internal consumption to meet the above demand
CHAPTER 7 Matrices Algebra: Application of Matrices 130

Solution
To produce one unit of the chemical goods requires 0.1 units of mining product

M C T
M 0.4 0.1 0.1
C  0.1 0.4 0.3
 
T 0.2 0.2 0.2 

50 × 0.1 =
5 millions

To produce 1 unit of mining products we require

0.4 + 0.1 + 0.2 =


7 units

Chemical goods are required as input for one unit

0.1 + 0.4 + 0.3 =


0.8

Since we need to express in amounts then

0.8 ×100 =
80 millions

Mining is the largest consumer of its own input

Chemicals and Textile consume the least in the equal measure.

Given the matrix

0.4 0.1 0.1


 0.1 0.4 0.3
 
0.2 0.2 0.2 

We need to introduce the x = ( I - A ) this inverse is called the Leontief inverse


−1
CHAPTER 7 Matrices Algebra: Application of Matrices 131

1 0 0 
I = Is the identity matrix I = 0 1 0 
0 0 1 
0.4 0.1 0.1
A = Is the 3 x 3 matrix under consideration A =  0.1 0.4 0.3
0.2 0.2 0.2 

1 0 0  0.4 0.1 0.1


I − A = 0 1 0  −  0.1 0.4 0.3
0 0 1  0.2 0.2 0.2 

 0.6 −0.1 −0.1


I − A =  −0.1 0.6 −0.3
 −0.2 −0.2 0.8 

We need to get the inverse ( I − A ) by getting the cofactors as shown below


−1

 0.6 −0.1 −0.1


A =  −0.1 0.6 −0.3 =
 −0.2 −0.2 0.8 

0.6 −0.3
A11 =
+ 0.42
=
−0.2 0.8

−0.1 −0.3
A12 =
− 0.14
=
−0.2 0.8

−0.1 0.6
A13 =
+ 0.14
=
−0.2 −0.2

−0.1 −0.1
A 21 =
− 0.1
=
−0.2 0.8
CHAPTER 7 Matrices Algebra: Application of Matrices 132

0.6 −0.1
A 22 =
+ 0.46
=
−0.2 0.8

0.6 −0.1
A 23 =
− 0.14
=
−0.2 −0.2

−0.1 −0.1
A 31 =
+ 0.09
=
0.6 −0.3

0.6 −0.1
A 32 =
− 0.19
=
−0.1 −0.3

0.6 −0.1
A 33 =
+ 0.35
=
−0.1 0.6

Hence the cofactor matrix

0.42 0.14 0.14 


 0.1 0.46 0.14 
 
0.09 0.19 0.35

The determinant can be obtained through talking any row or column with its corresponding row
and column
Let take the first row

=A 0.6 ( 0.42 ) + ( −0.1) 0.14 + ( −0.1


= ) 0.14 0.224

Hence transpose and multiply with the determinant

0.42 0.1 0.09 


1 
 0.14 0.46 0.19 
0.224
0.14 0.14 0.35

The above matrix is called the Leontief inverse


We were given that
CHAPTER 7 Matrices Algebra: Application of Matrices 133

 672 
d =  224 
 448

Using the equation

x = (I - A) d
−1

To understand what the input-output model means


x = Total output
A= Matrix of technical coefficients
(I − A) =Leontief matrix
(I − A)
−1
Inverse of the Leontief matrix
=
D = Final demand or external demand

Then we solve for x

0.42 0.1 0.09   672 


1 
x= 0.14 0.46 0.19   224 
0.224 
0.14 0.14 0.35   448 

 ( 0.42 × 672 + 0.1× 224 + 0.09 × 448 ) 


1 
=x  ( 0.14 × 672 + 0.46 × 224 + 0.19 × 448)
0.224
( 0.14 × 672 + 0.14 × 224 + 0.35 × 448 ) 

344.96 
1 
x=  282.24 
0.224
 282.24 

344.96 0.224 
x =  282.24 0.224 
 282.24 0.224 

1540 
x = 1260  That is 1540 millions in Mining, 1260 in chemicals and 1260 in textiles.
1260 
CHAPTER 7 Matrices Algebra: Application of Matrices 134

However when the student is not asked for determinant there is another alternative method
which we are going evaluate
Alternative approach
We need still need to get the identity minus the matrix I − A

1 0 0  0.4 0.1 0.1


I − A = 0 1 0  −  0.1 0.4 0.3
0 0 1  0.2 0.2 0.2 

 0.6 −0.1 −0.1


I − A =  −0.1 0.6 −0.3
 −0.2 −0.2 0.8 

Given that

 672 
d =  224 
 448

Using the formula

(I − A) x =
d

We need to express the above in matrix notation

 0.6 −0.1 −0.1  x1   672 


     224 
 −0.1 0.6 −0.3  x2  =  
 −0.2 −0.2 0.8   x3   448

Solve for x1 , x2 and x3

0.6 x1 − 0.1x2 − 0.1x3 = 672


−0.1x1 + 0.6 x2 − 0.3 x3 = 224
−0.2 x1 − 0.2 x2 + 0.8 x3 =448

This will yield the same results and give the following answers

=x1 1540
= x2 1260
= x3 1260
CHAPTER 6 Matrices Algebra: Application of Matrices 135

The total internal consumption is expressed as AX

x−d =AX

1540   672   868 


1260  −  224  =  
    1036 
1260   448  812 

868 millions in mining


1036 millions in chemicals
812 millions in textile

Example 7.19
A manufacturing company is made of three industries namely steel rolling factory, iron melting
factory and Copper processing factory. The total output of steel, iron and copper is 500, 1000
and 1500 million shillings respectively. The final demand for steel, iron and copper is 150
million, 250 million and 400 million respectively. Out of what steel factory produces 200 million
is for its own use, 50 million is used by iron and 100 million is used by cooper processing factory.
Iron melting factory uses 200 million for its own production, 250 million by steel and 300 million
by copper. Copper processing factory uses 400 million for its own use, the rest 400 million and
300 million to steel and iron respectively. The final demand is expected to rise for steel and iron
by 50 million and for cooper to fall by 100 million, determine the new demand.
Determine the technical coefficient matrix
Given that the Leontief inverse matrix

0.24 0.57 0.45


 0.45 0.82 0.32 
 
 0.23 0.65 0.90 

a) Advice the manufacturing company what level each factory should produce to satisfy the
new demand.
b) Hence calculate the intermediate demand values
CHAPTER 7 Matrices Algebra: Application of Matrices 136

Solution

Let’s start by getting the technical coefficient matrix by first presenting the above data into a
table format.

Figure 1.3

Output
Steel Iron Cooper Final Total
demand output
Steel 200 50 100 150 500
Iron 200 250 300 250 1000
Input

Cooper 400 300 400 400 1500

The input of steel, iron, cooper and final demand is supposed to equal to the total output.

x1 + x2 + x3 + d1 =
x

That is 200 + 50 + 100 + 150 =


500

To obtain technical coefficient matrix we divided by the total output

 200 500 50 1000 100 1500 


 200 500 250 1000 250 1500 
 
 400 500 300 1000 400 1500 

0.40 0.05 0.67 


0.40 0.25 0.17 
 
0.80 0.30 0.27 

Given that

x = (I - A) d
−1

Then we need to get ( I - A )


CHAPTER 7 Matrices Algebra: Application of Matrices 137

We have been given the ( I - A )


−1
as

0.24 0.57 0.45


 
 0.45 0.82 0.32 
 0.23 0.65 0.90 

The final demand has been given as

There is expectation of the rise and fall of demand


So we get the final demand add or minus the change in demand where necessary.
The new demand is given by
x = d ± ∆d

 50 
∆d = 50 
 
100 

Hence

150  +  50   200 
New demand =  250  +  50  =  
    300 
 400  − 100  300 

Multiply new demand by the Leontief inverse

0.24 0.57 0.45


 0.45 0.82 0.32 
 
 0.23 0.65 0.90 

0.24 0.57 0.45  200  354 


=    432 
x  0.45 0.82 0.32  300   
 0.23 0.65 0.90  300   511

354 
x =  432 
 511

The intermediate demand at this level can be calculated as follows using the technical
CHAPTER 7 Matrices Algebra: Application of Matrices 138

coefficient matrix

0.40 0.05 0.67 


0.40 0.25 0.17 
 
0.80 0.30 0.27 

Multiply with the new demand of each level

354 
x =  432 
 511

0.40 0.05 0.67  354  505.57 


0.40 0.25 0.17   432  = 660.72 
    
0.80 0.30 0.27   511 853.47 

Practice Problem
1) An agricultural economy has three industries: Dairy, Beef and Crops.
To generate one shilling worth of Dairy requires KES 0.2 worth of dairy, KES 0.4 of beef and 0.3
of crops
To generated one shillings of Beef, we require KES 0.3 of dairy, KES 0.2 beef and 0.6 of crops.
To generate one shilling require KES 0.4 of dairy, KES 0.3 of beef and KES 0.1 of water.
Determine the internal consumption given that output is
 400 
X = 300 
 200 
First determine the technical coefficient matrix

2) ABC estate has three interrelated industries namely Coffee processing Factory, Tea
processing factory and Sisal bags weaving factory. The total output for coffee is 540 million
shillings, tea is 600 million shillings and Sisal is 900 million shillings. The final demand for coffee,
tea and sisal products is 75, million, 15 million and 130 million respectively. Out of what coffee
processing factory produces 90 million is for its own use, 225 million is used by the sisal bags
and tea processing uses 150 million. Tea processing uses 150 million for its own respectively.
Sisal bag weaving factory sells off 270 million to coffee processing factory and 200 million to tea
processing factory. Recently, the ministry of trade and Regional Cooperation project that coffee
processing tea processing factories, final demand will rise by 25 million and 5 million
respectively while sisal weaving factory will fall by 60 million in the next one year.
CHAPTER 7 Matrices Algebra: Application of Matrices 139

Determine the technological coefficients matrix for the economy


Given the Leontief matrix for the problem is

3.083 1.982 2.147 


2.642 3.413 2.697 
 
3.633 2.312 4.459 
Advise the ABC estate at what level each division should produce to satisfy the demand
What should be the intermediate demand values at these production levels?

3) In a relationship between steel and electricity 20% of the value of what steel produces is
consumed internally and 15% used to pay for electricity used in production. Also 35% of the
value of what the electricity industry produces is used to pay for steel consumed and 20% of it
pays for the electricity consumed internally.
Write the input- output matrix for steel and electricity
Given the production capacity of steel and electricity is shillings 90 billion and 75 billion
respectively, how much of each is consumed internally.

4) A miniature economy has three industries I1, I2 and I3, which are interdependent. Following
is s a transactions table for the industries. The figures show the value of inputs and output in sh.
Million

Inputs Final demand

I1 I2 I3

I1 119 225 300 56

Output I2 175 225 396 105

I3 350 297 396 157

a) Determine the primary inputs of each industry.


b) Determine the technical coefficients matrix.
c) Find the matrix (I – A)
d) Find the total output and the primary input required by each industry if the final demand
is 100, 150 and 200 for I1, I2, I3 respectively

5) A small economy has two strategic industries namely Agriculture and Manufacturing. To
produce KES 1.00 worth of agricultural products requires KES 0.30 of agricultural products and
CHAPTER 7 Matrices Algebra: Application of Matrices 140

KES 0.20 of manufactured products. To produce KES 1.00 worth of manufactured products
requires KEs 0.4 of agriculture produces and KES 0.10 of manufactured products.
If the gross output from the two industries is KES 800 and KES 700 million of agriculture and
manufacturing respectively what is the total internal consumption?
What is the total output required to meet a final consumer demand of KES 350 million and KES
250 million respectively?
CHAPTER 7 Matrices Algebra: Markov Process Model 140

Markov Process model

Markov process model is a stochastic model that is time based to make predictions of various
states in the future given the current states.
Markov process can be applied in accounts receivables, marketing in determining the brand
royalty and market share analysis and in insurance policies when calculating life insurance
payouts.
Markov process as stochastic process is used to analyze decision problems in which the
occurrence over specific events depends on the occurrence of the event immediately prior to
the current event. In general the Markov process helps to identify a specific state of the system
being studied and the state transition relationship. The occurrence of an event at a specified
point in time say period (n) and put and the system in a given state say (En).
If the passage of one period or unit of time events occurs during time period n + 1 the system
has improved to state to En + 1 in the next period.
Markov process model is useful in the studying the movement of certain system over repeated
trials, which are often successful time period where the system or outcome of the system in a
given time period cannot be given certainty.

Characteristics or Assumptions of Markov

Markov process has a finite number of state none of which is absorbing in nature (state is said
to be absorbing if a customer will never switch to another state)
The states are both connectively exhausted and mutually exclusive that is exhaust the market at
100%
The probability of moving from one state to another depends only on the immediately
preceding state.
Transition probabilities are stationary that is constant for duration of the analysis
The probability has a set of initial probability which may be given or determined.
The transition probability of moving two alternative states in the next period given a state in the
time period must sum to one unit that is 100%

Example 7.20
A Criso Ltd industry manufactures two products X and Y. The marketing manager noted that the
two brands have a very stiff completion in terms of advertising and promotion. The product
manager of product X complained to the marketing manager that brand Y is taking its market
CHAPTER 7 Matrices Algebra: Markov Process Model 142

share. He requested for a research to be done on the two products and the following
information was obtained. Of those loyal to brand X in a given week, 30% remained brand loyal
in the next week whereas the rest switched to brand Y. Of those loyal to brand Y, 40% switched
to brand X while the rest remained loyal. When the research report was submitted brand X and
Y were estimated to hold 65% and 35% 0f the market share respectively.
Determine the transition probabilities matrix for the above problem
Determine the market share in the first and second week?
Determine the long-run market share

Solution

We can present the following information into a tabular form so that we can be able to obtain a
transition probability matrix as shown below.

Figure 1.1
X Y
X 30% 70%
Y 40% 60%

Now we can obtain the transition probabilities matrix as

 0.3 0.7 
0.4 0.6 
 

During the first week the market share will change like shown below
We have to multiply the transition probability matrix with the vector matrix of the research
report

Vector matrix [ 0.65 0.35]

 0.3 0.7 
[0.65 0.35]   = [ 0.335 0.665]
0.4 0.6 

This means that 33.5% of the buyers of product X remained loyal whereas the 66.5% switched
to product Y in the first week.
To determine the next transition in the second week we need to use the vector matrix that we
have gotten that is [ 0.335 0.665]
CHAPTER 7 Matrices Algebra: Markov Process Model 143

 0.3 0.7 
[0.335 0.665]   = [ 0.3665 0.6335]
0.4 0.6 

This means that in the second week 36.65% of the buyers remained loyal to product X and
63.35% switched to product Y.

The long-run is also called the steady state, equilibrium state, it may be referred to us the
numerous run in the transition probability remain constant or unchanged. That is this is where
no further changes occur to the market share of each company in the analysis
In the long-run it is expected that the vector matrix is

 0.3 0.7 
[x 1 − x]  =[ x 1 − x]
0.4 0.6 

When we multiply we are supposed to get

0.3 x + 0.4(1 − x) =x
0.7 x + 0.6(1 − x) =1 − x

Using one of the equation find the value of x

0.3 x + 0.4 − 0.4 x =


x
−0.1x + 0.4 = x
1.1x = 0.4
x = 0.363

Then Y is given by 1 − x

Thus 1 − 0.363 =
0.637

Thus at long run the products are as follows

X = 0.363
Y = 0.637

When the transition matrix is 100% it is said to be exhausted for instance in the following
example
CHAPTER 7 Matrices Algebra: Markov Process Model 144

x1 x2 x3
A 40% 60% 45%
B 60% 40% 55%
100% 100% 100%

Example 7.21
Three companies A, B and C are in competition and are currently holding 30%, 30% and 40% of
the total market share respectively. Company A retains 60% of its customers and loses 20% to B
and 20% to C. Company B retains 70% of its customers and loses 10% to B and 20% C. Company
retains 60% while it loses 30% to B and 10%
a) Constitute the transition probability matrix
b) Express or interpolate in terms of gains in words
c) What will each company market share be in 1 year time and 2 year time
d) Determine the market share in the long-run

Solution
The transition probability matrix will be as follows
A B C
A 60% 20% 20% 
B 10% 70% 20% 
C 10% 30% 60% 
Thus

A B C
A 0.6 0.2 0.2 
B  0.1 0.7 0.2 
C  0.1 0.3 0.7 

This means that company A retains 60% of the customers and loses 20% to B and 20% to C.
Secondly Company B loses 10% of its customers to company B and 20% of its customers to C
and retains 70% to itself. Finally company C retains 60% of its customers and loses 30% to
company B and 10% to company C
CHAPTER 7 Matrices Algebra: Markov Process Model 145

A B C
A 0.6 0.2 0.2 
B  0.1 0.7 0.2 
C  0.1 0.3 0.6 

0.6 0.2 0.2 


[0.3 0.3 0.4]  0.1 0.7 0.2 = [0.25 0.39 0.36]
 0.1 0.3 0.6 

This means that Company retained 25% of its customers, company B retained 39% and company
C retained 36% in the first year.

In the second year the transition probability will change and we are going to multiply with the
new vector matrix we have gotten in above part as given below

0.6 0.2 0.2 


[0.25 0.39 0.36]  0.1 0.7 0.2 = [0.225 0.431 0.344]
 0.1 0.3 0.6 

In the second year the market share for company A is 22.5% and company B is 43.1% and
company C is 34.4%

The steady state or the long-run and in the long-run the matrix is expressed as given below
In general the steady state is given by

[x y z ][ P ] = [ x y z]

Thus in the given situation the steady state is expressed as follows considering the market share
of A, B and C is x, y and z

0.6 0.2 0.2 


[ x y z ]  0.1 0.7 0.2 = [ x y z]
 0.1 0.3 0.6 
We are supposed to multiply

0.6 x + 0.1 y + 0.1z =


x
0.2 x + 0.7 y + 0.3 z =
y
0.2 x + 0.2 y + 0.6 z =
z
CHAPTER 7 Matrices Algebra: Markov Process Model 146

Given the standard equation is


x+ y+z = 1

We can still correct the like terms together in the above equations.

−0.4 x + 0.1 y + 0.1z =0


0.2 x − 0.3 y + 0.3 z =
0
0.2 x + 0.2 y − 0.4 z =
0

Now we can use the following above two equations as below

−0.4 x + 0.1 y + 0.1z = 0.....................................(i )


0.2 x − 0.3 y + 0.3 z = 0........................................(ii )
x+ y+z = 1.......................................................(iii )

The value of x. y and z are


Take the first two equations that equation (i) and (ii) and eliminate x by multiplying by 2

−0.4 x + 0.1 y + 0.1z =0................................(i )


0.4 x − 0.6 y + 0.6 z =
0..................................(ii )

To obtain by adding both equations

−0.4 x + 0.1 y + 0.1z = 0


0.4 x − 0.6 y + 0.6 z =0
− 0.5 y + 0.7 z = 0............................(iv)

Take equation (ii) and (iii) and still eliminate x by multiplying by 0.2

0.2 x − 0.3 y + 0.3 z = 0........................................(ii )


x + y + z 1.......................................................(iii ) × 0.2

To obtain by subtracting both equation


0.2 x − 0.3 y + 0.3 z =
0
0.2 x + 0.2 y + 0.2 z =
0.2
−0.2.............................. ( v )
− 0.5 y + 0.1z =

Solve now equation (iv) and (v) subtracting both equations


CHAPTER 7 Matrices Algebra: Markov Process Model 147

−0.5 y + 0.7 z =
0
−0.5 y + 0.1z =−0.2
0.6 z = 0.2
z = 0.33

Using any equation between (iv) and (v) we can solve for y, for our case let use equation (v)

−0.5 y + 0.1( 0.33) =


−0.2
−0.5 y + 0.033 = −0.2
−0.5 y = −0.2 − 0.033
−0.5 y = −0.233
y = 0.466

From the standard equation we can obtain the value of x

x+ y+z = 1
=x ?= y 0.466 =z 0.33
x + 0.466 + 0.33 =
1
x + 0.796 = 1
x = 1 − 0.796
x = 0.204

x = 0.204
y = 0.466
z = 0.33

This means that at the long-run company A is expected to maintain 20.4%, while the rest move
to company B will maintain 46.6% and company C maintain 33%

Example 7.23
The market share of two companies A and B is 30% and 70% in the current time period. The
information obtained in terms of the customer lose and retention is given by the matrix D

 0.3 0.7 
D= 
0.2 0.8 

Determine the transition probability matrix in the 1st and 2nd month
What is the steady state of the two companies
CHAPTER 7 Matrices Algebra: Markov Process Model 148

Solution

In the first month


 0.3 0.7 
[0.3 0.7]   = [ 0.23 0.77 ]
0.2 0.8 

Second month
 0.3 0.7 
[0.23 0.77]   = [ 0.223 0.777 ]
0.2 0.8 
Steady state
 0.3 0.7 
[ x 1 − x]  = [ x 1 − x]
0.2 0.8 
0.3 x + 1 − x ( 0.2 ) =x
0.7 x + 1 − x ( 0.8 ) = 1 − x

0.3 x + 0.2 − 0.2 x =


x
0.3 x − 0.2 x − x =−0.2
−0.9 x = −0.2
x = 0.22
y = 0.88

Example 7.24
Three companies X, Y, and Z are in stiff competition and the relevant managers have asked for a
market research to determine the market share. The research report indicated that company X
had retained 55% of its customers and lost 25% to company Y and 20% to company Z. Company
Y retained 60% of its customers and 30% switched to X and 10% switched to Z. Company Z
retained 90% and 5% switched to company X and rest to company Y. The companies’ current
states are 40%, 30% and 30% respectively

a) Determine the transitional probability matrix


b) What will be the market share in the third year
c) Determine the market share in the steady state
CHAPTER 7 Matrices Algebra: Markov Process Model 149

Solution

X Y Z
X 55% 25% 20% 
Y 30% 60% 10% 
Z  5% 5% 90% 
Transition probability matrix
X Y Z
X  0.55 0.25 0.20 
Y 0.30 0.60 0.10 
Z  0.05 0.05 0.90 
Market share in third year
First calculate the market share in the first year

 0.55 0.25 0.20 


[0.4 0.3 0.3] 0.30 0.60 0.10 = [0.325 0.295 0.38]
 0.05 0.05 0.90 
Market share in the second year

 0.55 0.25 0.20 


[0.325 0.295 0.38] 0.30 0.60 0.10 = [0.286 0.277 0.437]
 0.05 0.05 0.90 

The market share in the third year is

 0.55 0.25 0.20 


[0.286 0.277 0.437] 0.30 0.60 0.10 = [0.262 0.260 0.478]
 0.05 0.05 0.90 

Steady state

In general the [ x y z ][ P ] = [ x y z]
CHAPTER 7 Matrices Algebra: Markov Process Model 150

 0.55 0.25 0.20 


[x y z ] 0.30 0.60 0.10  = [ x y z]
 0.05 0.05 0.90 

0.55 x + 0.3 y + 0.05 z =


x
0.25 x + 0.6 y + 0.05 z =
y
0.2 x + 0.1 y + 0.9 z =
z

Using the standard equation

1
x+ y+z =

We can collect the like terms together

−0.45 x + 0.3 y + 0.05 z = 0


0.25 x − 0.4 y + 0.05 z =0
0.2 x + 0.1 y − 0.1z = 0

Now we can use the standard

−0.45 x + 0.3 y + 0.05 z =0......................(i )


0.25 x − 0.4 y + 0.05 z =0.........................(ii )
1..........................................(iii )
x+ y+z =

Take the first two equations that equation (i) and (ii) and eliminate z by subtracting both
equations

−0.45 x + 0.3 y + 0.05 z =0................................(i )


0.25 x − 0.4 y + 0.05 z =0..................................(ii )

To obtain by adding both equations

−0.45 x + 0.3 y + 0.05 z = 0


0.25 x − 0.4 y + 0.05 z = 0
− 0.7 x + 0.7 y = 0..................................(iv)

Take equation (ii) and (iii) and still eliminate z by multiplying by 0.05
CHAPTER 7 Matrices Algebra: Markov Process Model 151

0.25 x − 0.4 y + 0.05 z = 0.........................(ii )


= x + y + z 1..........................................(iii ) × 0.05

To obtain by subtracting both equation

0.25 x − 0.4 y + 0.05 z =


0.........................(ii )
0.05 x + 0.05 y + 0.05 z 0.05..........................................(iii ) × 0.05
=
0.2 x − 0.45 y =−0.05.........................................(v)

Solve now equation (iv) and (v) by multiplying by 0.2 and 0.7 respectively

−0.7
= x + 0.7 y 0....................(iv) × 0.2
0.2 x − 0.45 y =−0.05.............(v) × 0.7
Eliminate x

−0.14 x + 0.14 yz =0
0.14 x − 0.315 y =−0.035
−0.175 y = −0.035
y = 0.2

Using any equation between (iv) and (v) we can solve for y, for our case let use equation (v)

0.2 ( 0.2 ) − 0.45 y =


−0.05
0.04 − 0.45 y =
−0.05
−0.45 y = −0.05 − 0.04
−0.45 y = −0.09
y = 0.2

From the standard equation we can obtain the value of z

x+ y+z = 1
=x 0.2= y 0.2
= z ?
0.2 + 0.2 + z =
1
z + 0.2 = 1
z = 1 − 0.4
z = 0.6
CHAPTER 7 Matrices Algebra: Markov Process Model 152

x = 0.2
y = 0.2
z = 0.6

=X 20%
= Y 20% =Z 60%

Practice Problem
1. A,B and C mobile companies are in competition and currently hold 50%, 30% and 20%
respectively of the total market share. A retains 70% of its customers, losses 20% to B
and 10% to C. B retains 80% of its customer; losses 10% to A and 10% to C. C retains 60%
of its customers; losses 105 to A and 30% to B

a) Develop the transition probability matrix for this system


b) What will each channel market be in two years time?
c) Calculate each channels market share equilibrium

2. State and briefly describe areas Markov analysis is used in modern business world.

3. Small town has only three banks B1, B2, B3. The three banks have a combined client
base of 21,000 customers. Each customer has only one savings account in any one of the
banks. However customers keep shifting their accounts from one bank to another. The
table below shows the movement of customers account in the 1st month.

To

B1 B2 B3

B1 4800 600 600

From B2 1200 8400 1800

B3 300 900 2400


CHAPTER 7 Matrices Algebra: Markov Process Model 153

At the start of the first month B1 had 6000 accounts, B2 had 12000 accounts and B3 had
3000 accounts.

a) Develop the probability transition matrix.


b) Determine the number of savings accounts in each bank at the beginning of the
2nd month.
c) Determine the distribution of the accounts in the long run.

4. There are three firms ABC, PQR and XYZ sharing a market as 40%, 40% and 20%
respectively on January 1, 2007. Over a year, the following developments took place.
ABC retains 80% of its customers, loses 16% to PQR and 4% to XYZ
PQR retains 84% of its customers, loses 12% to ABC and 4% to XYZ
XYZ retains 76% of its customers, loses 18% to ABC and 6% to PQR
Assuming that the total market size does not change,
a) What share of the market did each firm hold on January 1, 2012?
b) What would be the long-run shares of the firms at equilibrium if the buying
habits do not change?

5. The purchase patterns of two brands of toothpaste can be expressed as a Markov


process with the following transition probabilities:

Formula X Formula Z
Formula X 0.90 0.10
Formula Y 0.05 0.95

a) Which brand appears to have most loyal customers? Explain.


b) What are the projected market shares of the two brands?

6. Suppose that in the above problem (5) a new toothpaste brand enters the market such
that the following transition probabilities exist:

Formula X Formula Y Formula Z


Formula X 0.80 0.10 0.10
Formula Y 0.05 0.75 0.20
Formula Z 0.40 0.30 0.30

a) What are the new long-run market shares?


b) Which brand will suffer most from the introduction of the new brand of
toothpaste?
c) Explain the term ‘Steady State’ (Equilibrium) in Markov process.

7. Explain characteristics of a Markov process


CHAPTER 7 Matrices Algebra: Markov Process Model 154

8. A market survey conducted in January on behalf Clean Soap Manufacturers to


determining the selling pattern of their three soap brands, Simba, Chui and Twiga, gave
the following information: Out of a sample 2400 buyers interviewed, 750 bought Simba
and 900 bought Twiga, the rest bought Chui. Of those who bought Simba 90 will switch
to Chui next month and 120 will switch to Twiga. Of those who bought Chui, 60 will
switch to Simba and 90 will switch to Twiga. 90 of those who bought Twiga will switch to
Simba and 45 will switch to Chui.

a) The market share of each brand in January expresses as a percentage.


b) The market share of each brand in March.
c) The market shares of the brands in the long run
DECISION
CHAPTER 8 ANALYSIS
Learning Objectives
At the end of this section the students should be able to:

 Understand the basic of decision analysis


 Determine the Maxmin Minimax Approach
 Determine the Minimax Regret Approach
 Determine the Decision making with probabilities

A decision in general may be defined as the selection by the decision maker of an act considered
to be the best according to some pre-designated standard from the several available options.
Decision analysis can be determining optimal strategies when a decision maker is faced with
several alternatives and uncertainty or risks feared pattern of future. Irrespective of the type of
decision making problem they are certain elements which are common to all such problems.

Essential elements for decision

Cause of action- a decision made from among a set of defined alternative causes of action
State of nature – These are consequences or events of any causes of action are dependent upon
certain factors beyond the control of the decision maker. These factors are called status of
nature.
Uncertainty – there is uncertainty regarding which event or outcome will occur. This
uncertainty is indicated in terms of probabilities assigned to events.
Pay off – each combination of course of action and events or outcome is associated with a pay-
off which measures the net benefit to the decision maker that accrues from a given combination
of decision alternatives and events. They also known as conditional profits values or conditional
profit values or conditional economic consequences.
CHAPTER 8 Decision Analysis 156

Example 8.1
A phone manufacture is in the final stages of selecting a computerized assembly plant, for its
company untested model. The company needs to determine the size of the plant that will be
economical to build and maintain. There three sizes large, medium and small. The management
believes that the model overall level of acceptance in the market will be one or two possibilities.
High acceptance ( high sales)
Low acceptance (low sales)

To make decision these are the steps to follow


Step I
Determine the various alternatives course of action from which the final decision is to be made
that is
d1.............building large plants
d 2 .............building medium plants Course of action
d 3 .............building small plant

We should note that the selection of the best decision alternative will depend on what
management fore seen to be the best possible management acceptance of the model and
consequently the demand and resources.

Step II
Identify the possible outcomes and the state and the state of nature of events for the decision
problem. The events are beyond the control of the decision makers

S1...............high acceptance
S2 ...............Low acceptance

Step III
Determine the payoffs function which describes the consequences resulting from the
difference. Combination of the acts and events. Suppose the management has estimated the
profits from the sales of the model to be as follows

High Low
acceptance acceptance
S1 S2
Decision
alternative
d1 Large 250 -30

d 2 Medium 200 30
CHAPTER 8 Decision Analysis 157

d 3 Small 150 70

When we consider the payoff to be V then to calculate payoff we multiply the decision
alternative by the acceptance

S1 high
d1 Large
S2 Low

S1 High
d2 Medium
S2 Low
Decision node is
always a S1 High
d3 Small
square/rectangle
S2 Low

State on nature
node

Solution

High Acceptance Low Acceptance


Decision Alternative S1 S2
D1 Large 200 -20
D2 Medium 150 20
D3 Small 100 60

We can illustrate the above in a decision tree below


CHAPTER 8 Decision Analysis 158

S1 High 200

D1 Large
S2 Low -20

S1 High 150
D2 Medium
S2 Low 20
Decision node S1 High 100
(always represented
by a square or D3
rectangle) S2 Low 60
State of nature node
shown by a circle

Maxmin Minimax Approach

This method is also called conservative it evaluates each decision in terms of, first determine the
minimum pay offs for each alternative and secondly the alternative which correspond to the
maximum of the minimum pa offs

S1 S2 Min
D1 200 −20 −20
D2 150 20 20
Maximizes the
D3 100 60 60 minimum

Laplace – some probability for all states of


Optimizer – Maximize the maximum (if profit) or min (if costs)

Minimax Regret Approach

The minimax approach is based on the concept of regret opportunity cost or opportunity cost
and calls for selecting the cost of action that minimizes the pay offs

Step 1
Determine the amount of regret associated with each decision alternative
CHAPTER 8 Decision Analysis 159

S1 S2 Min Maximizes the


D1 200 −20 −20 minimum
D2 150 20 20
D3 100 60 60

Step 2
Construct a regret table as below

S1 S2
D1 200 − 200 60 − −20 Subtract the highest value in
D2 200 − 150 60 − 20 each column in our case it is
D3 200 − 100 60 − 60 200 in first column and 60 in
the second column

S1 S2
D1 0 80
D2 150 40
D3 100 0

Step 3
Extract the max regret associated with each decision alternative

S1 S2 Max regret
D1 0 80 80
D2 150 40 50
D3 100 0 100

Step 4
Pick a decision alternative that minimizes the max regret

S1 S2 Max regret
D2 to minimize
D1 0 80 80 the maximum
D2 150 40 50
D3 100 0 100
CHAPTER 8 Decision Analysis 160

Decision making with probabilities

When probability estimates for each forms of nature they are referred as expected value
approach and can be used to identify the best decision alternative.
Let n be the number of state of nature
P ( S j ) the probability of state of nature S j
Since only one of the n states of nature occurs the associated probabilities must satisfy the
following two actions;
P ( S j ) ≥ 0 for all states of nature
If you sum up the probabilities

∑ P ( S ) = P ( S ) + P ( S ) + P ( S ) + ... + P ( S ) = 1
j =1
j 1 2 3 n

The expected value (EV) of decision alternative di is defined as follows

n
EV ( di ) = ∑ P ( S j )V ( di S j )
j =1

Example 8.2

Given the following

S1 S2
D1 200 −20
D2 150 20
D3 100 60

Let

P ( S1 ) 0.3
= P ( S2 ) 0.7

To find the expected value


CHAPTER 8 Decision Analysis 161

Solution

EV ( d=
1) ( 0.3 × 200 ) + ( 0.7 × −20=) 46
EV ( d2 ) = ( 0.3 × 150 ) + ( 0.7 × 20 ) = 59
EV ( d3 ) = ( 0.3 × 100 ) + ( 0.7 × 60 ) = 72

D3 is the best alternative as it has the highest expected value.

Example 8.3

Consider the payoff table below (The payoff in millions of shillings and P ( S j ) the probability of
state of nature S j occurring)

Action S1 S2 S3
D1 20 25 -10
D2 30 15 30
D3 40 0 50
P(Sj ) 0.3 0.4 0.3
Recommend the best decision alternative using
a) Optimistic approach
b) Pessimistic approach
c) Bayes (Expected Value ) approach
d) Minimax Regret approach

Solution

Optimistic approach is based on extreme optimistic this extracts the maximum payoffs for each
decision alternative and selects the maximum
CHAPTER 8 Decision Analysis 162

Action S1 S2 S3 Best payoff ( Max )


D1 20 25 -10 25 Pick D3 to
maximize max
D2 30 15 30 30
D3 40 0 50 50

Pessimistic approach extracts the minimum (worst) payoff and maximize.

Action S1 S2 S3 Worst payoff ( Min )


D1 20 25 -10 −10 Pick D2 to
maximize
D2 30 15 30 15
minimum
D3 40 0 50 0

Bayes (Expected value) approach that is

n
EV ( di ) = ∑ P ( S j )V ( di S j )
j =1

EV ( d=
1) ( 0.3 × 20 ) + ( 0.4 × 25) + ( 0.3 × −10=) 13
EV ( d2 ) = ( 0.3 × 30 ) + ( 0.4 × 15 ) + ( 0.3 × 30 ) = 24
EV ( d3 )= ( 0.3 × 40 ) + ( 0.4 × 0 ) + ( 0.3 × 50 )= 27

Minimax Regret Approach

Action S1 S2 S3
D1 20 25 -10
D2 30 15 30
D3 40 0 50

Make the regret max table


Pick D2 to
Action S1 S2 S3 Regret minimize max
profit
D1 20 25 -10 60
D2 30 15 30 20
D3 40 0 50 25
CHAPTER 8 Decision Analysis 163

Example 8.4

A company has developed the following profit payoff table:

S1 S2
D1 200 450
D2 400 550
D3 650 350
The prior probabilities are given as P(S1) = 0.6 P(S2) = 0.4

A market research report has been offered to the company by a new research firm for which the
following probability information (conditional probabilities) is available.

P(I1/S1) = 0.9 P(I2/S1) = 0.2


P(I1/S2) = 0.1 P(I2/S2) = 0.8

What are the posterior probabilities?

What is the company’s optimal strategy?

Solution

a) Posterior probabilities
Sj P (S j ) P ( I1 /S1 ) P ( I1n S1 ) P ( S1 /J1 )
S1 0.6 × 0.9 = 0.54 0.54 = 0.93
0.58
S2 0.4 × 0.1 = 0.04 0.04 = 0.07
0.58
P ( I1 ) 0.58

Sj P (S j ) P ( I 2 /S1 ) P ( I 2 n S1 ) P ( S1 /J 2 )
S1 0.6 × 0.2 = 0.12 0.12 = 0.27
0.44
S2 0.4 × 0.8 = 0.32 0.32 = 0.73
0.44
P ( I 2 ) =0.44
CHAPTER 8 Decision Analysis 164

b).Company’s optimal strategy

probabilities P ( S1 /J1 ) 0.93


We use the posterior= = P ( S2 /J1 ) 0.07

+ ( 450 × 0.07 ) =
D1        ( 200 × 0.93)        217.5 
     
D 2        400 + ( 550 × 0.07 )   
( × 0.93)     = 410.5
        
+ ( 350 × 0.07 ) = 629
D3        ( 650 × 0.93)      

D3 is the best optimal strategy because it has the highest value of 629

probabilities P ( S1 /J 2 ) 0.27
Again when using the posterior= = P ( S2 /J 2 ) 0.73

D1        ( 200 × 0.27 )      


+ ( 450 × 0.73) =
 382.5
     
( × 0.27 )    
D 2        400 + ( 550 × 0.73)   
= 509.5
        
D3        ( 650 × 0.27 )      
+ ( 350 × 0.73) = 431

D2 is the best optimal strategy because it has the highest value of 509.5
CHAPTER 8 Decision Analysis 165

Practice Problem
1. Suppose that a decision maker faced with three decision alternatives and two states of
nature develops the following profit payoff table

S1 S2
D1 1300 1500
D2 1500 1150
D3 1400 1250

Draw a decision tree represent the above diagram scenario

2. John bakes and sells queen cakes. The demand per week ranges between 1000 and 1500
cakes. The cakes are in packets of 100. The table below shows the probability
distribution of the cakes.

Demand per week (cakes) Probability


1000 0.10
1100 0.30
1200 0.20
1300 0.20
1400 0.15

3. Suppose that a decision maker faced with four decision alternatives and four states of
nature develops the following profit payoff table:

States of Nature
S1 S2 S3 S4
Decision Alternatives D1 14 9 10 5
D2 11 10 8 7
D3 9 10 10 11
D4 8 10 11 13

If the decision maker knows nothing about the probabilities of the four states of nature,
what is the recommended decision using the approaches below
a) Laplace
b) Optimistic
c) Conservative
d) Minimax regret
CHAPTER 8 Decision Analysis 166

4. Suppose the decision maker in (I) above obtains information that enables the following
probability estimates to be made:

P(S1) = 0.50, P(S2) = 0.20, P(S3) = 0.20, P(S4) = 0.10

a) Use the expected value approach to determine the optimal decision


b) Now assume that the entries in the payoff table are costs; use the expected value
approach to determine the optimal decision
c) What is the value of perfect information (EVPI)?

5. A company has developed the following profit payoff table:

S1 S2
D1 100 350
D2 300 250
D3 450 450
The prior probabilities are given as P(S1) = 0.35 P(S2) = 0.65

A market research report has been offered to the company by a new research firm for
which the following probability information (conditional probabilities) is available.

P(I1/S1) = 0.95 P(I2/S1) = 0.05


P(I1/S2) = 0.15 P(I2/S2) = 0.85

a) What is the company’s optimal strategy?

b) What is the Expected Value of Perfect Information?

6. John can bake 1000 to 1500 cakes depending on the demand. However, since the
uncertain she has to decide in advance how many cakes to bake for the coming week.
Each cake costs sh. 10 and is sold at KES 15.
a) Construct a payoff table for John.
b) Use the expected monetary value criterion to determine the optimal number of
cakes she should bake for next week.
c) Repeat part (b) above using
i. Maximax criterion
ii. Maximin criterion
iii. Minimax regret criterion
CALCULUS I:
CHAPTER 9 DIFFERENTIATION

Learning Objectives
At the end of this section the students should be able to:
dy
• Recognize the notation f'(x) and for the derivative of an equation
dx
• Find the gradient of curve using graphs and symbolically differentiate.
• Identify a tangent, secant and chord of curve.
• Getting the derivative of a function
• Differentiate power fuctions

Given the following function y = 3 x 2 + 4 x + 10 we say that y is a function of x


The gradient of a curve at any point through a tangent at that point is called differentiation. The
gradient of a curve at any point is the tangent to the curve at that point.
Consider the following the following figure

Figure 9.0

Constant gradient

Change in y (∆y )

Change in x (∆x)

Linear equation
CHAPTER 9 Calculus I: Differentiation 168

The gradient which is also called the slope is constant through out the line whether positive
gradient like the above or a negative gradient. Taking any two points it will always give us the
same gradient. The slope is given by;
Change in y ∆y
=
Change in x ∆x
The above case is only true when dealing with the linear functions

Taking the figure 5.1 line A shows the tangent of the curve y = f ( x) .A tangent is a line the
touches a curve at a point.

Figure 9.1

y = f ( x)

Tangent A

Tangent to curve y = f ( x)

Figure 5.2 Line A is the tangent of the curve y = f ( x) . Line B and C are secant is an
approximation of a tangent and can be use to calculate the slope of a curve.
Line D and E is called a chord which is a line that join two points on a curve.
.
CHAPTER 9 Calculus I: Differentiation 169

Figure9.2

Chord

y = f ( x)

Secant C

C
Tangent A
B Secant B
A

Secant B and C approximating tangent A

Taking the second scenario of the curve below the gradient varies everywhere along the curve

Figure 9.3

C
Gradient variable
Tangent C
B

Tangent B
Tangent A

Non-linear equation
CHAPTER 9 Calculus I: Differentiation 170

A tangent at a particular point will be used to find the slope at that point the slope of a curve is
variable everywhere there is a tangent we have a different gradient.
Consider the curve ABC in the figure 5.3 let now consider a car moving from point A to C via B.
between A and C the slope is steadily increasing, when the car reaches point B the slope stops
increasing and remains constant.
The derived function is called the derivative of f and is denoted by the symbol f ' which is
read f prime.
The gradient function is called derivative function and denoted by the following notation
dy
read as dee y over dee x.
dx
We should note that dy / ∂x doesn’t mean dy divided by ∂x but it is a single symbol
representing derivative of y in respect to x

The gradient of a curve


Example 9.1

Consider the following function y = x 2

Solution

dy
In this we can use the or use f ( x) = x 2
dx
The graph is draw by taking the range of to be −2 ≤ x ≤ 2

Table 9.4

x -2.0 -1.5 -1 -0.5 0 0.5 1 1.5 2.0


x2 4 2.25 1 0.25 0 0.25 1 2.25 4

y = x2 4 2.25 1 0.25 0 0.25 1 2.25 4


CHAPTER 9 Calculus I: Differentiation 171

Figure 9.5

4
2
y=x

3
∆y =1.5
∆y =−1.5

2
∆x =−0.5 ∆x =0.5

Tangent
1
∆y =−0.5 ∆y =0.5

-1 ∆x =−0.5
∆x =0.5
-2 1 2

When the graph is plotted it appears like the above Figure 5.4 . Using the slope of the tangent
we find the derivative as follows.

−1.5
f '(−1.5) = = −3
0.5
−1.5
f '(−0.5) = = −1
0.5

f '(0) = 0
0.5
f '(0.5)
= = 1
0.5
1.5
f '(1.5)
= = 3
0.5
CHAPTER 9 Calculus I: Differentiation 172

Example 9.2

y x 2 − 1 using the range of −2 ≤ x ≤ 2 we can first obtain the


Consider the following function =
value of y by

Solution

Table 9.6

x -2.0 -1.5 -1 -0.5 0 0.5 1 1.5 2.0


x2 4 2.25 1 0.25 0 0.25 1 2.25 4
-1 -1 -1 -1 -1 -1 -1 - -1 -1
1
y x2 −1 3
= 1.25 0 -0.75 -1 -0.75 0 1.25 3

When we plot the graph it will appear as follows


CHAPTER 9 Calculus I: Differentiation 173

Figure 9.7

3
y x2 −1
=

Tangent
Tangent at x = 1
at x = 1

-2 ∆y =1 -1 1 ∆y =1 2

∆x =1 ∆x =1
Tangent at x = 0 -1

From the Figure 5.5 the gradient at the tangent x is -1, 0, and 0.5 is given as follows. We obtain
the gradient manually.
−2
When x = −1 we obtain = −2 , similarly when x = 0 the gradient is zero at the tangent
1

1 1
x= the slope is = 1
2 1

Now when using the graph method it is very involving and thus we can use differentiation
which is the process of finding the slope of a curve at part6icular point through a tangent at the
that point.
CHAPTER 9 Calculus I: Differentiation 174

Consider the function of the form f ( x) = x n when we get the derivative of the equation we get
f '( x) = nx n −1

dy
In terms of the y = x n we introduce to obtain a derivative of the same nature with the
dx
dy
above which is as follows = nx n −1
dx
When differentiating x n we multiply the coefficient of x with the power n and simultaneously
subtract 1 from the power n to get n − 1

Figure 9.8

Coefficient of x
Power

y = 1x n

Now let take our previous function y = x 2


To get the derivative of the equation we multiply the coefficient of;

dy
= 2 × x 2−1
dx
dy
= 2= x1 2 x
dx

We can verify the result we had gotten when using the graph through this symbolic method as
follows
Taking the values of x to -1.5, -1, -0.5, 0, 0.5, 1, 1.5 then with the derivative of 2x

dy
= 2 x = (2 × −1.5) = −3 Given that x = −1.5
dx

dy
= 2 x = (2 × −1) = −2 Given that x = −1
dx

dy
= 2 x = (2 × −0.5) = −1 Given that x = −0.5
dx
CHAPTER 9 Calculus I: Differentiation 175

dy
= 2 x = (2 × 0) = 0 Given that x = 0
dx

This will apply to the positive values of x a given below

dy
=2 x = (2 × 0.5) = 1
dx
x = 0.5
dy
= 2 x = (2 × 1) = 2
dx
x =1
dy
=2 x =(2 × 1.5) =3
dx
x = 1.5

The above results are similar to the result we got when using the graphical method.

Example 9.3

Given the following differentiate


1
(i) y = x (ii) y = 3 (iii) y = x5 (iv) y=4 (v) y = x
x

Solution

When differentiating y = x we write the function in the format

1
y = x = y = x2

1
dy 1 −1
= × x2
dx 2
CHAPTER 9 Calculus I: Differentiation 176
1
1 −
= x 2
2
1
= 1
2x 2
1
=
2 x

1
Differentiate y = 3
we can express this function in this as y = x −3
x

dy
= −3 x −3−1
dx
= −3 x −4
3
= − 4
x

dy
When differentiating y = x5 we need to in form of y = x n thus = nx n −1
dx

dy
= 5 x5−1 = 5x 4
dx

To differentiate y = 4 we must remember that x 0 is raised to power zero


And any number raised to power zero is equal to 1 thus

y = 4 x0
dy
= 0 × 4 x 0−1
dx
0 × 4 x −1 =
0

We use the general formula of y = x n we can differentiate

dy
x1−1 1x 0 Since any number raised to power zero is 1
= 1=
dx

Thus x 0 = 1

dy
=1
dx
CHAPTER 9 Calculus I: Differentiation 177

Practice Problem

(i) y = 5 x

1
(ii) y =
x

(iii) y = 1

(iv) y = x10

(v) y = 3 x

1
(vi) y = 3
x
RULES OF DIFFERENTIATION

Learning Objectives
At the end of this section the students should be able to:
• Use constant rule to differentiate a function.
• Use sum rule to differentiate a function.
• Use the difference rule to differentiate a function.
• Use a combination of sum and difference rule to differentiate a
function.
• Use product or multiplication rule to differentiate a function.
• Use quotient or division rule to differentiate a function.
• Use chain rule to differentiate a function.

1 Constant Rule
Given that y = cx n then we differentiate x n first and then multiply with c. The general formula
dy
is = nx n −1
dx
The constant rule states that we differentiate the function and multiply by the constant

Example 9.4

Differentiate

(a) y = 3x3

(b) y = 6x

(c) y = 4 8 x
CHAPTER 9 Calculus I: Differentiation, Rules of differentiation 179

Solution
(a) y = 3 x3 Using the constant rule we are supposed to differentiate x 3 then multiply with 3

dy
= 3(3
= x3−1 ) 9 x 2
dx

(b) y = 6 x First differentiate x and multiply with the constant of 6

dy
= 6( x1=
−1
x0 6
) 6=
dx

(c) y = 4 8 x We first differentiate 8


x and then multiply by 4

dy  18   1 18 −1  1 −7 8
= 4=
 x  4  x=  x
dx    8  2

1
= 7
2x 8

Practice Problem

(a) y = 6 x 7

(b) y = 15 x

(c) y = 6 x 2

(d) y = 5 6 2x
CHAPTER 9 Calculus I: Differentiation, Rules of differentiation 180

2 The Sum Rule

In sum rule we differentiate each function separately and add thus given that
y = ax n + bx n + cx n

dy
Thus the in general = nax n −1 + nbx n −1 + ncx n −1
dx

Example 9.5

Differentiate
i) y = 3 x 2 + 2 x + 2

y 4 x3 + 6
ii) =

iii) y = 5 x3 + x 6 + 4 x

Solution

To differentiate y = 3 x 2 + 2 x + 2 we need to differentiate 3x 2 , 2x and 2

3x 2 Differentiate to 6x

2x Differentiate to 2

2 Differentiate to 0

Then we add them together

dy
= 6x + 2
dx

ii) y 4 x3 + 6 we differentiate 4x3 and 6


To differentiate =

4x3 Differentiate to 12x 2


CHAPTER 9 Calculus I: Differentiation, Rules of differentiation 181

6 Differentiate to 0

dy
= 12 x 2
dx

To differentiate y = 5 x3 + x 6 + 4 x we take 5x3 , x 6 and 4x differentiate

Separately

5x3 Differentiate to 15x 2

x 6 Differentiate to 6x 5

4x Differentiate to 4

dy
= 15 x 2 + 6 x5 + 4
dx

Practice Problem

i) y = 4 x5 + x8 + 6

a 7b 2 + b
ii)=

iii) m = 6n 4 + 3n 2 + n 6 + 7

3. The Difference rule

This rule is that we differentiate separately and the subtract in general given
y = ax n − bx n − cx n we can differentiate to:

dy
= nax n −1 − nbx n −1 − ncx n −1
dx
CHAPTER 9 Calculus I: Differentiation, Rules of differentiation 182

Example 9.6

y 2x2 − x
a) =

= n6 − n2
b) m

p = 4q 2 − 3 p − 9

Solution

y 2 x 2 − x we need to differentiate 2x 2 and x then subtract


To differentiate=

2x 2 Differentiate to 4x

x Differentiate to 1

dy
= 4x − 1
dx

= n 6 − n 2 we similarly differentiate n 6 and n 2 , then subtract


We differentiate m

n 6 Differentiate to 6n5

n 2 Differentiate to 2n

dy
= 6n 5 − 2n
dx

We differentiate p = 4q 2 − 3 p − 9 we similarly differentiate 4q 2 and 3 p , then subtract

4q 2 Differentiate to 8q

3 p Differentiate to 3
CHAPTER 9 Calculus I: Differentiation, Rules of differentiation 183

Practice Problem

Differentiate

a) y = 5 x5 − 2 x 2 + 4 x

b) s= t 5 − t 2

m 4n 6 − n
c) =

1 5
d) =y 2
−3
2x 3x

4. The Combination Sum rule and Difference


rule

A combination of the sum rule and the difference rule that we have computed in the previous
examples

Example 9.7
Differentiate

i) y = 7 x 4 + x3 − 3 x 2 + 4

ii) s =t 7 − 4t 4 + 2t

iii) y = x10 + x8 − x5 − 1

Solution

In this case we differentiate separately and add, subtract where there is positive or negative
sign respectively.
CHAPTER 9 Calculus I: Differentiation, Rules of differentiation 184

i) y = 7 x 4 + x3 − 3 x 2 + 4

dy
= 4 × 7 x 4−1 + 3 x3−1 − 2 × 3 x 2−1 + 0 × 4 x1−0
dx

= 28 x3 + 3 x 2 − 6 x + 0

= 28 x3 + 3 x 2 − 6 x

To differentiate s =t 7 − 4t 4 + 2t we must differentiate s in respect to t

dy
= 7t 7 −1 − 4 × 4t 4−1 + 2t1−1
dx

=7t 6 − 16t 3 + 2

iii) y = x10 + x8 − x5

dy
= 10 x10−1 6 x8−1 5 x5−1
dx

= 10 x 9 + 6 x 7 − 3 x 4

Practice Problem
Differentiate

1. f '( x) = 4 x 6 + 2 x 4 − x 2 + x

2 1
2.. y = 4
2x − 2
+
x x

3. k = l 5 − 2l 3 + l 2 − 4l
CHAPTER 9 Calculus I: Differentiation, Rules of differentiation 185

5. The Product rule or Multiplication rule

When differentiate in this rule we are dealing with the product of two functions. We
Can also call this multiplication rule
The concept behind this rule is multiplying each function by the by the derivative of
the other function.

Example 9.8
Differentiate the following functions.

1). y = (3 x + 1)(8 x + 4)

2) y
= x (4 x + 3)

3) y =( x + 4)(2 x + 2)

Solution

1) y = (3 x + 1)(8 x + 4)

Let the fist function be u and the second function be v

u 3 x + 1 and=
That is= v 8x + 4

dy dv du
We can now = u + v
dx dx dx

Consider= u 3 x + 1 as a function and thus we must differentiate the function


du
Separately so as to obtain= u 3 x + 1 then =3
dx
dv
Again consider the function = v 8 x + 4 and differentiate to =8
dx
dv du
Then we multiply u we have (3 x + 1)8 = 24 x + 8 and the second function is given by v we
dx dx
get (8 x + 4)3 = 24 x + 12 then we must add
CHAPTER 9 Calculus I: Differentiation, Rules of differentiation 186

dy dv du
= u +v
dx dx dx

24 x + 8 + 24 x + 12

24 x + 24 x + 8 + 12

= 48 x + 20

2) y
= x (4 x + 3)

v 4 x + 3 and
We have u and v to be u = x and = u = x can be written as

1 du 1 − 1 2 dv
u=x 2
and = x also =4
dx 2 dx

dy dv du
= u +v
dx dx dx

dy 1 1 1
= ( x 2 )4 + (4 x + 6) x − 2
dx 2

1 1 1
= 4x 2
+ 2x 2
+ 3x − 2

1 3
= 6x 2
+
x

3) y =( x + 4)(2 x + 2)

dy dv du
Let=
u x + 4 and =
v 2 x + 2 then using = u + v then=
u x +4
dx dx dx

1 du 1 − 1 2
can be written as=
u x 2
+ 4 and = x v 2 x + 2 and
then given that =
dx 2

dv
=2
dx
CHAPTER 9 Calculus I: Differentiation, Rules of differentiation 187

dy dv du
= u +v
dx dx dx

dy 1 1 1
= ( x 2 + 4)2 + (2 x + 2) x − 2
dx 2

1 1 1
= 2x 2
+8+ x 2
+ x− 2

1 1
= 3x 2
+ x− 2
+8

Practice Problem

1) y =( x 2 + 2)(5 x + 4)

2) y
= 4
x (7 x + 4)

3) y =(3 x + 8)( x + 7)

6 The Quotient Rule or Division Rule


The quotient rule is to differentiate division of two functions. The
denominator multiply with derivative of the numerator, subtract the
numerator multiply with the derivative of the denominator and divide
the result with the denominator squared.
u
Let consider the function of the format y = then the above can be represented
v

du dv
dy v dx − u dx
=
dx v2
CHAPTER 9 Calculus I: Differentiation, Rules of differentiation 188

Example 9.9

2− x
(a) y =
4 + x2

2x
(b) y =
1+ x

8x − 5
(c) y =
3x 2 − 4

Solution

2− x
(a) y =
4 + x2

Given that u= 2 − x and v= 4 + x 2

du dv
Differentiate u= 2 − x to = −1 and again differentiate v= 4 + x 2 to = 2x
dx dx

du dv
dy v dx − u dx
then =
dx v2

dy (4 + x 2 )(−1) − (2 − x)2 x
=
dx (4 + x 2 ) 2

−4 − x 2 − 4 x − 4 x 2
=
(4 + x 2 ) 2

−5 x 2 − 4 x − 4
=
(4 + x 2 ) 2
CHAPTER 9 Calculus I: Differentiation, Rules of differentiation 189

2x
(b) y =
1+ x

Given that u = 2 x and v = 1 + x

du dv
Differentiate u = 2 x to = 2 and v = 1 + x to = 1 given that
dx dx

du dv
dy v dx − u dx
= So we can differentiate
dx v2

dy (1 + x)2 + (2 x)(1)
=
dx (1 + x) 2

2 + 2x + 2x
=
(1 + x) 2

2 + 4x
=
(1 + x) 2

8x − 5
(c) y =
3x 2 − 4

du dv
v 3 x 2 − 4 we obtain
u 8 x − 5 and=
Take = = 8 and = 6 x respectively thus
dx dx

du dv
dy v dx − u dx
Using the formula =
dx v2

dy (3 x 2 − 4)8 − (8 x − 5)(6 x)
=
dx (3 x 2 − 4) 2

24 x 2 − 32 − 48 x 2 − 30 x
=
(3 x 2 − 4) 2

−24 x 2 − 30 x − 32
=
(3 x 2 − 4) 2
CHAPTER 9 Calculus I: Differentiation, Rules of differentiation 190

7. The chain rule


y ( x + 3) 2 we can expand ( x + 3)( x + 3) = x 2 + 6 x + 9
If for instance we have =
then we can differentiate

dy
= 2 x + 6 = 2( x + 3)
dx

y ( x + 3)8
Consider the following function =

We can expand it ( x + 3)( x + 3)( x + 3)( x + 3)( x + 3)( x + 3)( x + 3)( x + 3) . It is very
involving and laborious thus where chain rule applies.
Again given= y (4 x + 8)5 we cannot say that the derivative is 5(4 x + 8) 4
In chain rule differentiate the function that is outside the bracket and multiply
with derivative of the function inside the bracket.
In general given y is a function of u which is the function of x , we can use the

dy dy du
Formula = ×
dx du dx

Example 9.10

Differentiate

y (4 x + 8)5
1) =

2)
= y (2 x 2 + 4 x)7

10
1 
3)=y  x − 3
2 
CHAPTER 9 Calculus I: Differentiation, Rules of differentiation 191

Solution

y (4 x + 8)5
1) =

u 4 x + 8 and
To solve the above equation we must establish = y = u 5 then

dy dy du du
using the general formula = × we can obtain = 4 and
dx du dx dx

dy
= 5u 4 Respectively then we can multiply
du

dy
When we have = 5u 4 we know that =
u 4 x + 8 then we can say that
du

dy
= 5(4 x + 8) 4
du

Thus

dy
=5(4 x + 8) 4 (4) =20(4 x + 8) 4
dx

2) =y (2 x 2 + 4 x)7

u 2 x 2 + 4 x and y = u 7 the derivative we get


In the above function =

du
for the above functions are = 4x + 4
dx

dy
u 6 7(2 x 2 + 4 x)6
and = 7=
du

dy dy du
= ×
dx du dx
CHAPTER 9 Calculus I: Differentiation, Rules of differentiation 192

dy
=7(2 x 2 + 4 x)6 (4 x + 4)
dx
=(28 x + 28)(2 x 2 + 4 x)6

10
1 
3)=y  x − 3
2 

1
In the above function=
u x − 3 and y = u10 thus the derivative are
2

9
du 1 dy 1 
= and= 10
=u 9 10  x − 3  thus in general we use the
dx 2 du 2 

dy dy du
following formula = ×
dx du dx

and we obtain
9 9
dy 1  1 1 
= 10  x − 3  = 5  x − 3 
dx 2  2 2 

Practice Problem

y (5 x + 8)8
(a) =

(b) =y (6 x 2 + 3 x)10

7
1 
(c) =y  x − 8
4 
Maximization and Minimization of Function

Learning Objectives

At the end of this section the students should be able to:


• Use of first derivative to find the stationary points of a function.
• Use the second derivative to determine whether the stationary point is
maxima and minima.
• Find the maximum and minimum points of a function.
• Use stationary points to sketch graphs of fuctions.

Stationary Points

This section is concerned with the application in which will are interested with the stationary
points. When we have a non-linear equation which have take the shape of U and inverted U at
the symmetry of the parabola we have the turning points. The turning points which are also
called the stationary point are either maxima or minima.
It is maxima when the shape is inverted U and minima when it is U shaped.

Figure 9.9

Turning point
(maxima)
CHAPTER 9 Calculus I: Differentiation, maximization and minimization of functions 194

Figure 9.10

Turning point
(minima)

Example 9.11

Consider the following equation


y =x 2 − 10 x + 9

Differentiate

Solution
dy
= 2 x − 10
dx

For stationary point

dy
=0
dx

2 x − 10 =
0

2 x = 10

x=5
CHAPTER 9 Calculus I: Differentiation, maximization and minimization of functions 195
We can calculate the value of y by substituting x =5 to the equation y =x 2 − 10 x + 9

52 10 ( 5 ) + 9
y =−

y = 25 − 50 + 9

y = −16

The stationary points are (5, -16)


To know whether the points are minima or maxima we get the second derivative indicated as

d 2y
or f "( x )
dx 2

d 2y
=2
dx 2

When the second derivative is positive then the stationary points are at minima

To obtain the points at which it cut the x axis we solve the equation as quadratic

y =x 2 − 10 x + 9

−b ± b 2 − 4ac
x=
2a

10 ± 102 − 4 (1)( 9 )
=
2

10 ± 100 − 36
=
2

10 ± 64
=
2

10 ± 8
=
2
CHAPTER 9 Calculus I: Differentiation, maximization and minimization of functions 196

10 + 8 18
x
= = = 9
2 2

10 − 8 2
x= = = 1
2 2

=x 1=
or x 9

Sketching the when using the value of the x we have obtained after solving the quadratic
equation as where the graph will cut the x-axis

Figure 9.11
y =x 2 − 10 x + 9
9

5
1 9
(5, -21) minima
-16

1
To obtain the symmetry of the parabola we (1 + 9 ) =
5
2

To whether the minima remember to differentiate the second time in our case we had obtained

d2y
=2
dx 2

When the second derivative is positive the stationary point is minima


CHAPTER 9 Calculus I: Differentiation, maximization and minimization of functions 197

Example 9.12

Lets’ consider another non – linear system

y=− x2 + 6x + 7

Differentiate

Solution

dy
=−2 x + 6
dx

For a stationary point

dy
=0
dx

−2 x + 6 =0
−2 x =
−6
x=3

Substitute x to obtain y

y=− x2 + 6x + 7
x=3
− ( 32 ) + 6 ( 3) + 7
y=
y =−9 + 18 + 7
y = 16

The stationary points are (3, 16)

To determine whether they are maxima or minima we get the second order derivative

d 2y
=- 2
dx 2
CHAPTER 9 Calculus I: Differentiation, maximization and minimization of functions 198

When the second derivative is negative we say it is maxima for our case the second derivative is
negative 2 thus the stationary points are maxima

We can determine the points at which the curve cut the x –axis by solving as quadratic equation.

− x2 + 6 x + 7 =0

−b ± b 2 − 4ac
x=
2a

−6 ± 62 − 4 ( −1)( 7 )
=
2 ( −1)

−6 ± 36 − −28
=
−2

−6 ± 64
=
−2

−6 ± 8
=
−2

−6 + 8 2
x= = = −1
−2 −2

−6 − 8 −14
=x = = 7
−2 −2

−1
x= or 7
x=

We can now sketch the graph as shown below


CHAPTER 9 Calculus I: Differentiation, maximization and minimization of functions 199

Figure 9.12

(3, 16) maxima

16

y=− x2 + 6x + 7

-1 3
7

Application of differentiation
In most case the application of differentiation to business is inform of the differentiating total
revenue (TR) to get marginal revenue (MR)

dTR
= MR
dq

On the other hand the application is used to different ate total cost (TR) to get the marginal cost
(MC) which is shown below.

dTC
= MC
dq
CHAPTER 9 Calculus I: Differentiation, maximization and minimization of functions 200

Example 9.13

Company A sells its output to company B for KES 200 per unit. The Cost of the sales per week in
company A are given by the function TC =2Q2 + 40Q − 80 where Q is the value of the weekly
sales.
Company B uses the output of company A to manufacture a product whose demand is
dependent on the sale price. The revenue per week of company B is given= as R 1000Q − 16Q2
and the cost per week of company B excluding the cost of the products bought from company A
are given C =2Q2 − 120Q − 400
Company A can restrict the weekly supply of its product to company B but cannot raise the price
above KES 200. The two companies are considering whether to merge together into a single
company.

Determine:
At what weekly sales will company A maximizes it profit.

What will be the profit or loss in company B if company A were to supply a profit maximizing
quantity of its product weekly?

If the two companies were to merge into one what would be the profit maximizing output per
week and what would be the weekly profit.

Solution

Weekly sales of company A

TRA = PQ

TRA= 200 × Q ⇒ 200Q

TC A =2Q2 + 40Q − 80

π=
A TRA − TC A
CHAPTER 9 Calculus I: Differentiation, maximization and minimization of functions 201

= 200Q − ( 2Q2 + 40Q − 80 )

= 200Q − 2Q2 − 40Q + 80

= 160Q − 2Q2 + 80

dπ A
= 160 − 4Q
dQ

When we consider the stationary points


Then

dπ A
=0
dQ

Equate the above to zero

160 − 4Q =
0

−4Q =
−160

Q = 40

To determine whether it is maximum we get the second derivative

d 2π A
= −4
dQ2

Company A profit function is maximum because the second derivative is negative 4 at the level
of output of 40 units.

The total revenue of company B can be obtained as

TRB = PQ

TRB Q (1000 − 16Q )


=

TRB 1000Q − 16Q2


=
CHAPTER 9 Calculus I: Differentiation, maximization and minimization of functions 202

The total cost will be added the revenue of company A which is a cost to company B

TC B = 2Q2 − 80Q − 400 + 200Q

=2Q2 + 120Q − 400

The profit function of company B

π=
B TRB − TC B

= 1000Q − 16Q2 − ( 2Q2 + 120Q − 400 )

= 1000Q − 16Q2 − 2Q2 − 120Q + 400

= 880Q − 18Q2 + 400

Q = 40

880 ( 40 ) − 18 ( 402 ) + 400

= 6800

To determine whether it maximum profit we get the second derivative

dπ B
= 880 − 36Q
dx

d 2π B
= −36
dx 2

The second derivative is negative thus the profit KES 6800 is maximum

When the two companies merge we add both the total cost

TC A + TC B
CHAPTER 9 Calculus I: Differentiation, maximization and minimization of functions 203

TC A+B = 2Q2 + 40Q − 80 + 2Q2 + 120Q − 400

=4Q2 + 160Q − 480

The total revenue to be used is that of company B because total revenue for company A is a cost
to company B

π A+B = 1000Q − 16Q2 − ( 4Q2 + 160Q − 480 )

= 1000Q − 16Q2 − 4Q2 − 160Q + 480

= 1040Q − 20Q2 + 480

dπ A+B
= 1040 − 40Q
dQ

dπ A+B
=0
dQ

1040 − 40Q =
0

1040 = 40Q

Q = 26

The second derivative

d 2π A+B
= −40
dQ2

When Q = 26 the profit are maximum to produce a profit of KES 14,000

π A+B = 1040 ( 26 ) − 20 ( 262 ) + 480

π A+B = 14000
CHAPTER 9 Calculus I: Differentiation, maximization and minimization of functions 204

Example 9.14

The weekly cost of producing q calculators is estimated using the function:

=C 0.2q 3 – 6q 2 - 220q + 500 ( KES 00 , 000 )

Where q is the number of calculators produced per week in ten thousands of units. The
revenue made in selling the calculators is given by the equation:

 R = 214 – 0.25q 2 ( KES 00, 000 )


Find the:
a) The initial set up cost before production begins.
b) An equation for estimating the weekly profit and level of output that maximize the
profit.
c) The maximum the profit at this level.

Solution

Initial cost is given when q = 0 which is determined as follows

C
= 0.2q 3 – 6q 2 − 220q + 500 ( KES 00 , 000 )

q=0

0.2 ( 0 ) – 6 ( 0 ) − 220 ( 0 ) + 500 ( KES 00 , 000 )


3 2
=

= KES 50,000,000

An equation for estimating the weekly profit.


CHAPTER 9 Calculus I: Differentiation, maximization and minimization of functions 205

Profit
= TR − TC

π 214 – 0.25q 2 – ( 0.2q 3


– 6q 2 - 220q + 500 )

π =− 286 + 5.75 q 2 – 0.2q 3 + 220q

−0.2 q 3 + 5.75 q 2 - 220q − 286 ( KES 00, 000 )


π =

Level of output

π =
−0.2 q 3 + 5.75 q 2 +220q − 286


−0.6q 2 + 11.5q + 220
=
dq


=0
dq

The second derivative will be obtained as


−0.6q 2 + 11.5q + 220
=
dq

d 2π
−1.2q + 11.5
=
dq 2

−0.6q 2 + 11.5q + 220 =


0

−b ± b 2 − 4ac
q=
2a

Hence
CHAPTER 9 Calculus I: Differentiation, maximization and minimization of functions 206

−11.5 ± 11.52 − 4 ( −0.6 )( 220 )


=
2 ( −0.6 )

−11.5 ± 660.25
=
−1.2

−11.5 ± 25.7
=
−1.2

−11.5 + 25.7 14.2


= = = −11.83
−1.2 −1.2

−11.5 − 25.7 −37.2


= = = 31
−1.2 −1.2

−11.83
q= or 31
q=

−1.2 ( −11.83) + 11.5 =


= 25.696

−1.2 ( 31) + 11.5 =


25.696 =
−25.7

d 2π
Thus the quantity of 31 units will give a maximum profit because the is -25.7
dq 2

π =
−0.2 q 3 + 5.75 q 2 +220q − 286

q = 31

−0.2 ( 313 ) + 5.75 ( 312 ) +220 ( 31) − 286


π =

π =
−5958.2 + 5525.75 + 6820 − 286

π = 610,155, 000
CHAPTER 9 Calculus I: Differentiation, maximization and minimization of functions 207

Practice Problem

1. A Company estimates that the total cost of producing x units of its popular product
Alpha is given by the equation: C =+2000 200x – 10x 2 + 2x 3 . The marketing
department further estimates that the revenue made in selling x units is given by:
R = 800x – 2x 2 .

Determine the number of that should be produced to maximize the total profit.

2. A manufacturer expects that the total cost (in Kenya shillings) of producing x units of his
product will be given

( x ) 0.000001x 3 − 0.0015x 2 − 240 x + 112500


C=
Required
a) Define the terms average cost and marginal cost
b) Derive both the average cost and marginal cost functions for the above
c) How many units would he have to produce in order to minimize his total costs?
d) Explain your findings in c) above
e) What is the cost of producing the 7000th unit of the product?

3. What is the importance of second derivative in business


Partial Differentiation

Consider the function that relates the dependent variable y and three different independent
variables. The process of determining the rate of change with the three variables is called
partial differential. The procedure involves differentiating y with respect to x1 when keeping x2
and x3 constant.

Example 9.15
y = x 5 + 3xz + z 2 + 100

Solution

∂y
= 5 x 4 + 3z
∂x
∂y
=0
∂x
5 x 4 + 3z =0
∂y
= 3x + 2z
∂z
∂y
=0
∂z
3x + 2z = 0

Solve for the value of x and z


CHAPTER 9 Calculus I: Partial Differentiation 209

∂2 y ∂2 y
20
= x3 3
∂x 2 ∂x∂z

∂2 y ∂2 y
2= 3
∂x 2 ∂z∂x

So that to find the Hessian determinant that has got a positive we use the following

 ∂2 y ∂2y 
 ∂x 2 ∂x∂z 
 2
∂y ∂2y 
 ∂z∂x ∂z 2 

 ∂2y ∂2y   ∂2y ∂2y 


H=  2 × 2  −  × 
 ∂x ∂z   ∂z∂x ∂x∂z 

Example 9.16
Given the TR = 5Q + A − Q2 + AQ − A2 where Q represent the total output and A advertising
expenditure. Determining the level of output at which the TR will be maximized.

Solution

TR = 5Q + A − Q2 + AQ − A2
∂TR
=5 − 2Q + A =0
∂Q
∂TR
=1 + Q − 2 A =0
∂A

Solve the simultaneous equation

TR = 5Q + A − Q2 + AQ − A2
5 − 2Q + A = 0 ⇒ 5 = 2Q − A
1 + Q − 2 A =0 ⇒ 1 =−Q + 2 A
CHAPTER 9 Calculus I: Partial Differentiation 210

5 2Q − A
=
1 =−Q + 2 A
3Q = 7
7
Q= = 2.33
3
11
A = 3.67
=
3

Second order derivative

∂ 2TR ∂ 2TR
=
−2 1
=
∂Q2 ∂Q∂A

∂ 2TR ∂ 2TR
=
−2 1
=
∂A2 ∂A∂Q

 ∂ 2TR ∂ 2TR   ∂ 2TR ∂ 2TR 


H = 2 × 2 − × 
 ∂Q ∂A   ∂Q∂A ∂A∂Q 

H= ( −2 × −2 ) − (1 × 1) = 3

Example 9.17
A company produces two goods A and B and the prices are given below

P=
1 50 − x
P=
2 95 − 3y

The total cost function is given as x 2 + 3xy + y 2

a) Determine the profit function

b) Determining the value of x and y that maximizes profit

c) Deduces the corresponding prices


CHAPTER 9 Calculus I: Partial Differentiation 211

Solution

TR1 + TR2 =
=PQ
TR1= x ( 50 − x ) + y ( 95 − 3y ) ⇒ 50 x − x 2 + 95y − y 2
π= TR − TC

= 50 x − x 2 + 95y − 3y 2 − ( x 2 + 3xy + y 2 )
= 50 x − x 2 + 95y − 3y 2 − x 2 − 3xy − y 2
= 50 x − x 2 − x 2 + 95y − 3y 2 − y 2 − 3xy
= 50 x − 2 x 2 + 95y − 4 y 2 − 3xy

π = 50 x − 2 x 2 + 95y − 4 y 2 − 3xy
∂π
= 50 − 4 x − 3y = 0
∂x
∂π
= 95 − 8y − 3x = 0
∂y

Solve the simultaneous equation

−4 x − 3y =
−54
−8y − 3x =−95
x=5
y = 10

To determine whether Hessian determinant is positive

∂ 2π ∂ 2π
=
−4 −3
=
∂x 2 ∂x∂y

∂π ∂ 2π
−8
= −3
=
∂y 2 ∂y∂z
CHAPTER 9 Calculus I: Partial Differentiation 212

 ∂ 2π ∂ 2π   ∂ 2π ∂ 2π 
H =  2 × 2  - × 
 ∂x ∂y   ∂x∂y ∂y∂z 

( −4 × −8 ) − ( −3 × −3) =23
It is maximum because the Hessian determinant is positive 23
The price can be calculated as follows
We can find the price by multiply with the value of x and y to P1 and P2

P=
1 50 − x
x=5
P1 = 50 − 5 = 45

P=
2 95 − 3y
y = 10
P=
2 95 − 30
P2 = 65
The profit can be calculated as follows
π = 50 x − 2 x 2 + 95y − 4 y 2 − 3xy
π = 50 ( 5) − 2 ( 52 ) + 95 (10 ) − 4 (102 ) − 3 ( 5)(10 )
= 600

Practice Problem

1. A firm selling trade directory has developed a profit function as follows

P= 9D − 0.0005D2 + 0.06DA − 80 A2 − 5000


Where D is the number of directory sold, a is the number of advert pages
How many directories containing how many advert pages should be sold in order to
maximize profit.

2. An non-profit making organization has started a small project whereby the aim is to
provide employment to as many people as possible without loss or profit. The weekly
total earnings of x workers can be estimated using the following equation

2
E =98 + 16 x + x 2
3
The revenue generated by the x workers
= C 100 x − 4 x 2 .
Required
CHAPTER 9 Calculus I: Partial Differentiation 213

a) Determine how many workers will generate the maximum possible revenue

b) Determine the maximum no of workers the organization will employ with making
a loss

3. A company has determined through regression analysis that its sales are in two different
medium. This is given by the following relationship

Z = 200 x + 100y − 10 x 2 − 20y 2 + 20 xy

Where x is number of news paper and y the number of magazines.


Determine the company’s optimal level of sales.

4. The total profit per acre on a wheat farm, had been found to be related to the
expenditure per acre for labor and sol conditions as given by the following function

Profit =48 x =60y + 10 xy − 10 x 2 − 6y 2

Where x represents the expenditure per acre spent on labor and y represent the
expenditure per acre spent on soil improvement.
Determine the value of x and y that maximize profits

5. A company deals with two types of ladies wear with demand functions;

P=
x 25 − 4 x for product x
P=
y 10 − 6y for product y
Total cost function
TC =20 + 3x 2 =2y 2 =2 xy

a) What is the level of output to maximize the profit of both products

b) Compute these maximum profit for both products

c) Suppose that only one product is produce.


d) Compute the production level and profits to maximize profit level x when y is not
being produced

e) Production level and profit to maximize. Profit when x is not produced.


Constrained Optimization
Managers face many constrains in their decision making for example marketing manager is
required to maximize sales subject to an advertising budget, a production manager is required
to maximize output subject a minimum use of labor, machines and other inputs. This are
constrained optimization problems to solve this problem we can use Lagrange multipliers. It
handles non linear constrains and problems that involve more than two variables with ease.
We have two types of function in Lagrange multiplier
Objective functions that is what you opt to achieve f ( x , y )
Constrains
Subject to ∅ ( x , y ) =
m
When dealing with Langrage multiplier we need to define the Lagrange function that is take the
objective function and constrained in a single function and rearrange the function and multiply
by the Lagrange which is usually scalar λ (lambda) so in general

Objective function
f ( x,y )
Subject to a constant

∅( x,y ) =
m

g ( x , y , λ )= f ( x , y ) + λ M − Q ( x , y ) 

Differentiate and solve the resulting simultaneous functions

∂g
=0
∂x
∂g
=0
∂y
∂g
∂λ
CHAPTER 9 Calculus I: Constrained Optimization 215

Example 9.18

Find the optimal value from this function

x 2 − 3xy + 12 x

Subject to
2 x + 3y =6

Solution

f ( x , y ) =x 2 − 3xy + 12 x
Q ( x , y ) = 2 x + 3y = 6

g ( x , y , λ ) =x 2 − 3xy + 12 x + λ 6 − ( 2 x + 3y ) 
= x 2 − 3xy + 12 x + 6λ − 2 xλ − 3yλ
∂g
= 2 x − 3y − 12 − 2λ = 0..................................................... ( i )
∂x
∂g
= −3x − 3λ = 0................................................................. ( ii )
∂y
∂g
=6 − 2 x − 3y =0.............................................................. ( iii )
∂λ

Now we can solve the above three simultaneous equation


2 x − −3y − 12 − 2λ =0................................................................ ( i )
0............................................................................... ( ii )
−3x − 3λ =
6 − 2 x − 3y =0............................................................................ ( iii )
Take the first two equations
CHAPTER 9 Calculus I: Constrained Optimization 216

2 x − 3y − 2λ 12................................................................ ( i ) × 3
−3x − 3λ 0...................................................................... ( ii ) × 2

36................................................................ ( i )
6 x − 9 y − 6λ =
0...................................................................... ( ii )
−6 x − 6λ =
36..................................................................... ( iv )
−9y − 12λ =

Take equation (ii) and (iii)

−3x − 3λ 0............................................................................... ( ii ) × 2
2 x + 3y 6.................................................................................. ( iii ) × 3

0............................................................................... ( ii )
−6 x − 6λ =
18.................................................................................. ( iii )
6 x + 9y =
18..................................................................................... ( v )
9 y − 6λ =

Solve equation (iv) and (v)

36..................................................................... ( iv )
−9y − 12λ =
18............................................................................ ( v )
9 y − 6λ =
−18λ = 54
λ = −3
36..................................................................... ( iv )
−9y − 12λ =
λ = −3
−9y − 12 ( 3) =
36
−9y + 36 =
36
−9y = 0

y=0
x = −3
CHAPTER 9 Calculus I: Constrained Optimization 217

Practice Problem

1. A firm manufactures’ specialty bicycles has a profit function P = 5x 2 + 10 xy + 3y 2 + 240 x


where x denotes the number of frames and y the number of wheels. Assuming that the firm
doesn’t want any spare frames or wheels left over at the end of the production run , find
the maximum profit.

2. The total profit per acre on a wheat farm has been found to be related to the
expenditure per acre for labor and soil conditioners given the following function:

Profit = 440x + 60y + 10xy − 10x 2 − 6y 2

Where x represents the expenditure per acre spent on labor and y represents the
expenditure per acre spent on soil improvement
Determine the value of x and y that maximizes profit hence maximum profit

3. A producer of two goods A and B has a total cost given as where x and y denotes the
quantity of goods respectively. If P1 and P2 denotes the corresponding prices then the
demand equation are ;

Required
Determine the optimal profits if the companies total cost are fixed at KES 100

4. A company produces and sells two products A and B in related markets. The total
revenue made in selling x units of A and y units of B is given by:

R = 13x + 13y – 2xy – 2x2 – 2y2


The total cost is given by C = x + y.

a) Determine the total profit function

b) Compute the output level that would lead to maximum profit.

c) Explain the use of calculus in business decision making


CHAPTER 10 CALCULUS II:
INTEGRATION

Learning Objectives
At the end of this section the students should be able to:
 Integrate function
 Recognize the notation for indefinite integration
 Find total cost given the marginal cost
 Find the total revenue given marginal revenue

The anti – derivative

Integration can be regarded as the opposite process of differentiation hence called the anti
differentiation. However for the definite integral the result is single number or numeric result.

∫ (2x + 20 x )dx
4

Integral of Variable integrated


is x

Example 10.1
Evaluate
∫ (2x + 20 x )dx
4

Solution

∫ (2x + 20 x )dx
4
CHAPTER 10 Calculus II: Integration 219

 2 x 1+1 20 x 4 +1 
∫  1 + 1 + 4 + 1 + c dx
=x 2 + 4 x 5 + c

A problem occurs when differentiating a constant will always give zero.


The best that can be done is integrate and add a c which is constant of integration.
There are times when information is known about a particular situation that enables the
constant of integration to be calculated.
The rule for integration is a simple function of x, kx n is to increase the power of x by 1 and
divided the whole function by n + 1
kx n +1
That is ∫ ( kx n )dx ⇒ +c
n +1

Example 10.2

Evaluate
∫ ( 3x − 4 x )dx
2

∫ (8x − 3x 2 + 6 x − 10 )dx
3

∫ (14 + 12x )dx


2

Solution

∫ ( 3x − 4 x )dx
2

3 x 2 +1 4 x 1 +1
− +c
2+1 1+1
=x 3 + 2 x 2 + c
CHAPTER 10 Calculus II: Integration 220

∫ (8x − 3x 2 + 6 x − 10 )dx
3

8 x 3 +1 3 x 2 +1 6 x 1 +1
− + − 10 x 0 +1 + c
3+1 2+1 1+1

2 x 4 + x 3 + 3x 2 + 10 x + c

∫ (14 + 12x )dx


2

14 x 1+1 12 x 2+1
+ +c
2 3

7x2 + 4 x 3 + c

Definite Integral
In general when dealing with definite integral we just integrate just like the other normal
integration but his time with definite values.
a
The symbol for definite integral ∫ ( )dx where a is a definite and b is a lower value.
b

Example 10.3
Evaluate
4

∫ ( 3x + x )dx
2
a)
0

∫ (5x + 3 x 2 + 5 x )dx
4
b)
−2

Solution

∫ ( 3x + x )dx
2

0
CHAPTER 10 Calculus II: Integration 221

4
 3 x 2+1 x1+1 
∫0  3 + 2 + c dx

4
x2
∫x +
3
+c
0
2

When the value of x = 4 or x = 0

 3 42   3 02 
 4 + + c  − 0 + + c 
 2   2 

[64 + 8 + c ] − [0 + 0 + c ]

[72 + c ] − [0 + c ]

72 + c − c

= 72

∫ (5x + 3 x 2 + 5 x )dx
4

−2
CHAPTER 10 Calculus II: Integration 222

6
 5 x 4+1 3 x 2+1 5 x1+1 
∫−2  5 + 3 + 2 + c dx

6
 5 5x2 
∫−2 
3
x + x + + c dx
2 

6  5 ( 62 )   5 ( −22 ) 
∫    
5 3 5 3
6 + 6 + + c − −2 + −2 + + c
−2 
2   2 
  

( 7776 + 216 + 90 + c ) − ( −32 − 8 + 10 + c )

= 8082 − −30

= 8052

Application of Definite Integral

When you require a numeric insert say the total revenue between two activity levels the
expression is termed as a definite integral. In the indefinite integration the result is a function of
x or any variable that is given say a, b, c, d ... .
a
The symbol for definite integral ∫ ( )dx where a is a definite and b is a lower value
b

Example 10.4

The total revenue obtained from selling x hundred items in a particular day is given by R which is
a function of variable x. The marginal revenue MR
= 20 − 4 x
Determine the total revenue function

Find the number of items sold in one day that will maximize the total revenue and evaluate TR
at that point.
CHAPTER 10 Calculus II: Integration 223

Solution
MR
= ∫ (20 − 4 x ) dx
= 20 x − 2 x 2 + c
c=0
TR 20 x − 2 x 2
=

dTR
= 20 − 4 x = 0
dx
20 − 4 x = 0
4 x = 20
x=5
dTR
= 20 − 4 x = 0
dx
20 − 4 x =0
4 x = 20
x=5
d 2TR
= −4
dx 2

TR 20 ( 5) − 2 ( 52 )
=
TR 100 − 50
=
= 50

It is a maximum because the second derivative is negative 4 and gives 50

Example 10.5
Your firm has recently stated to give economic advice to your client acting as a consultant you
have estimated the demand function to be AR = 200 − 8 x where AR is the average cost and x is
the output. AR is selling per unit. Investigation of the client cost profile the marginal cost is
given by MC =x 2 − 28 x + 211
Further investigation has shown that the firm’s cost when not producing output are 10
a) Determine the TC

b) Determine the TR
CHAPTER 10 Calculus II: Integration 224

c) Find the level of max profit

Solution

AR x ( 200 − 8 x )
=
a).
TR 200 x − 8 x 2
=

MC =x 2 − 28 x + 211
b).
∫(x − 28 x + 211)
2
TC =

x 2+1 28 x 1+1
= − + 211x + c
2+1 1+1
1 3
x − 14 x 2 + 211x + c
3
c = 10
1 3
x − 14 x 2 + 211x + 10
3

c)Profit function
CHAPTER 10 Calculus II: Integration 225

1 
200 x − 8 x 2 −  x 3 − 14 x 2 + 211x + 10 
3 
1
200 x − 8 x 2 − x 3 + 14 x 2 − 211x − 10
3
1
200 x − 211x − −8 x 2 + 14 x 2 − x 3 − 10
3
1
−11x + 6 x 2 − x 3 − 10
3

= −11 + 12 x − x 2 = 0
dx

x =1 or 11

d 2π
= 12 − 2 x
dx 2
x = 11
12 − 2 (11) =
−10
12 − 2 (1) =
10

When x is 11 it is at maximum because the second derivative is -10

Practice Problem

1. The marginal revenue (MR) of a product is KES 30 and we know that total revenue is zero
when is zero, total cost equal KES 250,000
a) Find the total revenue
b) Find the total cost
c) Find the profit function

2. Evaluate;

a.

∫ (1 − 2 x + 10 x
2
b. − 12 x 4 )dx
CHAPTER 10 Calculus II: Integration 226
5
c. ∫ (200 x − 10 x
2
+ 21x 3 + 100)dx
0

3. If he marginal cost of making x pairs of gloves is shillings, find the total cost of
increasing production from 400 to 500 pairs.

4. XYZ that a company is faced by the following MC= 16 − 2Q , MC = 40 − 16Q . It is known


that fixed cost are 8 when production is zero. Find the output to give maximum profit.

5. Consider a firm whose AR and the TC are given as follows

AR = 20 − 5Q − Q2
4 14Q − 5Q2 + Q3
TC =+

a) Determine the firms MR and MC.


b) Find the firms maximizing output and unit price.

6. Suppose that a firm has its MR and TC functions estimated as follows;-

MR
= 300 − 6Q
MC= 50 + 4Q
The fixed costs are set to 500 when the firm is not producing.
a) Find the profit function of the firm
b) Find the quantity of output that maximizes the firm’s profit hence the maximum
profit

7. Company XYZ employed a cost accountant who developed two functions to describe the
operation of the firm. He found the marginal revenue to be MR = 25 − 5x − 2 x 2 and the
marginal cost function to be MC = 15 − 2 x − x 2 where x is the level of output. Determine
the profit maximizing output of this firm and the total profit at that point.

8. The marginal revenue (MR) of a product is KES30 and we know that total revenue is zero
when x (number of units sold) is zero. The marginal cost (MC) is KES 22.50 and we are
given that when x is zero, total costs equal KES 250,000
a) Find the total revenue function
b) Find the total cost function
c) Find the profit function
CHAPTER 10 Calculus II: Integration 227

9. A company manufactures large scale items. It has been shown that the marginal cost , is
thousands of shillings where is the number of units of output per
annum. It has also been shown that the marginal revenue is thousands
of shillings.

a) Establish the total cost function


b) Establish the total revenue function
c) Establish the break-even situation of the company
d) Determine the number of units of output that would maximize
CHAPTER 11 NON – LINEAR
EQUATIONS

Learning Objectives
At the end of this section the students should be able to:
−b ± b 2 − 4ac
Solve a quadratic equation using the quadratic formula x =
 2a
 Solve a quadratic equation by factorizing method
 Sketch the graph of a quadratic equation on a graph
 Determine the equilibrium price given quadratic demand equation and supply
function

Quadratic functions
In a quadratic the highest power of the independent variable is 2. Quadratic is solved by finding
the values of x that satisfy it solution or roots. The shape of the quadratic equation is called a
parabola.

A quadratic equation can either have two, one or no solution.

This is where the curve crosses the x-axis and has two solutions

−ax 2 + bx + c =0
2
ax + bx + c =0
CHAPTER 11 Non-Linear equations 229

The curve turn round just as it touches the x-axis, it has one solution.

One
solution

The curve turns round before it ha a chance to cross the x – axis, it has no solution

These are equation of the form ax 2 + bx + c =0

Discriminant

The expressions b − 4ac which appear under the square root sign of the quadratic formula
2

determine the character of the solution and is called the discriminant. Specifically when the
value of b2 − 4ac is greater than zero then we have two solution, If the discriminant is equal to
zero we have one solution and if it is less than zero we have no solution.

b2 − 4ac > 0 two solution


b2 − 4ac =
0 one solution
b2 − 4ac < 0 no solution
CHAPTER 11 Non-Linear equations 230

One can solve the quadratic equation by:

 Factor method
 Completing the square method
 Quadratic formula
 Graphical method

In this level we are going to solve the quadratic equation by only two methods that is the facto
method and quadratic method

Example 11.1
Solve the following quadratic equations factor method

x 2 + 13x + 42 =
0

6 x 2 − 5x − 6 =0

Solution

x 2 + 6 x + 7 x + 42

Take the number that is before x2 called the coefficient of x2 and multiply with the constant
which the number without the variable x and x2
What we obtain is called the product
Product =1 × 42 =42
Identify two numbers when multiplied we get 42 and when we add them together we obtain 13
i.e. the coefficient before x is called the sum

For our case the numbers are 6 and 7


We need to factor as follows.
CHAPTER 11 Non-Linear equations 231

x 2 + 6 x + 7 x + 42

Find what is common when they are in pairs


x 2 + 6 x + 7 x + 42
x ( x + 6) + 7( x + 6)
( x + 7 )( x + 6 )( x + 6 )
Since these two are the
same only consider one

( x + 7 )( x + 6 ) =
0

Now solve for the value of x

( x + 7) =
0
x +7= 0
x = −7
( x + 6) =
0

x +6= 0
x = −6
x=
−6 or −7

6 x 2 − 5x − 6 =0
CHAPTER 11 Non-Linear equations 232

Product = 6 × −6 = 36
Sum = −5
Numbers= 4, −9
6x2 + 4 x − 9x − 6 =0
2 x ( 3x + 2 ) − 3 ( 3x + 2 ) =
0
(2 x − 3)( 3x + 2 )( 3x + 2 ) =
0
(2 x − 3)( 3x + 2 ) = 0
2x − 3 =0
2x = 3
x = 1.5
3x + 2 =0
3x = −2
1
x= −
2
1
=x 1.5 or −
2

Practice Problem
Use factor method to solve the following quadratic functions

a) x 2 + 17 x + 70 =
0

b) 1 − x 2 =
0

c) 2 x 2 − 4 x + 16 =
0

d) 7 x 2 + 21x − 42 =
0

1 2
e) x + 6 x + 27 =
0
3
CHAPTER 11 Non-Linear equations 233

Example 11.2
Solve the following quadratic equation using the quadratic formula

2x2 + 9x + 5 =0

x 2 − 10 x + 9 =0

Solution

The quadratic formula

−b ± b2 − 4ac
x=
2a

Where the equation of the form

ax 2 + bx + c =0

Then we can solve

2x2 + 9x + 5 =0

When

a 2,=
= b 9 and=
c 5

x 2 − 10 x + 9 =0
CHAPTER 11 Non-Linear equations 233

x 2 − 10 x + 9 =0
−b ± b 2 − 4ac
x=
2a
− − 10 ± ( −10 ) − 4 (1)( 4 )
2

x=
2 (1)
10 ± 100 − 36
x=
2
10 ± 64
x=
2
10 ± 8
x=
2
10 + 8 18 10 − 8 2
=x = or =x =
2 2 2 2

x = 9 or 1

Practice Problem

Using quadratic formula solve for x

a) 3 x 2 − 15 x + 6 =0

b) 2 x 2 − 19 x − 10 =
0

c) x 2 − 3 x + 10 = 2 x + 4

d) 4 x 2 + 12 x + 9 =0

e) x 2 + x + 1 =0
CHAPTER 11 Non-Linear equations 235

Application of Quadratic equation


Example 11.3

A company invests in a particular project and it is estimated that after x months of running the
cumulative profit (000’s of shillings) from the project is given by the equation
y= 31.2 x − 3x 2 − 60 where x represents the time in months. The project can run for 9 months at
the most.

Required

a) Calculate the breakeven time points for project

b) What is the initial cost of the project?

c) Draw a sketch of the profit function

d) Use the graph to estimate the best time to end the project (time of maximum profits)

Solution
To determine how to draw the curve there are three steps to remember

Step 1

Determine the basic shape that is whether it has U shaped if a > 0 , inverted U shape if a < 0

Step 2

Determine the y-intercept by identifying the constant that has no variable or substitute x = 0

Step 3

Determine the x-intercept by solving the quadratic equation

i). At Break even point profit is equal to zero


CHAPTER 11 Non-Linear equations 236

31.2 x − 3x 2 − 60 =
0
−b ± b2 − 4ac
x=
2a
−31.5 ± 31.52 − 4 ( −3)( −60 )
x=
2 ( −3)
−31.5 ± 992.25 − 720
x=
−6
−31.5 ± 272.25 −31.5 ± 16.5
=x ⇒
−6 −6

−31.5 + 16.5 −15


=x = 2.5

−6 −6
−31.5 − 16.5 −48
=x = 8

−6 −6

x = 2.5 or 8
ii).The initial cost of the project will be given by the y-intercept on the graph or the cost when
the project is at time zero

y= 31.2 x − 3x 2 − 60
x=0
y= 31.2 ( 0 ) − 3 ( 02 ) − 60
y = −60,000
This imply the initial cost is KES 60,000

We have to follow the three steps

Step 1- The x2 coefficient is less than 1 that is a > 0


Step 2 – The y-intercept is -60
Step 3- we have already solved for x that is x = 2.5 or 8
CHAPTER 11 Non-Linear equations 237

Maximu
m profit
21.1125
Profits

y= 31.2 x − 3 x 2 − 60

2.5 5.25 8 Time in months

-60

The symmetry of the parabola will give the best time to end the project

1
(2.5 + 8 ) =
5.25 months when the project making maximum profit
2

Example 11.4
Given that the supply and demand functions

−QD 2 − 10QD + 150


P=
Calculate the equilibrium price
Plot the curves on a graph paper

Solution
At equilibrium the supply is equal to the demand
CHAPTER 11 Non-Linear equations 238

Q2 + 14Q + 22 =
−Q2 − 10Q + 150
Q2 + Q2 + 14Q + 10Q + 22 − 150 =
0
2Q2 + 24Q − 128 =
0

Now we can solve the above as simultaneous equation

−b ± b2 − 4ac
x=
2a
2
2Q + 24Q − 128 = 0
−24 ± 242 − 4 ( 2 )( −128 )
x=
2 (2)
−24 ± 576 + 1024
x=
4
−24 ± 1600
x=
4
−24 ± 40
x=
4

−24 + 40 16
x
= = = 4
4 4

x =4

−24 − 40 −64
x= = = −16
4 4

x = −16

=x 4 or − 16

To plot the graph we need to plot the to check the point of intersection of the two curves.
The equilibrium quantity is 4 units hence we can calculate the price by either using the demand
function or supply function which gives the same answer.
CHAPTER 11 Non-Linear equations 239

P=−QD2 − 10QD + 150


Q=4
− ( 42 ) − 10 ( 4 ) + 150
P=
P=
−16 − 40 + 150
P = 94

The Equilibrium price is 4 units and the price is KES 94

150
−QD2 − 10QD + 150
P=
(4, 94)

22

-16 4

Practice Problem

1. Given the supply and demand functions

P = 2Q2 + 10Q + 10
−Q2 − 15Q + 52
P=

Determine the equilibrium price and quantity; hence sketch the graph on a graph paper
CHAPTER 11 Non-Linear equations 240

2. The total cost and revenue functions of producing q units of a certain product are given by

TC = 2q + 10
TR = – 2 q2 + 14q
Draw the graph of the profit function and determine the breakeven quantity q and also the
quantity that yields the maximum quantity.

3. The profit function of a certain firm is given by the function

π = aQ2 + bQ + c
It is known that if π =9, 34, and 19 then Q = 1, 2, and 3 respectively. Solve for the values of a, b,
and c. Hence determine the breakeven point.

4. Use Ms Excel to tabulate values of the profit function π = – 0.01q3 + 5q2 + q – 1000 for q = 0,
2, 4, 6, …, 30 and hence plot this function using Ms Excel . Use your graph to determine the
breakeven points and the maximum profit.

5.The demand and supply functions of two interdependent products are given by

Qd1 = 40 − 5 P1 − P2
Qd 2 = 50 − 2 P1 − 4 P2
Qs1 = 4 P1 − 3
Qs2 = 3P2 − 7

6. Where, Qd and Qs denote the respectively quantity demanded and supplied for each product.
Determine the equilibrium price and quantity for each product.

The demand and supply functions of three interdependent products are

Qd1 = 15 − P1 + 2 P2 + P3
Qd 2 = 9 + P1 − P2 − P3
Qd 3 = 8 + 2 P1 − P2 − 4 P3
Qs1 = P1 − 7
Q s2 = 4 P2 − 4
Qs3 = 2 P3 − 5

Determine the market equilibrium price and quantity for each product.
CHAPTER 12 REVENUE, COST
AND PROFIT

Learning Objectives
At the end of this section the students should be able to:
 Sketch the graph of total revenue, total cost, and average cost and
profit functions

 Determine the level of output that maximizes total revenue

 Determine the level of output that minimizes the total cost

 Determine the level of output that maximizes profit

Total revenue is the amount received by the company for the sale of its commodities or goods
or services.

Total revenue (TR) is given by price (P) by the number of units (Q)

TR = PQ

Example 12.1
It is given that where TR is the total revenue and Q is quantity of
commodity. On the graph paper plot the graph of TR against Q

Use the graph to determine the value o Q when TR is maximum


CHAPTER 12 Revenue, cost and profit 242

Solution
Step 1- the is inverted U because the coefficient of Q2 is less than 1 thus it is -2
Step 2 – y-intercept is zero
Step 3 – Solving for the value of Q
TR 100Q − 2Q2
=
100Q − 2Q2 =
0
Q (100 − 2Q ) =
0
Q=0

100 − 2Q =
0
−2Q =
−100

Q = 50

Maximum
profit
1250

TR 100Q − 2Q2
=

25 50

Maximum Profit
CHAPTER 12 Revenue, cost and profit 243

TR 100Q − 2Q2
=
Q = 25
=TR 100 ( 25) − 2 ( 252 )
TR 2500 − 1250
=
TR = 1250

Total cost is any item that can be traced in the production or the facilitation on earning
revenue. Total cost involves two variables that are the fixed cost and the variable cost.
Total cost is given by adding the fixed cost and variable cost per unit

= FC + VC ( Q )
TC

The total cost function is a linear function such that when draw on a graph it appear a straight
line.

The graph of the total cost is shown in the following page.

Variable
cost

Total Fixed
cost cost

Quantity

When we divide the total cost by the quantity then we get the average cost

TC
AC =
Q
CHAPTER 12 Revenue, cost and profit 244

In details we can consider the formula of total cost

= FC + VC ( Q )
TC

FC + VC ( Q )
AC =
Q

FC VC ( Q )
AC
= +
Q Q

FC
AC
= + VC
Q

Profit is given by the total revenue minus the total cost

P TR − TC
=

Example 12.2
Given that the fixed costs are KES 1000 and the variable cost are KES per unit express total cost
and average cost as functions of Q

Solution

TC = VC ( Q )
= FC
= 1000 + 4 ( Q )
= 1000 + 4Q
CHAPTER 12 Revenue, cost and profit 245

Example 12.3
If the fixed costs are KES 4 and variable cost per unit is KES 1 and the demand function is
P 10 − 2Q
=
Required
a) Obtain the profit function
b) Find the break even point
c) Find the Maximum profit
d) Sketch the graph

Solution
= FC + VC ( Q )
TC
= 4+Q
TR = PQ
= Q (10 − 2Q )
= 10Q − 2Q2
π= TR − TC
= 10Q − 2Q2 − ( 4 + Q )
= 10Q − 2Q2 − 4 − Q
−2Q2 + 9Q − 4
=

ii).At Break even point (BEP) profit is equal to zero or the Total Revenue is equal to Total cost

P = 0 or TC = TR

−2Q2 + 9Q − 4
P=
−2Q2 + 9Q − 4 =0

We can solve it as a quadratic equation

−2Q2 + 9Q − 4 = 0
Product = −2 × −4 = 8
Sum = 9
Numbers = 1,8
CHAPTER 12 Revenue, cost and profit 246

−2Q2 + Q + 8Q − 4 =0
−Q ( 2Q − 1) + 4 ( 2Q − 1) =
0
( −Q + 4 )(2Q − 1) =0
Q = 4 or 1

Maximum
profit
6.125

π=
−2Q2 + 9Q − 4

1 4

Break even
points

Maximum profit can be calculated by substituting Q to the profit function

π =−2Q2 + 9Q − 4 =0
Q=4
π =−2 ( 42 ) + 9 ( 4 ) − 4 =0
π =−2 ( 42 ) + 9 ( 4 ) − 4 =0
−10.125 =20.25 − 4
π = 6.125
CHAPTER 12 Revenue, cost and profit 247

Practice Problem

1. A manufacturing plant has KES 8,400 in fixed costs and it uses KES1.25 to produce a single
unit of its products. The selling price has been set at KES 2.0 per unit by the laws of demand and
supply.

Required:
a) How many units must be produced in order for the operation to break even?

b) Draw a graph of both Cost and Revenue functions marking clearly the break even
point, the profit and loss regions

c) How many units must be produced to make a profit of KES 6,000?

d) How many units must be produced to make a profit of 25% on sales?

2. A photographer is willing to supply 280x – 500 cheap passport photographs per year at a price
of x dollars each. The public will buy 1000 – 120x such photographs at x dollars.

Required:
a) What is the equilibrium price and equilibrium demand?

b) Plot the two functions on a graph and show the equilibrium point?

3. The yearly demand for a certain style of gloves is x = 20,000 – 2000p, where p denotes the
price per pair (in dollars). The total cost of producing x pairs of gloves is

C = 30,000 + 1.50x dollars.

Required:
a) Formulate the Revenue and Cost functions in terms of price, p

b) Plot on the same graph the two functions and mark clearly the loss and profit regions.
Let p be on the x axis

4. Determine the price range that will earn the manufacturer profits i.e. establish break even
point by solving quadratic equations for this problem.
CHAPTER 12 Revenue, cost and profit 248

5. 20 units of a product cost KES. 200,000 and 50 units cost KES 272,000. Determine

(i) the unit variable cost

(ii) (ii) the linear cost equation

(iii) (iii) fixed cost (iv) cost of 80 units

6. For each of the following determine the linear cost equation, revenue and profit
equation and the breakeven point

.
Unit variable cost = KES 3200, fixed cost = KES 72,000 and unit price = KES 4,900

Unit variable cost = KES 7,200, fixed cost = KES 160,000 and unit price = KES 8,000

Unit variable cost = KES 8,000, fixed cost = KES 640,000 and unit price = KES 11,200

Unit variable cost = KES 4,000, fixed cost = KES 720,000 and unit price = KES 6,400

7. For each of the following P denotes the unit price and x the number of units produced
and sold of a product

i) Demand function P =− x + 1200


Cost function
= C 200 X + 160000

ii) Demand function P =


−2 x + 2400
Cost function
= C 400 x + 180000

iii) Demand function P =


−2 x + 2700
Cost function C =
−300 x =
540000

Required
a) Determine the revenue function for each

b) How many units of the product should sold in order to break even

c) Sketch these graphs using excel hence estimate the number of units produced for
the firm to maximize profit
CHAPTER 12 Revenue, cost and profit 249

8. A chocolate manufacturer has fixed cost of KES 26,000 per week. The variable cost to
produce one chocolate is KES 25. If the chocolate is then sold for KES 38 per bar,
determine how many bars must be produce and sold each week to;

a) Break even

b) Earn a profit of KES 50,000 per week


CHAPTER 13 DECISION TREES

Learning Objectives
At the end of this section the students should be able to:

 Identify and draw a decision tree


 Compute the expected value
 Use the roll back method to make the best decision

A decision tree is a branching diagram which is similar to a probability tree. It is used to


represent problems in which a series of decision has to be made under conditions of
uncertainty. Any one of the decision may be dependent on the outcome of preceding decisions
or the outcomes of a trial.
Decision tree is constructed from left to right.
The branches represent the possible alternative decisions which could be made and the various
possible outcomes which might arise.
The square or rectangle nodes from the decision branches are drawn represent the points the
decision maker selects his decision.
The round or circle nodes represent the points at which the outcome of the decision arises.
CHAPTER 13 Decision trees 251

Example 13.1

The District Agricultural Officer of a certain District has advised farmers to apply pesticides
costing KES 2000 per acre to better their crop on the weather; this pesticide might have no
effect on their crops per acre, 10% increase in yield, or a 15% increase in yield. The probability
of each effect is shown below.

Increased Yield
Weather 0% 10% 0.5%
Dry 0.1 0.4 0.5
Normal 0.2 0.6 0.2
Wet 0.3 0.5 0.2

Based on the experience and without pesticide application, the farmer can expect the following
probabilities, yields and crops prices for the different weather conditions.

Increased Yield
Weather Probability 50kg bags per Selling price per
acre bag -KES
Dry 0.3 48 700
Normal 0.6 55 620
Wet 0.1 60 580

Use a decision tree to decide whether or not the farmers should supply the pesticide.
CHAPTER 13 Decision trees 252

Solution
The farmer has only one decision to make whether to use the pesticide or not. Thus the decision
tree for this problem has only one choice

0% 0.1 31600

10% 0.4 34960


C

15% 0.5 36640


Dry 0.3
0% 0.2 32100

Normal 0.6 10% 0.6 35510


B D

15% 0.2 37215


Pesticide
Wet 0.1
0% 0.3 32800

A E 10% 0.5 36280

15% 0.2
38020
Non pesticide 0% 0.3 33600

F 10% 0.6 34100

15% 0.2 34800

If the pesticide is not applied we arrive at choice node F with the following payoffs

Dry ⇒ 48 bags × 700 =Kshs 33600


Normal ⇒ 55 bags × 620 = Kshs 34,100
Wet ⇒ 60 bags × 580 = Kshs 34,800
CHAPTER 13 Decision trees 253

When the pesticide is used, we must perform a similar calculation in which the yield per acre is
applied by the appropriate factor to account for the increase in yield. For example if there is dry
weather and the pesticide increase yield by 10% then the yield per acre is

(1.10 ) 48 = 52.8 bags per acre

Finally to obtain the payoff in dry weather when the pesticides increase yield by 10% we must
subtract the pesticides cost per acre KES 2000 from the return per acre.

Kshs 36960-2000 = Kshs 34,960

We now assign numbers to the nodes using Bayes theorem and working right to left of the tree.
To chance nodes C, D, E and F we assign expected values of the values are 35654, 35,167,
35,584 and 34,020 respectively.
To chance node B we assign expected value of the numbers assigned to nodes, C, D and E

0.3 ( 35464 ) + 0.6 ( 35169 ) + 0.1 ( 35584 ) =


35299

Finally we assign to choice node A the maximum of the numbers assigned to nodes B and F
which is KES 35,299. Thus the farmer maximum expected gain is 35299 per acre. He should
apply the pesticides.

Example 13.2

ANU has hired a new campus Y. They can either test the market or abandon project. The details
are set out below.
Test cost KES 5 million; likely outcome are:
Favorable 0.7
Unfavorable 0.3
If favorable they could either abandon or continue with the project. The demand is anticipated
to be:
Low P = 0.25 loss KES 10 million
Medium P = 0.6 profit KES 15 million
High P = 0.15 profit KES 45 million

If the test market indicates failure the project would be abandoned. Abandonment at any stage
results in loss of KES 3 million from rent that would have been paid.

Draw a decision tree diagram with probabilities cost and revenues incorporated appropriately
Evaluate the decision tree and recommend the action company should take
CHAPTER 13 Decision trees 254

Solution

D2 3m

Low -10m
Failure
0.25
A
Medium
Favorable Continue B 15m
Test 0.5
D1 D3
High 45m
0.15

3m
Abandon

3m
Abandon

Evaluation

Expected value at node B

( 0.25 × −10
= ) ( 0.6 × 15) + ( 0.15 × 45
= ) 13.25 million

Comparison at D3 is 3 million versus 13.25 million so proceed with EV of 13.25 million

Expected value at node A

( 0.3 × 3) + ( 0.7 × 132500 ) =


10.175 million

Comparison at D1 is

10.175 − 5 =5.175 compared by 3 million


CHAPTER 13 Decision trees 255

Test market with EV of 5.175 million

ANU should test the market at cost of KES 5 million and go ahead with the favorable program if
these are favorable indicator.

Practice Problem

1. The management of AZT Company Ltd. are planning for the future needs of the
company. They are faced with three decisions either to expand the present factory, or
try to cope with demand in the present location without expanding or move to a new
location with larger facilities. If they choose to expand the factory, it will cost them KES
800,000 and moving to the larger location will cost KES 1.6 million. The following
information is available to help them make the decision.
Demand Probability
High 0.5
Medium 0.3
Low 0.2

Conditional payoffs (KES Millions)


State of demand
Location high medium low
Same 2.4 1.6 0.8
Expand 4.8 2.8 1.6
New 5.6 3.2 0.8

Use a decision tree to determine the best decision the management could arrive at.

2. Analysis of a questionnaire completed by holidaymakers showed that 0.75 classified


their holiday as good at a beach hotel. The probability of hot around the beach is 0.6.
The probability of regarding a holiday as good given that the weather is hot is 0.9. If a
holidaymaker chosen at random considers the holiday to good, what is the probability
the weather is hot?

A firm has developed a new product. The firm can either test the market or abandon the
project. Testing the market would cost KES. 6 million and the test would either be
CHAPTER 13 Decision trees 256

success or failure. The probability of a success outcome is 0.7 and failure is 0.3. If the test
shows failure the company will abandon the project. Abandoning the project at any
stage would cause the company sell the patent at KES. 3.6 million. If test is successful,
the company can either abandon the project or market the product. If they market the
product with successful test results the anticipated profit is;

Demand Profit
Low – KES. 12 million
Medium KES. 18 million
High KES. 54 million

Draw a decision tree for the above and use the expected monetary value criterion to
advise the management on the best decision.

3. The Beta Company is proposing to introduce to the market a remote controlled toy car.
There are three different possible models A, B and C they models they manufacture.
However, they have the capacity of manufacturing only one of them. The probable
acceptance of any of the three models is given below.

profits (KES 00,000)


Model acceptance Probability Model A Model B Model C
Excellent 0.4 120 100 60
Moderate 0.5 80 60 50
Poor 0.1 -30 -20 10

Determine the best decision if:


a) the maximax, maximin or minimax regret criteria are used.
b) The expected monetary value criterion is used.

4. A product manager of newly established company wishes to determine whether or not


to market a new toothpaste. He is also considering a consumer-testing programme that
will cost KES. 75,000 before marketing the product. If they market without the
consumer-testing programme, the probability of success is 0.5. The consumer –testing
programme can produce a favourable result with probability 0.40. Given a favourable
test result, the probability of product success is 0.80 but is only 0.30 if the test result is
unfavourable. If the product is marketed after the testing programme an additional cost
of KES. 50,000 for distribution will be incurred. If the product is marketed without the
testing programme, KES. 100,000 will be spent on distribution. The projected profits are
KES 1,500,000 and an unsuccessful launch would result in a loss of KES 800,000. No
profits or losses if the product is not marketed.
CHAPTER 13 Decision trees 257

Use a decision tree to determine the manager’s best course of action.


CHAPTER 14 MATHEMATICS OF
FINANCE

Learning Objectives
At the end of this section the students should be able to:

 Use simple method to determine the interest


 Use compound method to find interest
 Use the payback period to determine the worthwhile of a project
 Use the Net present value method to determine the worth while of the
project

Simple Interest

Given that A want to invest his KES10, 000 in bank, for five years and the bank offers a simple
interest at rate of 10%. Calculate the interest that the money earns after the end of the period.

Interest = Principal × interest rate × time


I = P r t
CHAPTER 14 Mathematics for finance 259

Solution
P× r× t
I=
100

r = 10%

t = 5years

P = 10,000

10,000 × 10 × 5
I= = 5000
100

To calculate the value of future value

FV= P + Pr t
= P (1 + rt )

To calculate the future value for the above example


= P (1 + rt )
FV
 10 
= 10,000  1 + × 5
 100 
= 10,000 (1.5)
= 15000

Compound Interest

For example to invest KES 10, 000 in three years at compounded interest of 10% p.a

Solution

Compound interest at year 1, 2 and 3


CHAPTER 14 Mathematics for finance 260

 10 
Year 1 ⇒
= FV 10,000  1 + (1,1) 11000
 ⇒ 10,000=
 100 

 10 
Year 2 ⇒
= FV 11,000  1 + (1,1) 12100
 ⇒ 11,000 =
 100 

 10 
Year 3 ⇒
= FV 12100  1 +  ⇒ 12,100 (=
1,1) 13310
 100 

To avoid doing that given that one may compound for several years we use the following
formula

FV PV (1 + r )
n
=

FV = Future value

PV = Present value

r = interest rate in %

n = time periods of investment

3
 10 
FV = 10,000  1 + 13310
 =
 100 

Example 14.1

What is the value of sum of KES 50,000 invested at rate of 5% compounded interest for interest
for 10 years.
CHAPTER 14 Mathematics for finance 261

Solution

FV PV (1 + r )
n
=

10
 5 
50,000  1 + 
 100 

50,000 (1.05)
10

= 81,445

Investments Analysis

An efficient allocation of capital is one of the most important functions of financial


management. Investment analysis is needed in order to make optimal use of available
resources. To evaluate an investments proposal the study of the following component is
necessary.
 The initial cash outflow
 The returns (net cash inflow)
 Economic life of the assets
 Minimum return desired such investment

The most widely used method of evaluating an investment appraisal can be grouped into two
categories
 Traditional method
 Payback period method
 Average rate of return
 Discounted cash flow methods
 Net present value
 Internal rate of return method (IRR)
 Discounted payback period
CHAPTER 14 Mathematics for finance 262

Payback Period Method

Payback period is the number of years required to recover the cash invested in a project if the
annual cash inflows are the same. The payback period can be computed by dividing the cash
invested by annual cash inflow

Example 14.2
A project requires KES 60,000 and yield inflows of KES 12000 p.a for 7 years. What is the
payback period?

Solution

Cash outlay
Pay back period =
Average net cash inflow per year

60,000
= = 5 years
12,000

Example 14.3

Suppose the cash inflows are unequal


CHAPTER 14 Mathematics for finance 263

Y1 10,000
Y2 15,000
Y3 20,000
Y4 15,000
Y5 12,000
Y6 15,000
Y7 20,000

Calculate the payback period for the project if the initial out flow is KES 60,000 , KES, 40,000

Solution
Add the year 1, 2, 3 and 4 until you get 60,000

10,000 + 15,000 + 20,000 + 15,000 =


60,000

The payback period is 4 years

When the initial cash flow is 40, 000 and year until you get 40,000

10,000 + 15,000 =
25,000

Up to second year we can recover 25, 000 the remainder which is gotten from the difference of
the initial out flow

40.000 − 25,000 = 15,000


Divide the difference with the next cash flow in this case it is 20,000

15,000 3
= 0.75 equivalent to
20,000 4

When multiplied by 12 months we get 9 months

Hence it will take 2 years and 9 months


CHAPTER 14 Mathematics for finance 264

Net Present Value

This method takes into account the time value of money. It correctly shows that cash flows
arising at different time periods differ in value and are comparable only when they are
equivalent present and are calculated as .

 1 
PV = FV  n 
1+r 

Example 14.4

What is the present value of 10,000 in 5ears at a rate of interest of 10%

Solution

 1 
PV = FV  n 
1+r 

10,000
= = 6209.2
(1 + 0.1)
5

Example 14.5

A businessman has two projects, A and B but can only invest in one. He has approached you for
advice and given you the cash flows for each projects as below.
CHAPTER 14 Mathematics for finance 265

Cash flows (Millions) A B


Initial cost 212.5 250
Year 1 expenses 25 0
Year 1 income 25 0
Year 2 expenses 25 0
Year 2 income 275 302.5

a) Calculate the Net present value (NPV) of each project using a discount rate of 7% p.a.
b) Compute the internal rate of return for each project
c) Which of the two projects would you recommend

Solution

275 − 25 25 − 25
NPVA = + = 212.5
1.072 1.07

218 − 4 − 212.5 =5.9 million

302.5 − 25
NPV
= B 250 212.5
−=
1.07

264.5 − 250 =
14.2 million

275 − 25 25 − 25
NPV=
A + − 212.5
= 0
(1 + A ) 1 + A
2

250
= = 212.5
(1 + A )
2

(1 + A )
2
1.1765
=

1+ A =
1.085

IRRA = 8.5%
CHAPTER 14 Mathematics for finance 266

IRRB = iA
302.5
NPVB
= = 250
(1 + A )
2

(1 + A )
2
1.21
=

1+ A =
1.1

IRRB = 10%

Project B is the best suitable project than project A. Since it has a higher NPV and IRR

Example 14.6

A new machine is expected to last for six years and to produce annual (year end) saving of KES
10,000. What is the maximum sum worth paying for machine now assuming compound interest
at 10% per annum?

Solution

To calculate the NPV of annuity (equal cash flows) check from the table of annuity factor for 6
years
This is how we check from the table which is always given

Present Value of annuity of 1 Per Period for n


Year 1
1−
(1 + r )
1
n
1
PVIFrt ∑
= =
i =1 ( 1 + r )
1
r

10%

Year 6 PFVA = 4.355


CHAPTER 14 Mathematics for finance 267

From the real annuity table this how you trace the value

Time Value of Money


Present Value of Annuity Factors

n/r 1% 2% 3% 4% 5% 6% 7% 8% 9% 10%
1 .9901 .9804 .9709 .9615 .9524 .9434 .9346 .9259 .9174 .9091
2 1.9704 1.9416 1.9135 1.8861 1.8594 1.8334 1.8080 1.7833 1.7591 1.7355
3 2.9410 2.8839 2.8286 2.7751 2.7232 2.6730 2.6243 2.5771 2.5313 2.4869
4 3.9020 3.8077 3.7171 3.6299 3.5460 3.4651 3.3872 3.3121 3.2397 3.1699
5 4.8534 4.7135 4.5797 4.4518 4.3295 4.2124 4.1002 3.9927 3.8897 3.7908
6 5.7955 5.6014 5.4172 5.2421 5.0757 4.9173 4.7665 4.6229 4.4859 4.3553

PFVA = 4.3553

Maximum sum = 10,000 × 4.355 = 43,550

Example 14.7

A company is considering the launch of a product, for which an investment in equipment of KES
150,000 would be required. The project life would be limited to five years by the expected life of
the product. It is expected that the equipment could be sold for KES10,000 in year 6. Market has
indicated being low. Cash inflows are forecast as follows;

High Demand Low Demand


Year Kshs.'000' Kshs.'000'
1 60 50
2 62 50
3 65 50
4 70 50
5 70 50

If the new product is now, an existing product, which could otherwise be retained for a further
five years, would be discounted immediately. If retained cash inflows of KES 12,000 p.a. would
CHAPTER 14 Mathematics for finance 268

be expressed for the existing product. The company’s discount rate is 15% p.a. (Assume that all
cash flows occur at year ends).
a) Calculate the expected Net Present value of the new product
b) Advise the company on whether to launch the new product, or to retain the existing
product

Solution

Capital investment

Year 0 (150,000 ) × 1.000 =


150,000
Year 6 10,000 × 0.432 =4320
145,680

Cash inflows – high demand


Check from the Present value table

Present Value of 1 Received at the end of n Periods


Year 1
(1 + r )
−n
PVIFrt= =
(1 + r )
n

15%

Year 5 Use all the factors


up to year 5

Use all the factors


Present Value Factor for a Single Future Amount up to year 5
(Interest rate = r, Number of periods = n)
n/r 6% 7% 8% 9% 10% 11% 12% 13% 14% 15%
1 .9434 .9346 .9259 .9174 .9091 .9009 .8929 .8850 .8772 .8696
2 .8900 .8734 .8573 .8417 .8264 .8116 .7972 .7831 .7695 .7561
3 .8396 .8163 .7938 .7722 .7513 .7312 .7118 .6931 .6750 .6575
4 .7921 .7629 .7350 .7084 .6830 .6587 .6355 .6133 .5921 .5718
5 .7473 .7130 .6806 .6499 .6209 .5935 .5674 .5428 .5194 .4972
CHAPTER 14 Mathematics for finance 269

Year 1 60,000 × 0.870 = 52,200


Year 2 62,000 × 0.756 = 46,872
Year 3 65,000 × 0.658 = 42,770
Year 4 70,000 × 0.572 = 40,040
Year 5 70,000 × 0.497 = 34,790
216,672

Cash inflows – low demand

Present Value of annuity of 1 Per Period for n


Year 1
1−
(1 + r )
1
n
1
PVIFrt ∑
= =
i =1 ( 1 + r )
1
r

15%

Year 5 PFVA = 3.353

Time Value of Money


Present Value of Annuity Factors

n/r 6% 7% 8% 9% 10% 11% 12% 13% 14% 15%


1 .9434 .9346 .9259 .9174 .9091 .9009 .8929 .8850 .8772 .8696
2 1.8334 1.8080 1.7833 1.7591 1.7355 1.7125 1.6901 1.6681 1.6467 1.6257
3 2.6730 2.6243 2.5771 2.5313 2.4869 2.4437 2.4018 2.3612 2.3216 2.2832
4 3.4651 3.3872 3.3121 3.2397 3.1699 3.1024 3.0373 2.9745 2.9137 2.8550
5 4.2124 4.1002 3.9927 3.8897 3.7908 3.6959 3.6048 3.5172 3.4331 3.3522

PFVA = 3.353

Year 1 - 5 50,000 × 3.353 = 167,650


CHAPTER 14 Mathematics for finance 270

Net present value

High demand 216,672 - 145,68 = 70,992

Low demand 167,650 - 145,680 = 26,970

Expected Net Present value

( 70,992 × 0.7 ) + (21,970 × 0.3) =


56,285

PV of existing product = 12,000 × 3.353 = 40,236 based on expected value there is potential
value of
56,285 − 40,236 = 16,049 From accepting the new product

Practice Problem

1. An investor projects that she will require KES. 3.0 million in 4 years’ time from now to
repay a debt. She decides to deposit KES P now into an account that pays 10% interest
p.a. compounded annually. She withdraws the money plus the accrued interest after 4
years. Determine how much she deposits into the account so that the accumulated
amount to equal the debt.

a) Determine the effective rate of interest that corresponds to a nominal rate of


18% p.a. compounded monthly.

b) Find the nominal interest rat p.a. compounded quarterly that corresponds to an
effective rate 10.38% p.a.

2. Find the future value of KES 200,000 in three year’s time compounded monthly at a
rate of 18% p.a.

3. A lady decides to save KES 10,000 per year and deposits the money each year into an
account that pays interest at a rate of 10% p.a. Calculate the accumulated value after 10
years.
CHAPTER 14 Mathematics for finance 271

4. An investment requires an initial capital outlay of KES 12 million and will produce a
return of KES 25 million at the end of 5 years. Use the internal rate of return and
determine if the investment is worthwhile given that the cost of capital is 12% p.a.

5. Find the simple interest on KES 5000 invest for 3 years 2 months at 10% p.a.
Calculate the accrued amount for KES 4,500 borrowed for 90 days at 15% p.a. simple
interest.

6. A promissory note with a maturity value of KES120, 000 and a simple interest rate of
12% p.a. is sold 3 months prior to its due date. What is the present value on the day it is
sold?

7. Determine the simple discount on a promissory note of maturity KES. 300,000 due in
eight months at a discount rate of 15% p.a. What is the discounted value of the note?
What is the equivalent simple interest rate?

8. A bank’s simple discount rate is 18% p.a. If you sign a promissory note to pay KES
400,000 in six months, how much would you receive from the bank now? What is the
equivalent simple interest rate?

9. Jack borrows a sum of money from a bank and has to pay back KES.100, 000 in 9 months
time from now. The agreed interest rate is 12%p.a. How much does he receive now?
How much does he owe the bank 4 months from now? If he reschedules to repay the
debt at the end of the year, how much will he have to pay?

10. KES 20,000 is invested for 3 years at an interest rate of 14% p.a. compounded semi-
annually. Determine the accrued amount.

11. What is the present value if the compounded amount is KES 1,000,000 at 18% p.a. for 5
years and interest is calculated on quarterly basis?

12. An obligation of KES 50,000 fall due in one and a half years time. What amount will be
needed to the debt if it paid in six months?

13. Calculate the amount accrued on KES 300,000 invested at 12% p.a. for 3 years
compounded daily.
CHAPTER 14 Mathematics for finance 272

14. What is the present value of a maturity value of KES. 500,000 at 16% p.a. compounded
quarterly and invested for two and half years?

15. Suppose KES. 120,000 is invested now and gives a return of KES 136,000 in one year’s
time, calculate the internal rate of return.

16. KES 1,000,000 is invested now and gives a return of KES 800,000 in the first year and
further KES 560,000 in the second year. Determine the internal rate of return.

17. A project requires an initial investment of KES 12,000,000. It has a guaranteed return of
KES. 8 million at the end of year 1 and a return of KES. 2 million each at the end of years
2, 3, 4. Estimate the IRR. Would you recommend that someone invest in the project if
the prevailing market rate is 8% compounded annually?

18. A firm decides to invest in new machinery, which is expected to produce additional
revenue of KES. 8million at the end of each for ten years. At the end of this period the
firm plans to sell the equipment for KES. 5 million. What is the maximum amount the
firm should pay now if it not to suffer a net loss in the investment at discount rate of
6%?

19. A principal of KEs. 560,000 is invested at 9% p.a. compounded monthly for 5 years.
Determine the future value and interest after 5 years.

20. Mr. X opens a savings account for his daughter. Each month he deposits KES 4,000 into
this account. Calculate the future value of the amount after 3 years if interest is
computed at 12% p.a. compounded monthly.

21. Find the present value of an annuity with a yearly payment of KES 100,000 at an interest
rate of 9% p.a. after 10 years.

22. You have been given the opportunity to invest in any one of three projects A, B, or C.
Projects A, B, and C require an initial cash outlay of sh. 1.6 million, sh. 2.4 million and sh.
8 million respectively. The guaranteed return after 3 years is sh. 2.0 million, sh. 2.96
million and sh. 9.36 million for A, B, and C respectively. If the market rate is 5% p.a.
compounded annually, explain which one of these projects you would invest in.

23. The county of Narok has decided to engage your services as a financial consultant. The
Governor require to know which proposed investment would represent the best use if
CHAPTER 14 Mathematics for finance 273

the allocation from the National Government of KES 4,900,000 for three strategic passed
projects by the county assembly.

24. The dilemma arises because each of the three projects have an economic life of three
years and can yield a total cash inflow of KES 6,000,000. However estimates show that
the pattern of inflows, for each project differs as below.

Year Annual Cash inflows (Kshs '000')


Project 1 Project 2 Project 3
2010 1,000 2,000 3,000
2011 2,000 2,000 2,000
2012 3,000 2,000 1,000

The county has checked on a hurdle rate of 10% p.a. as the minimum discounting rate for
its projects and it is anticipating that the cash inflows will occur evenly over any given
year. The PV interest factors at 10% are as follows:

Using payback period and NPV techniques, advice the Governor on which project is most
viable.
CHAPTER 15 LINEAR PROGRAMMING

Learning Objectives
At the end of this section the students should be able to:

 Identify the region defined by a linear programing


 Sketch the feasible region defined by simultaneous linear inequalities
 Formulate a LP problem
 Solve a LP problem

Introduction

There are many activities in organization which involve the allocation of resources. These
resources include labor, raw materials, machinery and money. The allocation of these resources
is sometimes called programming. Linear programming is suitable method for modeling an
allocation problem if the objective and constraints on the resources can be expressed as linear
relationships of the variables
Many practical problems involve maximizing and minimizing a function to constraints. For
example we may want to maximize a profit function subject to certain limitation.
There are many activities in organizations which involves the collection of resources machinery,
money, man, material.
The allocation of these resources is sometimes called programming. Problem arises because the
resources are usually limited or scarce supply. Linear programming is a mathematical
techniques concerned with allocation of scarce resources.
The objective is to determine the most efficient method of allocating these resources to
valuable so that some measures of performance is optimized.
Linear programming can be used to solve problem that satisfies the following.
Problem must be capable of being stated in numerical terms.
All factors involved in the problem must have linear relationships
The problem must per5mit choice of choices between alternative causes of action
There must be one or more restrictions on the factors involved for example a fertilizer must
contain a minimum of 155 potassium, 15% phosphate and 30% nitrogen.
CHAPTER 15 Linear programming 275

Steps involved in linear programming


Formulate a linear programme mathematically
This involves identification of the objectives and constraints. These are then written down as
linear relationship in terms of variables
Identify the feasible combination of valuables
This is a combination which optimizes the objective. If only to variable s involved a graphical
solution is possible if however we have more than to variables than simplex method is used
Once the optimum solution is identified it must be evaluated
This will include a sensitivity analysis

Problem Formulation programming


The basic procedure is the same for all linear programs
Steps involved are.
1. Identify the valuable in the problem for which the value can be chosen with the limits of
the constraints
2. Identify the objectives
3. Write down the objective in terms of variables.
4. Identify the constraints.
5. Write down the constraints in the terms of variables.

Example 15.1
A launcher make two blend of animals feeds that is daily meal and Maize jam by mixing three
inputs coded x, y and z and the weekly cost and availability of the input are as below.

Inputs Amount in (Kg)


x 25,000
y 50,000
z 75,000

The blend requirement of each type of input as below.

% of each inputs
x y z
Daily meal 25 55 20
Maize jam 40 20 40
There is a requirement that the launcher must produce 50,000 kg of each blend.
Given that the cost of making each unit of dairy meal and maize jam is KES 100, KES 120
CHAPTER 15 Linear programming 276

respectively.
Formulate the above problem as linear programming problem if the objective function of the
launcher is to make blend of the least possible cost.

Solution

Let the dairy meal be p


Let the maize jam be q

Constrains
The launcher must produce at least 50,000 of p and q inputs
p ≥ 50000
q ≥ 50000
The value of x, y and z is given as
x = 25000
y = 50000
z = 75000
We can form the following equation
0.25 p + 0.4q = 25000
0.55 p + 0.2q = 50000
0.2 p + 0.4q = 75000

Example 15.2

Linear program
Minimize cost C 100 p + 120q
=

Subject to:
CHAPTER 15 Linear programming 277

Example 15.3

A family bakery produces two types of cakes Black forest and banana bread. The bakery can sell
all that it bakes but production is limited by the supply of major ingredients and by the amount
of oven capacity available.
The production of one cake black forest requires 0.04 hours of oven time while as the
production of banana bread requires 0.08 hours of oven time.
0.02 kilogram of special ingredients is required for one black forest cake and 0.08 kg is required
for one banana bread cake.
Each day the bakery has 21 oven hours available and 160 kilograms of the special ingredients
The contribution is KES 200 for the black forest cake and KES 300 for the banana bread cake.
How much of each should be baked if the bakery wishes to maximize the daily production.

Solution
Let the black forest cake be x
Let the banana bread cake be y

Objectives=π 200 x + 300 y

Constraints
0.04 x + 0.08 y ≤ 24
0.02 x + 0.08 y ≤ 160

Linear program

Maximizing profit
Subject to:
CHAPTER 15 Linear programming 278

Solving linear program using graphical


Example 15.4

A firm produces two types cooking fat supper and premier. The raw material to make super cost
KES 20 per kilogram and to make premier cost KES 30 per kilogram.
The raw materials pass through three machines processors. The time taken in each processor is
as given below in minutes.

Fat Processor 1 Processor 2 Processor 3


Supper 4 8 5
Premier 6 10 8

The three processor are available for a maximum of twelve hours a day. The contribution is KES
45, 50 for supper and premier respectively.
Required
Define the decision valuables and the constraints for the above problem.
Formulate the problem as a linear programming model given the objective of the firm is to
maximize profit.
Solve the above problem graphically and hence indicate the optimal solution and the optimal
objective value.

Solution

First define the valuables that is supper and premier


Let the supper be x
Let the premier be y

Objective function
=π 45 x + 50 y
The time constraints are,
Machine processor 1
Machine processor 2
Machine processor 3
The equation of constraints are given as below
CHAPTER 15 Linear programming 279

Linear program
Objective:= π 45 x + 50 y
Subject to:

To draw the linear program on graph we need to get specific points which we are going to use
We calculate the points as follows

4x + 6 y =
720 with the value of x = 0 then we can get the value of y as follows

4x + 6 y =720
x=0
6 y = 720
y = 120

We obtain the point as (0, 90)


Again we can equate y = 0

4x + 6 y =
720
4 x = 720
x = 180

We obtain the second point of that line to be (180, 0)


So when plotting the line we use the two points. We can do likewise to the other two equation
to obtain the following
CHAPTER 15 Linear programming 280

Identify the points where the lines are intersecting,

A(0, 72)
B(90, 0)
C (0, 0)

Use the points to substitute the objective function to see what will give the highest value since
our objective is to maximize profit and we need the highest profit values

A =45(0) + 50(72) =3600


B = 45(90) + 50(0) = 4050
C =0

To get maximum profit we need to produce

90 kg of supper fat
0kg of premier fat
CHAPTER 15 Linear programming 281

Example 15.5

Using the linear program in example 1 we can solve the problem graphically as follows.
Minimize cost = C 100 p + 120q

Subject to:

Solution

We obtain the points for each equation as follows

Solving the linear program graphically it will appear like the figure below.
CHAPTER 15 Linear programming 282

Using point A and B we can be able to calculate the minimum cost


The objective function is to minimize cost

C 100 p + 120q
=

A(50000, 162500) and B(27500, 50000)

Thus

Essential functions in linear

Objective function- it is the maximizing profit and revenue or minimizing function of costs
Constraints – Are the resources that are limited but necessary to maximize or minimize an
objective function.

Formulating a linear programming

The basic procedure is the same for formulation of all linear programmes:
CHAPTER 15 Linear programming 283

Step 1
Identify the variables in the problem for which the values can be chosen, within the limits of the
constraints

Step 2
Identify the objective and the constraints on the allocation

Step 3
Write down the objective in terms of the variables

Step 4
Write down the constraints in terms in terms of the variables

Application of linear program


Example 15.6

University administration is planning a team building trip for its staff. Two types of buses are
available. Type A can carry people and 1000 kg of baggage and costs KES 20,000. Type B carry 50
people and 750 kg of baggage and costs KES 24,000. A total of 800 people are going on trip, and
they have 18,000 of baggage.
Form a linear program for above information
How many of each type bus should be chartered to minimize the cost of the trip.

Solution
To solve the above linear programming we say that;
Let the number of type A of bus be x
Let the number of type B of bus be y

Write down the objective function

Minimize 20,000 x + 24,000y

40 x + 50y ≥ 800
Subject to
1000 x + 750y ≥ 18000

Remember we don’t need the negative part thus we always introduce the non-negativity
function
CHAPTER 15 Linear programming 284

x≥0 and y≥0

To plot the points we need to get two points in every line that is find the value of y when x is
zero and x when y is zero and use the points to plot the graph
40 x + 50y ≥ 800

x y
0 16
40 0

For the second line then get the value of x when y is equal to zero and similarly get the value of
y when x is equated to zero

1000 x + 750y ≥ 18000

x y
0 24
18 0

24 B

Feasible
15 A Region

C
4 18 40
40 x + 50y ≥ 800
1000 x + 750y ≥ 18000
CHAPTER 15 Linear programming 285

The boundary and interior of ABC is called the feasible region, feasible production set,
production possibility set or opportunity set
The solution lies on the vertexes where the lines are cut each other but must within the feasible
region

Let’s evaluate points A, B and C

20,000 x + 24,000y
20,000 (15) + 24,000 ( 4 ) =
396,000
20,000 ( 0 ) + 24,000 ( 24 ) =
800,000
20,000 ( 40 ) + 24,000 ( 0 ) =
960,000

We should take the least because we are trying to minimize cost that is hire 15 buses of type A
and 4 buses of type B

Example 15.7

A gem dealer buys rough – cut amethysts and beryls of uniform size, which she finishes for sale
retail jewelers. Each amethyst requires one hour on a grinder, one hour on a sander and two
hours on a polisher and yields a net profit of KES 7500. Each beryl requires two hours on a
grinder, five hours on a sander and two hours on a polisher and yields a profit of KES 10,000.
The grinder, sander and polisher are available for a maximum of 40 hours a week.
Formulate the above as a linear programming problem
By use of suitable scale solve the linear programming problem by graphical method and
establish the optimal mix
The gem dealer wants to introduce a new gem and wants to know which machine has idle
capacity. Evaluate the resource utilization

Solution

Let x be the number of amethysts sold


Let y be the number of beryl

The tabulation of the resource utilization


CHAPTER 15 Linear programming 286

Products Availability of machines


Amethysts Beryls
Grinder 1 2 40 Hours
Sander 1` 5 40 hours
Polisher 2 2 40 Hours
Profits per unit Shs 7500 shs10,000

The objective is to maximize profits subject to hours available

Maximize = 7500x+10000y
Subject to:
Grinder: x +2y ≤ 40....................................................... ( i )
Sander: x + 5y ≤ 40....................................................... ( ii )
Polisher: 2x + 2y ≤ 40.................................................... ( iii )
for x ≥0 and y ≥ 0

Plot this on graph

Grinder: x +2y ≤ 40....................................................... ( i )


x y
0 20
40 0
Sander: x + 5y ≤ 40....................................................... ( ii )

x y
0 8
40 0

Polisher: 2x + 2y ≤ 40.................................................... ( iii )

x y
0 20
20 0
CHAPTER 15 Linear programming 287

x + 5y ≤ 40

Feasible Region

2x + 2y ≤ 40 x + 2 y ≤ 40

From the above we find that the solution is 15 Amethyst and 5 Beryls

Example 15.7

K ltd produces two products x and y. X has a contribution of KES 3 per unit and y has a
contribution of KES per unit. The manufacturer wishes to establish the weekly production plan
which maximizes contribution.

Machine Labor Materials


(hours) (hours) (hours)
X 4 4 1
Y 2 6 1
Total available 100 180 40
CHAPTER 15 Linear programming 288

Because of trade agreements, sales of x are limited to weekly maximum of 20 units and to
honor an agreement with an old established customer at least 1o units of y must be sold per
week.
Formulate a linear programming model that could be used to determine the production
quantities of each product so as to maximize profit

Solution

Objective function; 3x + 4 y

Subject to constraints

i) Machine hours constraints 4 x + 2y ≤ 100


ii) Labour hours constraints 4 x + 6y ≤ 180
iii) Material constraints x + y ≤ 40
iv) x sales constraints x ≤ 20
v) y sales constraints y ≥ 10
v) Non negativity x ≥ 0: y ≥ 0

Practice Problem

1. A firm makes two products Alpha and Beta both which require two raw materials RM1
and RM2. Each of the product, Alpha requires 2kg of RM1 and 3.5 kg of RM2. Each
kilogram of product beta requires 3kg of RM1 and 1.5 kg of RM2. Each week 100kg of
RM1 and 120kg of RM2 are available. There is an unlimited supply of labor and machine
time the firm can sell all its production. The unit profit on Alpha and beta is KES 50 and
KES 80 respectively
a) Set up a profit maximizing linear programming model for the problem

b) Set up the dual linear programming model

c) Find the optimum solution for the profit maximizing model graphically

2. A manufacturer who produces two products A and B has formulated the following linear
programming problem.
CHAPTER 15 Linear programming 289

Maximize: 640x + 480y

Subject to:

x + 2y ≤ 50

3x + 2y ≤ 180

x ≥ 10

y ≤ 20

x ≥ 0, y ≥ 0.

Solve the problem graphically stating the optimal solution and profit.

3. A Company manufactures two types of toys T1 and T2. Each toy is processed through
three machines A, B and C. Each T1 toy requires 4 hours of machine A, 2 hours of
machine B and 1 hour of machine C. A unit of T2 requires 2 hours on machine A, 2 hours
on machine B and 3 hours of machine C. EachT1 toy contributes KES 50 to profit and
each unit of T2 contributes KES 30 to profit. There are a maximum of 80 hours available
for machine A, a maximum of 50 hours available on machine B and 40 hours available on
machine C.

Formulate and solve the linear programming problem using the graphical method.

4. A mail order firm has to transport 900 parcels using a lorry, which can carry 150 parcels
at a time, and a van that can carry 80 parcels at a time. The cost each trip of a lorry is
KES. 500 and KES. 400 for the van. The total transportation cost must not exceed KES.
4,400 and the lorry make fewer trips than the van. Each lorry trip makes a profit of KES.
200 and a van trip KES. 100. Write down the relevant inequalities and determine the
best combination of trips for the lorry and the van that would yield maximum profit.

5. A shop stocks two types of detergents A and B. The shop can sell at least 3 times as many
units of A as of B but there room to store 32 units of both. Each unit of A sold yields a
profit of sh. 20 and each unit of B sh. 50. Write down the linear inequalities involved. By
graphing the inequalities, determine the number of units of each type required making
maximum profit.
CHAPTER 15 Linear programming 290

6. A Company produces two products A and B. Each unit of product A gives a profit
contribution of KES 60 and each unit of product B yields a profit of KES 50. Each unit of
product A requires 3 hours of polishing and 2 hours of plating. Each unit of product B
requires 4 hours of polishing and one hour of plating. There are 120 hours of polishing
available per week and 40 hours of plating available per week. Formulate and solve the
linear programming problem.

7. A paper manufacturing Company manufactures photocopying and duplicating papers.


Because of lack of raw materials not more than 400 tons of the photocopy paper can be
produced in a week and not more than 300 tons of duplicating papers per week. There are
160 production hours per week. Each ton of photocopy paper requires 12 minutes of
production time and each ton of the duplicating paper requires 24 minutes. The profit
contribution of the photocopy paper is KES. 500 per ton and that of the duplicating paper is
KES. 200 per ton. Formulate and solve the linear programming problem.

8. Graph the following inequalities and show the feasible region

(i) x + y ≥ 3 (ii) y < 3x + 5 (iii) y + x – 2 < 0 (iv) 3x + 2y – 6 ≥ 0 (v) 4x + 3y – 12 ≥ 0

(vi) x + y ≥ 8; x ≥ 0 ; y ≥ 0 (vii) x ≥ 1; y ≥ 1; 2x + 3y < 12; 5x + 2y < 20

(viii) x ≥ 2; y ≥ 1; x + y ≤ 7; y – x > 0

9. A Company produces two products X and Y. The plant capacity constrains the output of
X to a maximum of 50 units per day and that of Y to a maximum of 70 units per day. It
takes 4.8 minutes to manufacture a unit of X and 8 minutes to manufacture a unit of Y.
Each day has a maximum of 8 hours of production time. The profit per unit of X is KES. 80
and KES. 100 per unit of Y. Formulate and solve the linear programming problem.

10. S K ltd has received a special order from the ministry of special programme for high
quality protein biscuits for famine relief. S ltd must minimize costs and ensure that the
mix meets the minimum nutritional requirement set by the ministry of public health.
The LPO requires 1, 000 kgs of biscuit mix which is made of four ingredients, R, S,T, U
CHAPTER 15 Linear programming 291

that cost KES 8, KES 2, KES 3, and KES 1 per kilogram respectively. According to national
nutritional guidelines by the ministry of public health the batch must contain a minimum
of 400kgs of protein, 250 kgs of fat, 300 kgs of carbohydrates and 50 kgs of sugar.

11. The ingredients contain the following percentages by weight.

Protein Fat Carbohydrate Sugar Filler


R 50% 30% 15% 5% 0%
S 10% 15% 50% 15% 10%
T 5% 5% 30% 30% 5%
U 5% 5% 5% 30% 60%

a) Only 150 kg of T are immediately available


b) Formulate the linear programming problem in the standard manner

12. ANU is about to buy some publishing machines to enable it print it newly designed
distance learning packs and a choice of Type X or Type Y machine. The university has
budgeted KES 160,000 for the purchase of the machines. Type X machine costs KES 5, 00
each, Type Y machine costs KES. KES 10, 000 each require 10 hours of maintenance a
week and can produce 2,000 distance learning packs per week.

13. Use the graphical method to solve linear programming problems formulated below.

(i) Minimize: Cost, C = 30x + 20y (ii) maximize: Profit, P = 60x + 20y

Subject to: Subject to:

2x + 4y ≥ 10 4x + y ≤ 5

4x + 2y ≥ 10 and y ≥ 4, x ≥0 3x + 2y ≤ 7

x + y ≤ 3 and x, y ≥ 0

14. Each machine, X or Y needs 50 square meters of floor area. The available 1,000 square
meters of floor area and 400 hours of maintenance time each week. There is
overwhelming response to the distance learning mode and all distance learning packs
produced can be sold. The university management wishes to minimize output.
a) List objective function and constraints
b) Graph the constraints shading the feasible region
c) State the optimal mix of publishing machine to but with reasons
CHAPTER 15 Linear programming 292

15.A manufactures is to market a fertilizer which is to be a mixture of two ingredients


alpha and beta. The properties of the two ingredients are.

Bone Meal Nitrogen Lime Phosphate Costs


A 20% 30% 40% 10% KES 120
B 40% 10% 45% 45% KES 80

It has been decided that the fertilizer will be sold in bags containing minimum of 50kg
It must contain at least 15% nitrogen
It must contain at least 8% phosphate
It must contain at least 25% bone meal
The manufacturer wishes to meet the above requirement at the minimum cost possible-
16
INTRODUCTION TO
CHAPTER QUANTITATIVE TECHNIQUES

Learning Objectives
At the end of this section the students should be able to:

 Understanding the meaning of quantitative techniques


 Understand the Stages in Quantitative study
 Differentiate the different models

Quantitative techniques is the approach of modern science on complex problems arising in the
management of four Ms (man, machinery, money, in the large organization and also
government ministries and department.
It’s policy is to help management determine its policy and action scientifically.

Stages in Quantitative study

1) Problem Recognition and definition


This is the stage where one should appreciate that there is a problem in existence. One should
go ahead to understand the problem and explain it well.

2) Model building
A model building is any representation of reality and may be in graphical physical or
mathematical terms. It tries to show the working of the real world by means of simulators,
mathematical symbol, equations formula graphs.
A model can be classified into:

a) Normative model
Are concerned with finding the best optimum of ideal solution to a problem.

b) Descriptive model
This model describe the behavior of a system without attempting to find the best
solution to any problem. Example is simulation.
CHAPTER 16 Introduction to quantitative techniques 294

3) Data collection
Correct the relevant data in order to solve problem. Data collected is from revenue cost
production, quantity the uncertainty risk involved.

4) Problem solving
This involves the manipulation of the acquired data using standard mathematical means and
using recognized quantitative techniques where a large amount of data is used use of computer
is necessity.

5) Interpretation of the solution


This involves analysis a problem to whether it can work in real life situation. Accuracy of the
data used and assumption made must be properly analyzed.

6) Implementation of final solution


After careful interpretation of the result of the result of the study and modification where
appropriate then the resulting solution can be implemented.

7) Review and Maintenance


After implementation the performance of the model should be carefully monitored that it
actual does work and fulfill its objectives.
The review process should be done regularly and every stage of implementation.
All organization are subject to change and hence no solution and remain optimal for ever
It is therefore necessary for organization to continua review the model that is used and existing
solution to see if adjustment is required.

Example of Model
Mathematical Model
This are frequently used in management and particular in quantitative.

Iconic Model
This is visual model of the real object they represent. They may be larger or smaller the reality

Analagol Model
Use one self of physical movement or properties present another set

Simulation Model
Represent the behavior of a real system for example learning how to drive you use a table.

Heulistic Model
Use a set of intuitive rules which management hope will produce a worker able solution to a
better solution methods currently be used e.g. in programming where you set boundaries
17
LINEAR PROGRAMMING:
CHAPTER SIMPLEX METHOD

Learning Objectives
At the end of this section the students should be able to:

 Identify the slack variables


 Identify surplus variable
 Identify the feasible solution and region
 Identify shadow prices

Solving linear program using simplex

Simplex alogarithim is the most efficient method of solving linear programming. It is mostly
used where a linear program has more than two variables. The alogarithim use in built matrix
procedure as used in solving system of simultaneous equation. It is such method that seeks the
optimal solution using a matrix method.

Definition of Terms

Slack variable
Prior to solving linear problem using the simplex method the constraints variable must be
standardized .for the constraints of the form ≤ (less than) a value added to ensure equality. The
variable are called slack variable and they have a zero contribution towards the profit in
objective function.

Surplus variable
These are constraints of the form ≥ (greater than) a value is subtracted to ensure equality are
called surplus variable

Standardization
CHAPTER 16 Linear programming simplex method 296

The above subtraction and addition is called standardization. Standardization process should
not lead to the violation of non negativity requirement for all the variables.

Feasible Region
A an area that satisfies all the problem’s constraints that the area with the possible solutions

Feasible Solution
A point in the feasible or wanted region that satisfies all of the problem’s constraints.

Example 17.1

A steel structure produces two types of metal namely M 1 and M 2 subject constraints on three
raw materials a, b and c
The objective of the firm is to select a product mix which will maximize profit.
Linear program for the problem is
Produce x units of product M 1 per week and y units of M 2 per week
Maximize = p 2 x + y ( sh / week )

Subject to :
A : 3 x ≤ 27 kg
B : 2 y ≤ 30kg
C : x + y ≤ 20kg
x ≥ 0, y ≥ 0

Using the simplex method determine the product mix and maximum value of the weekly profit.

Solution

Steps in simplex method


Standardize the linear program
Maximize = P 2x + y
Subject to:
A : 3 x + S1 ≤ 27
B : 2 y + S 2 ≤ 30
C : x + y + S 3 ≤ 20
CHAPTER 16 Linear programming simplex method 297

Arrange the coefficient in the left hand side of the constraints equation in matrix format
Put the right-hand side of the constraints in separate column on the right of the variable
Label the row with the names of the variable which are basic that is the slack variable or the
surplus variable
Add the objective functions as an additional row to the table.

Find the lowest negative value in the objective function ( in our case it is -2 in the tableau 1.
Divide the right-hand side values (b column) by the corresponding number in the pivot column.
Choose the smallest positive ratio (in our case above) . The corresponding row is called pivotal
row. The intersection of pivotal row and pivotal column is the pivotal elements (in case 3)
Divide all the elements in the pivotal row by pivotal elements and replace the pivotal row by this
new row. Also replace the variable by the label from the pivotal column.

Tableau 3
Basic
Variables x y S1 S2 S3 b
R1 S1 1 0 1 0 0 9
3
R2 S2 0 2 0 1 0 30
R3 S3 1 1 0 0 1 20

R4 Z -2 -1 0 0 0 0

Using the arithmetic operation on the row reduce all the elements in pivot column to zero
This arithmetic operation must use only the pivot row as the bases.
CHAPTER 16 Linear programming simplex method 298

Tableau 4
Basic
Variables x y S1 S2 S3 b
R1 S1 1 0 1 0 0 9
3
R2 S2 0 2 0 1 0 30 R=
5 R3 − R1
R3
S3 0 1 −1 0 1 20
3
R4
Z -2 -1 0 0 0 0

Tableau 6
Basic
Variables x y S1 S2 S3 b
R1 x 1 0 1 0 0 9
3
R2 2
S2 0 0 1 -2 8
R5 3
y 0 1 −1 0 0 11
3
R=
7 R5 + R6
R6 2
Z 0 -1 0 0 18
3
CHAPTER 16 Linear programming simplex method 299

Tableau 7
Basic
Variables x y S1 S2 S3 b
R1 x 1 0 1 0 0 9
3
R2 2
S2 0 0 1 -2 8
R5 3
y 0 1 −1 0 0 11
3

R7 1
Z 0 0 0 1 29
3
Shadow prices

The value of x and y can be obtained from Tableau 7 as follows.

x=9
y = 11

Where x = 9 represent M 1 units per week.


Also where y = 11 represent M 2 units per week.
The maximum profit we must produce 9 units of M 1 and 11 units of M 2
The slack value S 2 which is equal to 8 means that at the end we shall have left 8kg of material B

The optimum profit is KES 29 and the shadow prices are


This means that 1 any extra kilogram of material A will add the profit by KES 1 and any extra
3 3
kilogram of raw material C will add the profit by KES 1

Example 17.2

Solve the following LP using simplex method


Maximum profit p = 8 x + 5 y + 10 z
CHAPTER 16 Linear programming simplex method 300

Subject to:

X- Represent number of product A


Y- Represent number of product B
Z- Represent number of product C

Solution

We must introduce the slack variables as follows

We can write this as follows

The above information can be tabulated in the tableau 1


CHAPTER 16 Linear programming simplex method 301

Divide the pivot row by 4 that is R3 so as to make the pivotal element to be 1

Basic
Variables x y z S1 S2 S3 S4 b

R1 S1 2 3 1 1 0 0 0 400
R2 S2 1 0 1 0 1 0 0 150 R=
6 R1 − R3
R3 S3 1 0 1 0 0 1 0 50 R= R2 − R3
2 4 7

R4 S4 0 1 0 0 0 0 1 50
R10= R5 + 10 R3
R5 Z -8 -5 -10 0 0 0 0 0

Using R3 as the base we can perform the above operation


We can obtain R= 6 R1 − R3 as follows.

R1 2 3 1 1 0 0 0 400
R3 1 0 1 0 0 1 0 50
2 4
R6 11 3 0 1 0 −1 0 350
2 4
R=
7 R2 − R3
R3 1 0 1 0 0 1 0 50
2 4
R2 1 0 1 0 1 0 0 150
R7 1 0 0 0 1 −1 0 100
2 4

R10= R5 + 10 R3
CHAPTER 16 Linear programming simplex method 302

R5 -8 -5 -10 0 0 0 0 0
10R3 5 0 10 0 0 0 21 500
2
R10 -3 -5 0 0 0 21 0 500
2
After performing the above with R3 changing to R8 and R4 changes to R9 form the table below.

We need to perform a similar operation that were done in Tableau 1 to the above Tableau 2
In the above where there is zero there are no operations to be made in the pivot column.

Tableau 2

Basic
Variables x y z S1 S2 S3 S4 b

R6 S1 11 3 0 1 0 −1 0 350
2 4
R7 S2 1 0 0 0 1 −1 0 100 = R6 − 3R9
R11
2 4
R8 S3 1 0 1 0 0 1 0 50
2 4
R9 S4 0 1 0 0 0 0 1 50
R=
15 R10 + 5 R9
R10 Z -3 -5 0 0 0 21 0 500
2

We can perform the following operation from the above tableau 2


= R6 − 3R9
R11
CHAPTER 16 Linear programming simplex method 303

R6 11 3 0 1 0 −1 0 350
2 4
3R9 0 3 0 0 0 0 3 150
R11 11 0 0 1 0 −1 -3 200
2 4

R=
15 R10 + 5 R9
R10 -3 -5 0 0 0 21 0 500
2
5R9 0 5 0 0 0 0 5 250
R15 -3 0 0 0 0 21 5 750
2

We can now put together the above column two R11 and R15 into the tableau 4 below.

Tableau 3

Basic
Variables x y z S1 S2 S3 S4 b

R11 S1 11 0 0 1 0 −1 -3 200
2 4
R12 S2 1 0 0 0 1 −1 0 100
2 4
R13 z 1 0 1 0 0 1 0 50
2 4
R14 y 0 1 0 0 0 0 1 50

R15 Z -3 0 0 0 0 21 5 750
2

We still have a negative in R15 and for the equation reach optimum we need to eliminate all
the negative since we have -3
CHAPTER 16 Linear programming simplex method 304

R13 can be changed to R18 by multiplying by 2 or dividing by 1 as given below


2

R13 z (1 0 1 0 0 1 0 50) ÷ 1
2 4 2
R18 x 1 0 1 0 0 1 0 100
2 2

When we replace R13 with R18 we are going to use that as our base to reduce the other
elements in the pivotal column to zero that is row R11 , R12 , and R15 . In R14 the element is
already zero so it is going to be transferred just the way it is.

Basic
Variables x y z S1 S2 S3 S4 b

R11 S1 11 0 0 1 0 −1 -3 200
2 4
R12 S2 1 0 0 0 1 −1 0 100 R= R11 − 1 1 R18
2 4 16 2
R18 x 1 0 1 0 0 1 0 100 R= R12 − 1 R
2 2 17 2 18
R14 y 0 1 0 0 0 0 1 50
R=
20 R15 + 3R18
R15 Z -3 0 0 0 0 21 5 750
2

First, R11 Will be replaced with R16 by subtracting 1 1 R18 from R11 as follows.
2
CHAPTER 16 Linear programming simplex method 305

R= R11 − 1 1 R18
16 2
R11 11 0 0 1 0 −1 -3 200
2 4
1 1 R18 11 0 3 0 0 3 0 150
2 2 4 4

R16 0 0 −3 1 -1 −3 3 50
4 4

Second R12 will be replaced by R17 subtracting 1 R18 from R12


2
R12 1 0 0 0 1 −1 0 100
2 4
1 R 1 0 1 0 0 1 0 50
2 18 2 4 4

R17 0 0 −1 0 1 −1 0 50
4 2

Final operation is changing R15 to R20 by adding R15 to 3R18 as given below.

R=
20 R15 + 3R18
R15 -3 0 0 0 0 21 5 750
2
3R18 3 0 3 0 0 3 0 300
2 2

R20 0 0 3 0 0 4 5 1050
2

We can now take R16 , R17 and R20 put them together to create a new tableau 4 as given below

Tableau 4

Basic
Variables x y z S1 S2 S3 S4 b

R16 S1 0 0 −3 1 -1 −3 3 50
4 4
R17 S2 0 0 −1 0 1 −1 0 50
4 2
R18 x 1 0 1 0 0 1 0 100
2 2
CHAPTER 16 Linear programming simplex method 306

R19 y 0 1 0 0 0 0 1 50

R20 Z 0 0 3 0 0 4 5 1050
2

Interpretation

x = 100 units are required to produce product A


y = 50 units are required to produce product B

Example 17.3

Slack variables
S1 = 50 It means that 50 machine hours are unused at optimum
S 2 = 50 It means that 50 components are unused at optimum

Shadow Prices

The shadow prices of 4 and 5 mean that:


For every extra kilo of alloy available 4 extra contributions would be gained.
Extra units of |B that was allowed to be produced overall contribution would increase by 5

1. Practice Problem
1. Duba Paints Company is specializing in the production of industrial varnish. The selling
prices and the associated unit variable costs for high gloss varnish and matt varnish are
shown in the table below.

Production Selling Price Unit Variable Cost


(KES) per Gallon (KES.) per Gallon
Matt 130 90
High Gloss 160 100

Each gallon of matt varnish requires 6 minutes of skilled labor and each gallon of high
gloss requires 12 minutes of skilled labor. In a given day there are 400 man haws of
CHAPTER 16 Linear programming simplex method 307

skilled labor available. Also there are 100 ounces of an important blending chemical
available each day, where each gallon of matt varnish needs 0.05 ounces of blending
chemical and each gallon of high gloss varnish needs 0.02 ounces of the chemical. The
processing capacity at the plant is limited to 3000 gallons of varnish per day.

The company is committed to supplying a leading retailer with 5000 gallons of matt
varnish and 2500 gallons of high gloss varnish each working week (consisting of five
days). In addition, there is an agreement with the unions that at least 2000 gallons are
produced each day. Dura Coat management would like to determine the daily
production volume of each of the two varnishes that will maximize total contribution.

a) Develop a linear model of the production problem facing Dura Coat paint.
b) Using a graphical approach determine the optimum daily production plan and the
consequent contribution.

2. A farm co-operative has 600 acre available to plant corn and soybeans. Each acre of corn
requires 9 gallons of fertilizer and hour of labor to harvest. Each acre of soybeans
require 3 gallons of fertilizer and I hour to harvest. The farm has available at most 40,500
gallons of fertilizer and at most 5250 of labor for harvesting. If the profits per acre are
KES 60 thousand for corn and KES 40 thousand for soybeans:

a) Form a linear program for this problem.


b) Use simplex method to determine; how many acres of each crop should the farm
plant in order to maximize profits and the maximum profit.

c) What will happen it 1000 gallons of fertilizer are added to the system.

3. Ken Ltd produce three kinds of malted drink. One of these they sell as a health drink
because it has less sugar; one they sell to hospitals as an invalid food as it has added
vitamins; the third one is a standard product.
The main ingredients, with their costs and normal weekly availabilities, are given in the
table below, as are the estimated maximum weekly demands for the three products.

Required kg per g of product


Estimated
Skimmed Maximum Sale price
Malt Milk Demand per Per kg
Sugar extract powder Week kg dollar
CHAPTER 16 Linear programming simplex method 308

Standard drink 0.30 0.30 0.35 2000 1.00


Health drink 0.15 0.25 0.55 1800 1.20
Invalid drink 0.15 0.30 0.25 1200 1.50

Costs per kg of 20p 60p 50p


raw material
Availability per 1000 1250 2200
Week of raw
Materials, kg

There is an unlimited supply of vitamin additives. Other variables costs are 10p per kg for
the standard drink, 9p per kg for the health drink and 12p per kg for the invalid drink.
a) Formulate the linear programming model for this problem with the objective of
maximizing total contribution per week.
b) The final simplex table for the solution to this problem is given below:

m h i S1 S2 S3 S4 S5 S6 b
S1 0 0 0 1 -1 0 0 0.1 0.15 110

m 1 0 0 0 3.333 0 0 -0.833 -1 1466.67

S3 0 0 0 0 -1.167 1 0 -0.258 0.1 396.67

S4 0 0 0 0 -3.333 0 1 0.833 1 533.33

h 0 1 0 0 0 0 0 1 0 1800

i 0 0 1 0 0 0 0 0 1 1200
p 0 0 0 0 1.617 0 0 0.251 0.56 3144.33

Where m, h, i refer to the standard, health and invalid drinks respectively. S1, S2, S3 refer to
the constraints on sugar, malt and skimmed milk, respectively S4, S5, S6 refer to the
constraints on the maximum demand for the standard, health and invalid drinks
respectively.

Determine:
(a) the optimum product mix;
(b) the maximum value of the weekly contribution;
(c) The spare capacity on the constraints.
CHAPTER 16 Linear programming simplex method 309

4. Mbeezi, a chicken feed manufacturer, is in the process of developing a new


revolutionary weaning Feed with the trademark “Kuku Mfalme” by mixing four
ingredients. The four ingredients, in, Combination, form three compounds that is
critical for the effectiveness of the feed. One kilogram of The feed must contain at
least 6 units of kemone, 5 units of kemtwo, and 4 units of kemthree. Kemone requires 9
units of ingredient one, 5 units of ingredient two, 4 units of ingredient three and 7 Units
of ingredient four. Kemtwo requires 10 units ingredient one, 3 units of ingredient two, 8
units of Ingredient three and 2 units of ingredient four. Kemthree requires 4 units
of ingredient one, 6 units of Ingredient two, 3 units of ingredient three and 7 units
of ingredient four. Ingredient one costs 8 shillings, ingredient two costs 4 shillings,
ingredient 3 costs 5 shillings and Ingredient four costs 4 shillings.

Required:
a) What is the objective?
b) Formulate the above problem as a linear programming model.
c) Using Solver in excel solve the problem.
d) Interpret the reduced cost associated with ingredient four.
e) How many units of the compounds Kemone, Kemtwo and Kemthree are in
five kilograms (standard packaging) of the optimal blend?
f) If the cost per kilogram of ingredient one increases from 8 shillings to 9
shillings, compute the cost of the five-kilogram package.
g) The minimum cost per kilogram of the blend needs to be reduced to 4.50
shillings. List the compounds whose requirements can be relaxed to realize
this goal.
h) An academic journal on animal nutrition has indicated that the requirements
for Kemone can relaxed to 5 units on the condition that the requirements for
Kemtwo are boosted to 7 units and that for Kemthree rose to 5.3 units. Is this
advisable? Explain.

5. A feed manufacturer produces three types of feeds; X1, X2 and X3. Production of each
type requires The use of three types of resources. Resource requirements (in
kilograms) and availability together With the profit contribution margins are shown
in the table below.

TYPE OF FEED
RESOURCES X1 X2 X3 AVAILABLE (KG)
1 7 8 7 6,500
2 2 3 3 1,800
3 6 4 5 5,000
PROFIT 7 9 9
MARGIN
CHAPTER 16 Linear programming simplex method 310

Required;
a) Formulate this manufacturing situation as a linear programming problem.
b) Using Solver in excel solve the problem.
c) Highlight the implication of the reduced cost for X3.
d) You have been informed that the availability of resource 2 can be raised to
2000kg and that the profit margin for X2 can go up to 10 shillings per unit.
Compute the new Z value.
e) The marketing department has established that there is a market for a new
product, which will require 5kg of resource 1, 4kg of resource 2 and 5kg of
resource 3. The new product has a contribution margin of 8 shillings per unit.
Advice management on the viability of the new product.
DUAL PROGRAM/ MINIMIZING SIMPLEX METHOD

Learning Objectives
At the end of this Section the students should be able to:

 Invert a standard Linear programming formulation


 Use simplex method to solve a minimizing linear programming problem
 Interpret the final tableau of minimizing problem

Example 17.4

Given the a problem in a standard manner


Maximum profit p = 3 x + 2 y + 4 z
Subject to:
5 x + y + z ≤ 300
2 x + 3 y + 5 z ≤ 600
x, y , z ≥ 0
Dual

Solution

Minimizing cost,
= C 300a + 600b
Subject to:
CHAPTER 16 Linear programming simplex method 312

5a + 2b ≥ 3
a + 3b ≥ 2
a + 5b ≥ 4
a ≥ 0, b ≥ 0
18
NETWORK ANALYSIS AND
CHAPTER PROJECT SCHEDULING

Learning Objectives

At the end of this chapter the student should be able to:


 Illustrate through a network flow diagram;
 Define and identify the critical path;
 Understand how find the critical path using the forward pass and
backward pass;
 Estimate the standard deviation of the project duration;
 Use probability in estimation of completion of project in network;
 Know what is meant by float and how it is calculated

Network analysis is a technique for planning of a project nature for example road construction,
building construction design and implementation program.
The analysis of a project falls into three categories:
The brake down of the project into set individual jobs (activities) and then arranging them in a
logical sequence.
Estimation of the duration of each activity and the identification of the jobs which control a
completion the project.
The estimation of the resource requirement of each activity the rescheduling of the activities to
meet a resource objective the reallocation of money or other resources to improve the
schedule.

Basic Network Terminologies


Activity
This is a task or job of work that use time and or resources.
We use arrow to represent activity

Event
It is a point in time and indicates the start or the finish of an activity or activities.

We use a circle to represent the event.

A Dummy activity
This activity that doesn’t consume time or resources. It is used to show logical sequence of
CHAPTER 18 Network Analyses and Project Scheduling 314

events. We use a dashed or dotted arrow line

Network
A combination of activities, dummy activities and events in logical sequence according to the
rules for drawing network

Critical path
It is a path in a network that takes the most time

Rules for drawing networks


There should be a starting event and ending events
Network should proceed from left to right
Networks are not drawn to scale that is the length of arrow doesn’t represent time used
Events should be progressively numbered from left to right
Two activities should share both starting and ending events

Starting event
Event number
Activity Time

Activity
Earliest starting latest
time starting time
Danglers are not allowed in networks

Example 18.1

A bridge construction project consist of the following activities precedence relationship and
time

Tasks Predecessor Duration


(weeks)
A - 2
B A 3
C A 5
D B,C 2
E B,D 4
F C 3
G E,F 1
CHAPTER 18 Network Analyses and Project Scheduling 315

Required
a) Draw a network diagram
b) Identity the critical path
c) Determine the project completion time

Solution

a)
4
7 7 3
5 F
C
5 2 6 4 7 1 8
1 2 2 7 7 D 9 9 E 13 13 G 14 14
0 0 A 2 2
3
B
3
5 7

b) Critical path

A-C-D-E-F

c) Project completion time

14 days
CHAPTER 18 Network Analyses and Project Scheduling 316

Example 18.2

Activity Predecessor Duration


(weeks)
A - 4
B - 6
C A 7
D B 3
E C 4
F D 8
G E,F 3

Solution

2 7 4
C 4
4 4 6 11 13 E
A 6 3 7
1
0 0 17 17 G 20 20
6 8
B 3 3 5 F
6 6 D 9 9
CHAPTER 18 Network Analyses and Project Scheduling 317

Example 18.3

Premier company is an engineering firm which has contracted to produce a bunch of machines
to be used by Beta shoes company.
Details are given below

Activity Immediate Activity time


predecessor (days)

A - 9
B - 8
C - 11
D A, B 12
E B, C 10
F B, C 15
G D, E 22
H F, G 8

Required
a) Illustrate the project by an arrow diagram
b) Identify the critical path
c) How many days will the project take?

Solution

a)

3
9 9
12
9
D
A

1 8 2 5 22 6 8 7
0 0 B 8 9 21 21 G 43 43 H 51 51

11 10 15
C E F
4
11 11
CHAPTER 18 Network Analyses and Project Scheduling 318

b) Critical path

A-D-G-H or C-E-G-H

c) 51 days

Using the Standard Normal Table

A standard normal table is one that has a mean of 0 and a standard deviation of 1. All the
normal table that you will come across are set up to handle random variables with µ = 0 and σ =
1.
χ −µ
Z= Where
σ

χ = Duration of the project

µ = The time calculated in the network diagram

σ = The standard deviation of the critical events

Z = number of standard deviation from X to the mean µ

Suppose we have µ = 100 and σ =15 and we are to calculate the probability of a random
variable X is less than 130, P(X < 130)

This means that the point X is 2.0 standard deviation to the right of the mean. We can look at
the table of normal curve areas attached at the appendices

But for our practice let use the following given table
CHAPTER 18 Network Analyses and Project Scheduling 319

Standard Normal Distribution: Table Values Represent AREA to the LEFT of the Z score.

Z .00 .01 .02 .03 .04 .05 .06 .07 .08 .09
0.0 .50000 .50399 .50798 .51197 .51595 .51994 .52392 .52790 .53188 .53586
0.1 .53983 .54380 .54776 .55172 .55567 .55962 .56356 .56749 .57142 .57535
0.2 .57926 .58317 .58706 .59095 .59483 .59871 .60257 .60642 .61026 .61409
0.3 .61791 .62172 .62552 .62930 .63307 .63683 .64058 .64431 .64803 .65173
0.4 .65542 .65910 .66276 .66640 .67003 .67364 .67724 .68082 .68439 .68793
0.5 .69146 .69497 .69847 .70194 .70540 .70884 .71226 .71566 .71904 .72240
0.6 .72575 .72907 .73237 .73565 .73891 .74215 .74537 .74857 .75175 .75490
0.7 .75804 .76115 .76424 .76730 .77035 .77337 .77637 .77935 .78230 .78524
0.8 .78814 .79103 .79389 .79673 .79955 .80234 .80511 .80785 .81057 .81327
0.9 .81594 .81859 .82121 .82381 .82639 .82894 .83147 .83398 .83646 .83891
1.0 .84134 .84375 .84614 .84849 .85083 .85314 .85543 .85769 .85993 .86214
1.1 .86433 .86650 .86864 .87076 .87286 .87493 .87698 .87900 .88100 .88298
1.2 .88493 .88686 .88877 .89065 .89251 .89435 .89617 .89796 .89973 .90147
1.3 .90320 .90490 .90658 .90824 .90988 .91149 .91309 .91466 .91621 .91774
1.4 .91924 .92073 .92220 .92364 .92507 .92647 .92785 .92922 .93056 .93189
1.5 .93319 .93448 .93574 .93699 .93822 .93943 .94062 .94179 .94295 .94408
1.6 .94520 .94630 .94738 .94845 .94950 .95053 .95154 .95254 .95352 .95449
1.7 .95543 .95637 .95728 .95818 .95907 .95994 .96080 .96164 .96246 .96327
1.8 .96407 .96485 .96562 .96638 .96712 .96784 .96856 .96926 .96995 .97062
1.9 .97128 .97193 .97257 .97320 .97381 .97441 .97500 .97558 .97615 .97670
2.0 .97725 .97778 .97831 .97882 .97932 .97982 .98030 .98077 .98124 .98169

Z = 2.000 will give a probability of


0.97725 which 97.73%

This that if this was to represent a group student sitting for quantitative techniques exam the
probability of a student randomly selected is sitting for the quantitative exam is less than 130 is
97.73%

We see that the probability could also be derived from the middle of the graph by

This is the area in the right-hand tail of curve


CHAPTER 18 Network Analyses and Project Scheduling 320

It can represented as below

Shaded area of
interest

55 70 85 100 115 130 145 X = Student

χ −µ
-3 -2 -1 0 1 2 3 Z=
σ

Another example is that a construction company can build a bridge in 100 days and the
standard deviation is 20 days. Recently the company signed a contract to build a bridge in 125
days. What is the probability that the company will not violate their construction contract?
χ −µ
Z=
σ

From the table Z = 1.25 can be located at 0.89435 by first looking up 1.2 left-hand column of
the table provided below and then moving to 0.05 column so as find the value of Z = 1.25
CHAPTER 18 Network Analyses and Project Scheduling 321

Standard Normal Distribution: Table Values Represent AREA to the LEFT of the Z score

Z .00 .01 .02 .03 .04 .05 .06 .07 .08 .09
0.0 .50000 .50399 .50798 .51197 .51595 .51994 .52392 .52790 .53188 .53586
0.1 .53983 .54380 .54776 .55172 .55567 .55962 .56356 .56749 .57142 .57535
0.2 .57926 .58317 .58706 .59095 .59483 .59871 .60257 .60642 .61026 .61409
0.3 .61791 .62172 .62552 .62930 .63307 .63683 .64058 .64431 .64803 .65173
0.4 .65542 .65910 .66276 .66640 .67003 .67364 .67724 .68082 .68439 .68793
0.5 .69146 .69497 .69847 .70194 .70540 .70884 .71226 .71566 .71904 .72240
0.6 .72575 .72907 .73237 .73565 .73891 .74215 .74537 .74857 .75175 .75490
0.7 .75804 .76115 .76424 .76730 .77035 .77337 .77637 .77935 .78230 .78524
0.8 .78814 .79103 .79389 .79673 .79955 .80234 .80511 .80785 .81057 .81327
0.9 .81594 .81859 .82121 .82381 .82639 .82894 .83147 .83398 .83646 .83891
1.0 .84134 .84375 .84614 .84849 .85083 .85314 .85543 .85769 .85993 .86214
1.1 .86433 .86650 .86864 .87076 .87286 .87493 .87698 .87900 .88100 .88298
1.2 .88493 .88686 .88877 .89065 .89251 .89435 .89617 .89796 .89973 .90147

Z = 1.25 will give a probability of


0.89435 which 89.43%

This can be drawn as follows

Shaded area of
interest

100 125 X = days

We are also told further that if the company is able to finish within 75 days it will be awarded by
the ministry of roads and works a bonus of KES 100,000, what is the probability that they will
receive the bonus?
CHAPTER 18 Network Analyses and Project Scheduling 322
χ −µ
Z=
σ

Sometime you may be given tables that do not have the negative which should be calculated as
follows

= 10. 57%
Alternatively this book provide the negative area of Z as given below
To look up for the Z = - 1.25 by first locating the column of – 1.2 and then moving to the 0.05
column and write down the figure.

Standard Normal Distribution: Table Values Represent AREA to the LEFT of the Z score
Z .00 .01 .02 .03 .04 .05 .06 .07 .08 .09
-1.2 .11507 .11314 .11123 .10935 .10749 .10565 .10383 .10204 .10027 .09853
-1.1 .13567 .13350 .13136 .12924 .12714 .12507 .12302 .12100 .11900 .11702
-1.0 .15866 .15625 .15386 .15151 .14917 .14686 .14457 .14231 .14007 .13786
-0.9 .18406 .18141 .17879 .17619 .17361 .17106 .16853 .16602 .16354 .16109
-0.8 .21186 .20897 .20611 .20327 .20045 .19766 .19489 .19215 .18943 .18673
-0.7 .24196 .23885 .23576 .23270 .22965 .22663 .22363 .22065 .21770 .21476
-0.6 .27425 .27093 .26763 .26435 .26109 .25785 .25463 .25143 .24825 .24510
-0.5 .30854 .30503 .30153 .29806 .29460 .29116 .28774 .28434 .28096 .27760
-0.4 .34458 .34090 .33724 .33360 .32997 .32636 .32276 .31918 .31561 .31207
-0.3 .38209 .37828 .37448 .37070 .36693 .36317 .35942 .35569 .35197 .34827
-0.2 .42074 .41683 .41294 .40905 .40517 .40129 .39743 .39358 .38974 .38591
-0.1 .46017 .45620 .45224 .44828 .44433 .44038 .43644 .43251 .42858 .42465
-0.0 .50000 .49601 .49202 .48803 .48405 .48006 .47608 .47210 .46812 .46414

The probability is 0.10565 which


gives 10.57%
CHAPTER 18 Network Analyses and Project Scheduling 323

Drawing this will yield the following diagram

Shaded area of
interest

75 100 X = days

Finally, what if the bridge is constructed in within 110 days what will be the probability
χ −µ
Z=
σ

Locate Z = 0.5 in the tables as shown below

Standard Normal Distribution: Table Values Represent AREA to the LEFT of the Z score

Z .00 .01 .02 .03 .04 .05 .06 .07 .08 .09
0.0 .50000 .50399 .50798 .51197 .51595 .51994 .52392 .52790 .53188 .53586
0.1 .53983 .54380 .54776 .55172 .55567 .55962 .56356 .56749 .57142 .57535
0.2 .57926 .58317 .58706 .59095 .59483 .59871 .60257 .60642 .61026 .61409
0.3 .61791 .62172 .62552 .62930 .63307 .63683 .64058 .64431 .64803 .65173
0.4 .65542 .65910 .66276 .66640 .67003 .67364 .67724 .68082 .68439 .68793
0.5 .69146 .69497 .69847 .70194 .70540 .70884 .71226 .71566 .71904 .72240

The probability is
0.69146
CHAPTER 18 Network Analyses and Project Scheduling 324

We need to subtract the probability of finishing within 125 days and that of finishing within 110
days.

0.89435 – 0.69146
= 0.20289
Thus the probability between 110 and 125 days is 20.29%
It can be represented as follows

Shaded area of
interest

100 110 125 X = days

Example 18.4

A Highway contract have been given a contract to recarpet Mai Mahiu road the project manager
has identified eight essential activities in this network. He has also listed the activity as below.

Preceding Time (in weeks)


Activity Activity Optimistic Most likely Pessimistic
A - 2 3 4
B - 5 5.5 9
C - 6 7 8
D A 4 8.5 10
E C 1 3.5 9
F B, E 1 3 5
G C 8 9 16
H F, G 1 3 5

Required
a) Calculate the expected time
b) Illustrate the project by an arrow diagram
CHAPTER 18 Network Analyses and Project Scheduling 325

c) What is the probability that the project will take less than 30 weeks

Solution

a + 4m + b
When calculating the expected time we use the formulae where;
6
a = Optimistic
b = Pessimistic
m = Mostly likely
2 + 4(3) + 4 18
A
= = = 3
6 6
5 + 4(5.5) + 9 36
B
= = = 6
6 6
6 + 4(7) + 8 42
C
= = = 7
6 6
4 + 4(8.5) + 10 48
D
= = = 8
6 6
1 + 4(3.5) + 9 24
E
= = = 4
6 6
1 + 4(3) + 5 18
F
= = = 3
6 6
8 + 4(9) + 16 60
G
= = = 10
6 6
1 + 4(3) + 5 18
H
= = = 3
6 6

The result will be as follows for the expected time

Activity Preceding Expected


Activity Time
A - 3
B - 6
C - 7
D A 8
E C 4
F B, E 3
G C 10
H F, G 3
CHAPTER 18 Network Analyses and Project Scheduling 326

b) An illustration of the project

2
3 12 8
3 D
A
4 3 5 3 6
1 6 11 14 17 17
F H 20 20
0 0 B
7 4 10
C E G
3
7 7

When calculating the probability that the project will take less than 30 days weeks, we use the

2
b−a
Following formulae δ =  
2

 6 

Using the critical path of the project C-G-H

Thus δ 2 = δ C2 + δ G2 + δ H2
2 2
8−6  2 4
δ  =
= 2
 =
C 
 6   6  36
2 2
 16 − 8   8  64
δ G2 
= =  = 
 6   6  36
2 2
 5 − 1   4  16
δ
= 2
 =
H  = 
 6   6  36
4 64 16 84
δ2 = + + = = 2.3
36 36 36 36
δ 2 = 2.3
δ = 2.3
= 1.52
CHAPTER 18 Network Analyses and Project Scheduling 327

Using the Chebychev’s theorem which shows the probability as given below by the chart

68%

95%
100%
−δ δ
Using the above we can be able to check the probability of 1.52 as follows

A 20 30 B
1
The total distance point A to B is equal to 1 since we are dealing with the probability and the
probabilities are equal to one.
That is we use the Z sore principle which is given by the formulae given below

χ −µ
Z= Where
σ

χ = Duration of the project

µ = The time calculated in the network diagram

σ = The standard deviation of the critical events

Hence χ = 30, µ = 20 and σ = 1.52


CHAPTER 18 Network Analyses and Project Scheduling 328

Example 18.5

Given the following information, Draw the Network diagram and identify critical path. What is
the probability that project will take 30 days to complete

Job No Immediate Time Estimates


predecessor a m b
A - 2 3 4
B A 1 2 3
C A 4 5 12
D A 3 4 5
E B 1 3 5
F C 1 2 3
G D 1 8 9
H E, F 2 4 6
I H 2 4 12
J G 3 4 5
K I, J 5 7 8

Solution

a + 4m + b
Using the formulae we have to calculate the expected time as given below
6

a = Optimistic
b = Pessimistic
m = Mostly likely
CHAPTER 18 Network Analyses and Project Scheduling 329

2 + 4(3) + 4 18
A
= = = 3
6 6
1 + 4(2) + 3 12
B
= = = 2
6 6
4 + 4(5) + 12 36
C
= = = 6
6 6
3 + 4(4) + 5 24
D
= = = 4
6 6
1 + 4(3) + 5 18
E
= = = 3
6 6

1 + 4(2) + 3 12
F
= = = 2
6 6
1 + 4(8) + 9 42
G
= = = 7
6 6

2 + 4(4) + 6 24
H
= = = 4
6 6
2 + 4(4) + 12 30
I
= = = 5
6 6
3 + 4(4) + 5 24
J
= = = 4
6 6
5 + 4(5) + 8 42
K
= = = 7
6 6

The expected time will be as given by the table below.

Job No Immediate Expected


predecessor Time
A - 3
B A 2
C A 6
D A 4
E B 3
F C 2
G D 7
H E, F 4
I H 5
J G 4
K I, J 7

Using the above table we can now use network diagram to represent the above information,
CHAPTER 18 Network Analyses and Project Scheduling 330

3
5 6
3
E 10
2 B 6 4 8
5 9 7
24 24
2 5 9 H 12 12 I 17 17 K
0 3 4 F
1 6
0 0 A 9 7
3 3 C
4 J
4
D
5 7 7
7 7 G 14 17

When calculating the probability that the project will take 30 day we must first calculate using
2
b−a
the following formulae δ 2 =  
 6 
We must identify the critical path which is

Example 18.6

Activity Predecessor Time in days Cost


a m b expected
in shillings
A - 3 4 5 1000
B - 4 7 10 1400
C - 4 5 6 3000
D A 5 6 7 1200
E B 2 2.5 6 900
F C 10 10.5 14 2500
G D, E 3 4 5 800
H G, F 1 2 9 300

A certain project had the following information given by the project manager as given in the
table above.
CHAPTER 18 Network Analyses and Project Scheduling 331

The project indirect costs are KES 1300. The contract specifies a penalty of KES 1000 if the
project is not finished by the end of 15 days.
Required

Draw the network and state overall expected project completion time.
Calculate the cost of the project.
State the critical activities.
What is the probability could be completed without incurring any penalty.

Solution

a + 4m + b
We have to calculate the expected time using the formulae as follows.
6
3 + 4(4) + 5 24
A
= = = 4
6 6
4 + 4(7) + 10 42
B
= = = 7
6 6
4 + 4(5) + 10 30
C
= = = 6
6 6
5 + 4(6) + 7 36
D
= = = 6
6 6
2 + 4(2.5) + 6 18
E
= = = 3
6 6
10 + (10.3) + 14 66
F
= = = 11
6 6
3 + 4(4) + 5 24
G
= = = 4
6 6
1 + 4(2) + 9 18
H
= = = 3
6 6

Now we have the table of the expected time as follows

Activity Predecessor Duration Cost Expected


A - 4 1000
B - 7 1400
C - 5 3000
D A 6 1200
E B 3 900
F C 11 2500
G D, E 4 800
CHAPTER 18 Network Analyses and Project Scheduling 332

H G, F 3 300

Using the table above we draw the network diagram below

2
4 6 6
4 D
A 5 4 6 3 7
3 10 12 G 16 16 H 19 19
1 7 3 E
0 0 B 7 9
11
5
F
C
4
5 5

The cost of the project will be


Add all the expected cost

1000 + 1400 + 3000 + 1200 + 900 + 2500 + 800 + 300 =


11100

Add the variable cost of 1300 per day

1300 ×19 =
24700

Add a penalty of 1000(19 − 15) =


4000

Total cost = 11100 + 24700 + 4000 = 39800

Critical path C-F-H


Probability that the project will be completed without incurring any penalty is calculated as
follows

δ 2 = δ C2 + δ F2 + δ H2
CHAPTER 18 Network Analyses and Project Scheduling 333
2
b−a
δ =
2

 6 
2
 6−4
=δ C2 =  0.11
 6 
2
 7−5
=δ =
2
F  0.44
 6 
2
 9 −1 
=δ H2 =  1.78
 6 
δ 2 = 0.11 + 0.44 + 1.78
δ= 0.11 + 0.44 + 1.78
δ = 2.33
= 1.53

Using the principle that we developed previously of the Z score we can calculate the probability
as follows
χ −µ
Z= Where
σ

χ = Duration of the project

µ = The time calculated in the network diagram

σ = The standard deviation of the critical events

Hence χ = 15, µ = 20 and σ = 1.53

15 − 19 4
Z= = −
1.53 1.53

= −2.62

From the tables we get 0.4955 hence 0.5000-0.4955

=0.0045
CHAPTER 18 Network Analyses and Project Scheduling 334

15 19 days
0.5

The total cost of the project depends on the cost of individual activities plus any additional
valuables of fixed cost. Since all activities must be completed whether they are critical or not
Note that the total; cost of the activities is simply the sum of individual values may be possible
to reduce the duration of a specific activity by employing additional resources.
The implication of this is the cost of that activity will increase. However if the activity is critical a
saving in its duration (time) may result in overall saving in time on the cost and consequently
reduction in the project cost.
The shortest possible time in which an activity can be completed is called crash time

Example 18.7
XYZ Company is involved in the following projects
Activity Predecessor Normal Crash Normal Crash Cost
Time Time Cost (KES) (KES)
(days) (days)
A - 8 4 15,000 18,000
B - 10 8 17,000 22,000
C - 6 5 12,000 14,000
D A, B 8 5 26,000 32,000
E B, C 9 6 28,000 33,000
F C 14 11 29,000 36,000
G D, E 14 10 27,000 37,000
H F, G 6 4 11,000 13,000

In addition to cost of each activity there is site cost KES 1500 per day

a) What is the minimum time in which the project can be completed?


b) What is the associated minimum additional cost
CHAPTER 18 Network Analyses and Project Scheduling 335

Solution
4
4 9
(8) 10 11 5
4 A D (8)

1 8 (10) 3 6 (9) 5 10 (14) 6 4 (6) 7


0 0 B 8 8 E 14 14 G 24 24 H 28 28
10 10 19 19 39 39
5 33 33
C (6) 11
F
2
5 8
8 10

We first calculate the normal cost by adding all the normal cost of every individual activity as
follows.

Normal cost = 15,000 + 17,000 + 12,000 + 26,000 + 28,000 + 29,000 + 27,000 + 11,000

= 165,000

Variable Cost = 39 × 1500 = 58,500

Total normal cost

= 165,000+58,500= 223,500

Calculating the crash time we are going to use the duration in the network diagram

Crash cost=18000 + 22000 + 14000 + 32000 + 33000 + 36000 + 37000 + 13000

= 205500
Variable Cost= 28 × 1500 = 42,000

Total crash cost


=205,500 + 42,000 = 247, 500

The additional cost will be given by getting the difference between the crash time and the
normal cost,
CHAPTER 18 Network Analyses and Project Scheduling 336

Additional cost = Total crash cost - Total normal cost

247, 500 - 223,500

=24,000

The critical activities are B – E – G – H

Look at the non critical events that is A, C, D, and F and see whether there is possibility of using
the normal time and still finishing the project 28 days (crash time)

If so uncrash A saving 18,000 – 15,000 = 3000

Uncrash C to save 14,000 – 12,000= 2000 and don’t uncrash D

Uncrash F to save 36,000 – 29, 000= 7000

We have save 247, 500 – 12,000 = 235,500

235,500 – 223,500 = 12, 000

Float time

Consider the following problem of network series

Tail Head

6 6 7
4 7 K 9 14
Earliest Tail Latest Head time
Time

Total float
Total float is the amount of time path activities could be delayed without affecting the overall
project duration. It is given by:

Total Float = Latest Head Time – Earliest Tail Time – Activity Duration

14 – 4 – 6 = 4
CHAPTER 18 Network Analyses and Project Scheduling 337

Free float
Free float time is the amount of time activity can be delayed without affecting the
commencement of a subsequent activity at its earliest start time but may affect float of a
previous activity

Free time = Earliest Head Time – Earliest Tail Time – Activity Duration

9–7–6=1
This means that there will be a deficit of on day before the project finishes.

Independent Float
Independent Float is the amount time activity can be delayed when all preceding activities are
completed as late as possible and all succeeding activities are completed as early as possible.

Independent Float = Earliest Head time – Latest Tail Time – Activity duration

9 – 7 – 6 = -4

Example 18.8
A manufacturer wishes to investigate the cause of engine failure in last 3 DC 10 plane crash.
Below are the activities their interrelation and their respective track of time.

Task Immediate Duration in


Predecessor days
A - 3
B A 20
C A 15
D A 10
E B, C, D 15
F E 10
G B, F 5
H F 3
I G, H 12
Draw the network diagram determine
J I 2
project completion time
Calculate the float in each activity
CHAPTER 18 Network Analyses and Project Scheduling 338

Solution

9
3 46 48
13 18
10
3 H
D

1 3 2 15 5 15 6 10 7 5 10 12 11 2 12
0 0 A 3 3 C 18 18 E 33 33 F 43 43 G 48 48 I 60 60 J 62 62

20
B
4
23 18
CHAPTER 18 Network Analyses and Project Scheduling 339

1.
Practice Problem
1. A certain project had the following had information

Activity Predecessor Normal Normal Crash Extra cost for


time cost Time reduction of
one week
A - 2 4000 1 4000
B A 1 0 1 0
C B 4 2000 2 1250
D B 6 4500 2 1750
E D 3 7000 1 2500
F C, E 3 2000 1 2000
G F 4 6000 1 1250
H D 2 0 2 0
I D 3 2500 2 2000
J H, I 8 6000 4 1000
K G, J 2 4500 1 2500
L K 2 2000 1 1000

Variable overhead cost of KES 3000 per week for the project duration
a) Determine the normal overall completion time and total cost of the project
b) Determine the minimum time in which the project can be completed and
associated minimum cost

2. PK Automation company has developed a proposal for introducing a new computerized


office system that will improve accounting processes together with the interface
communication for auditing firms with branches all over Kenya. As shown below

Activity Predecessor Normal Crash Normal Crash


Time Time Cost (KES) Cost
(weeks) (weeks) (KES)
A - 10 8 30,000 70,000
B A 7 6 120,000 150,000
C B 10 7 100,000 160,000
D A 7 6 40,000 50,000
E D 10 8 50,000 75,000
F C,E 3 3 60,000 60,000
CHAPTER 18 Network Analyses and Project Scheduling 340

In addition there is a variable cost of KES 1000 per week


a) Draw a network and identify the shortest possible time that can take
b) Identify the critical activities
c) If the company has KES 400,000 how much should they borrow if they want
complete the project in 25 weeks

3. Dema Builders Ltd has been awarded a tender to construct a bridge. The bridge
construction project consist of the following activities

Activity Predecessor Optimistic Most Pessimistic


Likely
A - 2 3 4
B A 1 2 3
C A 4 5 12
D A 3 4 11
E B 1 3 5
F C 1 2 3
G D 1 8 9
H E,F 2 4 6
I H 2 4 12
J G 3 4 5
K I,J 2 7 9

a) Draw the network diagram.


b) Identify the critical activities.
c) What is the probability that the project will take more than 30 days to complete.

4. The chartered institute of quantitative decision makers runs an annual programme of


meetings for its members. The staffs of the institute’s Head office begin to prepare the
programme for the year after the previous financial year. The programme includes
details of the speakers and their talks as well as the list of current members. The
activities necessary in the preparation of its programme are given below, together with
the immediate preceding activities.

a) Draw a network for the project.


b) Determine the overall time required to produce and distribute this programme if
no crashing is done.
c) What would be the effect on the overall duration of the project if the time taken
for activity C had been underestimated and it took 30 days to finish it?
d) What is the shortest possible time which the programme can be produced and
distributed?
CHAPTER 18 Network Analyses and Project Scheduling 341

Activity Predecessor Normal Crash Additional


time (day) time cost (KES)
(day)
A - 5 5 -
B A 20 10 10,000
C - 15 10 15,000
D - 15 5 20,000
E D 30 25 5,000
F B,C,E 10 5 10,000
G F 10 5 5,000
H G 15 10 7,500
I E 5 2 5,000
J H,I 5 2 5,000

5. Given the following information about a project by the table below.

Activity Immediate Optimistic Most Pessimistic Pessimistic


preceding likely Cost in KES at
activity expected
duration
A - 3 4 5 100,000
B - 4 7 10 140,000
C - 4 5 6 200,000
D A 5 6 7 120,000
E B 2 2.5 6 90,000
F C 10 10.5 14 250,000
G D,E 3 4 5 80,000
H G,F 1 5 9 30,000

The project indirect costs are KEs 30,000 per day. The contract with the customers
specifies a penalty of KES 10,000 per day if the project is not finished at the end of 15
days.

a) Draw a project network for this project


b) Identify the critical path
c) Determine the overall expected project completion time and its associated cost.
d) Determine the probability that the project could be completed without incurring
any penalty charge.

6. Explain the meaning of the following terms as used in network analysis


a) Crash time/cost
b) Non-critical activities.
c) A dummy
CHAPTER 18 Network Analyses and Project Scheduling 342
7. The following activities have been identified by Hydra Company Limited as the one
required in launching a new product.

Activities Preceding Time (Weeks)


Activities
A - 8
B - 4
C A 4
D A 2
E C, D 3
F B 2
G - 1
H E, F 2
I H, G 3
J I 4
K J 3
L K 4

a) Draw a network to represent the logical sequence of tasks.


b) How long will the product take before it is launched?

The time taken to complete tasks A, B, D, K and L is somewhat uncertain and so the following
optimistic and pessimistic estimates have also been made to supplement the most likely figure
given above. The additional estimates are:

Activities Optimistic Time Pessimistic Time


A 5 13
B 2 6
D 1 4
K 2 6
L 2 8

a) What is the expected time until the product is launched?


b) What is the probability of this time exceeding 35 weeks?
CHAPTER 19 TRANSPORTATION
MODEL

Learning Objectives

At the end of this chapter the student should be able to:


• Identify and recognize a transportation problem;
• Set up the starting transportation table;
• Make the starting feasible allocations;
• Improve the starting/initial allocation by calculating the shadow
costs;
• Know when to include dummy destination;

Transportation problem is concerned with allocation of items between suppliers (origin) and
consumers (destination) so that the total cost allocation is minimized. This kind of problem can
be solved using linear programming or a special transportation algorithm.
For as to solve a problem using transportation algorithm a number of conditions must be made.
a) The cost per item for each combination of origin and destination must be specified
b) The supply of items at each origin must be known.
c) The requirement of item at each destination must be known.
d) The total supply must be equal to the total demand

Stages of Transportation Algorithm


Stage I
Arrange the data in table form and find any feasible allocation. Note that a feasible allocation is
one which all demand at the destination is satisfied and all supply at the origins is allocated.
Stage II
Test the allocation to see whether it is the optimal allocation
Stage III
If the allocation is not optimal reallocation in order to move to a better lower cost solution.
Stage IV
Test again for optimality and repeat this process until an optimum allocation is found.

Finding an Allocation
There are two method of finding initial allocation which the book will focus on:
a) Minimum cost method
CHAPTER 19 Transportation model 344

b) Vogel method

Solving a transportation problem using minimum cost


Example 19.1
Three deports A, B, and C can supply 900, 400 and 800 bags of sugar respectively. Three
supermarkets at 1, 2 and 3 require 300, 500 and 600 bags respectively. What is the minimum
cost of allocating the bags of sugar from the depots to the supermarket if the unit
transportation cost are shown in the table below.

Table 1
Sugar Transportation cost in Total
KES to supermarkets available
Deports 1 2 3
A 100 200 50 900
B 20 100 80 400
C 10 200 70 800
Total 300 500 600
Required

While Using Minimum cost method we must:


a) Allocate as much as possible to the cell with the minimum unit cost
b) Adjust the remaining availability and requirement
c) Choose the next smallest cost and allocate as much as possible to this cell until supply
and demand are all zero
Note
If more than one cell has smallest value of unit cost choose one at random, we must introduce a
dummy since the supply is more than the demand

Table 2
Sugar Transportation cost in Total
shs to supermarkets available
Deports 1 2 3
A 100 200 50 900
B 20 100 80 400 A total of 2100
C 10 200 70 800
Total 300 500 600
Required
A total of 1400
CHAPTER 19 Transportation model 345

The table above will be added one column for the dummy supermarket that is to be created in
order the supply will be equal to the demand.

Table 3
Sugar Transportation cost in shs to Total
supermarkets available
Deports 1 2 3 4
A 100 200 50 0 900
B 20 100 80 0 400 Supply is 2100
C 10 200 70 0 800
Total 300 500 600 700
Required

Demand is 2100

Now we make our allocation as shown below in the table 4

Table 4
Sugar Transportation cost in shs to Total
supermarkets available

Deports 1 2 3 4
A 100 200 50 0 900
B 20 100 80 0 400
C 10 200 70 0 7001 800 100
Total 300 500 600 700
Required 0

In column 4 the cost are all zero and we can allocate at random by choosing one of the three
zeros
When allocating for example if we choose cell C4 we check what is the demand which is (700
bags) and supply which is (800 bags) subtract what we need and leave the rest for another
allocation.
If the allocation is exhausted that is it is have given us a zero after the allocation we no longer
need the cells in that column or row and we close them by putting a cross as given in the Table 5
CHAPTER 19 Transportation model 346

Table 5
Sugar Transportation cost in shs to Total
supermarkets available

Deports 1 2 3 4
A 100 200 50 0 900
B 20 100 80 0 400
C 10 200 70 0 7001 800 100
Total 300 500 600 700
Required 0

Now we move to the next smallest cost which is C1 (10) which requires 300 bags while our
deport can only supply 100 bags we allocate the 100 bags, the remain 200 bags to allocate later
as given in the table 6

Table 6
Sugar Transportation cost in shs to Total
supermarkets available

Deports 1 2 3 4
A 100 200 50 0 900
B 20 100 80 0 400
C 10 1002 200 70 0 7001 800 100 0
Total 300 500 600 700
Required 200 0

We can see that in row C the supply has been exhausted and it is now a zero, this means that
we don’t need the rest of the remaining cells in that row and we must cross them as in the table
7 given below.
CHAPTER 19 Transportation model 347

Table 7
Sugar Transportation cost in shs to Total
supermarkets available

Deports 1 2 3 4
A 100 200 50 0 900
B 20 100 80 0 400
C 10 1002 200 70 0 7001 800 100 0
Total 300 500 600 700
Required 200 0

We again check the smallest cost which is in row B (20) and we see that supermarket 1 require
200 bags and there is 400 bags which deport B is able to supply so we deduct what we require
that is 200 bags as in the table 8

Table 8
Sugar Transportation cost in shs to Total
supermarkets available

Deports 1 2 3 4
A 100 200 50 0 900
B 20 2003 100 80 0 400 200
C 10 1002 200 70 0 7001 800 100 0
Total 300 500 600 700
Required 200 0
0

The supermarket 1 demand has been exhausted and the total required is zero now and we need
to cross the remaining one cell in the column as shown in table 9
CHAPTER 19 Transportation model 348

Table 9
Sugar Transportation cost in shs to Total
supermarkets available

Deports 1 2 3 4
A 100 200 50 0 900
B 20 2003 100 80 0 400 200
C 10 1002 200 70 0 7001 800 100 0
Total 300 500 600 700
Required 200 0
0

Repeating the same procedure we check the smallest cost again on the remain cells which is in
row A (50), in that figure we see what is required is 600 bags by supermarket 3 and deport A is
able to supply 900 bags, thus we deduct what we need which is 600 bags as shown in the table
10

Table 10
Sugar Transportation cost in shs to Total
supermarkets available

Deports 1 2 3 4
A 100 200 50 6004 0 900 300
B 20 2003 100 80 0 400 200
C 10 1002 200 70 0 7001 800 100 0
Total 300 500 600 700
Required 200 0 0
0

The column of the supermarket 3 have been exhausted and now the required bags are zero thus
we need to cross the remaining one cell as in the table 11
CHAPTER 19 Transportation model 349
Table 11
Sugar Transportation cost in shs to Total
supermarkets available

Deports 1 2 3 4
A 100 200 50 6004 0 900 300
B 20 2003 100 80 0 400 200
C 10 1002 200 70 0 7001 800 100 0
Total 300 500 600 700
Required 200 0 0
0

Finally we can allocate the two remaining cell A2 and B2, using the same principle we check the
smallest cost between these two which is B2 (100) that means that supermarket 2 requires 500
bags and deport B is willing and able to supply 200 bags at cost of KES 100, so we take the 200
bags available and get the remaining 300 bags to deport A which is able to supply 300 bags at a
cost of KES 200.
Table 12 shows both allocations starting with the smallest cost which is (100)

Table 12
Sugar Transportation cost in shs to Total
supermarkets available

Deports 1 2 3 4
A 100 200 3006 50 6004 0 900 300
B 20 2003 100 2005 80 0 400 200
C 10 1002 200 70 0 7001 800 100 0
Total 300 500 600 700
Required 200 300 0 0
0 0

The minimum cost that the supermarket can incur is summarized below
From deport A allocate 300 bags to supermarket 2 300 × 200 = 60,000
From deport A allocate 600 bags to supermarket 3 600 × 50 = 30,000
From deport B allocate 200 bags to supermarket 1 200 × 20 = 4,000
From deport B allocate 200 bags to supermarket 2 200 × 100 = 20,000
From deport C allocate 100 bags to supermarket 1 100 × 10 = 1,000
700 bags will remain in deport C 700 × 0 = 0
115,000
CHAPTER 19 Transportation model 350

Alternative allocation

We can use the same minimum cost method to allocate by now we can select at random the
smallest cost to be A4 and start from there.
Following all the steps we have done in the above we can come up with the final table as shown
in table 13

Table 13
Sugar Transportation cost in shs to Total
supermarkets available

Deports 1 2 3 4
A 100 200 50 2003 0 7001 900 200
B 20 100 4005 80 0 400 0
C 10 3002 200 1006 70 4004 0 800 500 100 0
Total 300 500 600 700
Required 0 100 400 0

From the above alternative allocation the summary is given below

From A allocate 200 bags to supermarket 3 200 × 50 = 10,000

700 bags will remain at deport A 700 × 0 = 0

From B allocate 400 bags to supermarket 2 400 × 100 = 40,000

From C allocate 300 bags to supermarket 1 300 × 10 = 3,000

From C allocate 100 bags to supermarket 2 100 × 200 = 20,000

From C allocate 200 bags to supermarket 3 400 × 70 = 28,000

101,000
CHAPTER 19 Transportation model 351

Vogel method
This method uses penalty cost for each row and column the penalty costs is the difference
between cheapest available route (lowest cost) and the next cheapest.
When you calculate the difference between the lowest two costs in each row and column, pick
the highest penalty and allocate as much as possible to the cell with the smallest cost.
Using the example 1 we can illustrate Vogel method, first we introduce one row of penalty and
another column of penalty and get the difference between the two lowest costs and write them
down in the penalty column and row as in the table 14

Example 19.2

Table 14
Sugar Transportation cost in shs to Total Penalties
supermarkets available

Deports 1 2 3 4 P1
A 100 200 50 0 900 50
B 20 100 80 0 400 20
C 10 200 70 0 800 10
Total 300 500 600 700
Required

P1 10 100 20 0
Penalties

When we are allocating using this method we concentrate on the penalties and pick the column
and row the highest value on either, for our case in the table 14, pick 100 as the highest figure.
Using that 100 check the column and allocate to the lowest value and note that our interest is
the values on the Deport A, B and C in that column.
CHAPTER 19 Transportation model 352

The lowest value is 100 and that is where our allocation where supermarket 2 require 500 bags
and deport B is able to supply 400 bags , we deduct what the deport is supplying and we are
left with a demand of 100 bags which will be supplied later as shown in the table 15

Table 15
Sugar Transportation cost in shs to Total Penalties
supermarkets available

Deports 1 2 3 4 P1
A 100 200 50 0 900 50
B 20 100 4001 80 0 400 0 20
C 10 200 70 0 800 10
Total 300 500 600 700
Required

P1 10 1001 20 0
Penalties

The supply in row B is exhausted and all the cells in that row except where there is allocation
are to be crossed as in table 16

Table 16
Sugar Transportation cost in shs to Total Penalties
supermarkets available

Deports 1 2 3 4 P1
A 100 200 50 0 900 50
B 20 100 4001 80 0 400 0 20
C 10 200 70 0 800 10
Total 300 500 600 700
Required 100

P1 10 1001 20 0
Penalties
CHAPTER 19 Transportation model 353

Repeat the same process of getting the penalties (P2) from the remaining cost without
considering the crossed cells as in table 17

Table 17
Sugar Transportation cost in shs to Total Penalties
supermarkets available

Deports 1 2 3 4 P1 P2
A 100 200 50 0 900 50 50
B 20 100 4001 80 0 400 0 20
C 10 200 70 0 800 10 10
Total 300 500 600 700
Required 100

P1 10 1001 20 0
P2 90 20 0
Penalties

Identify the highest penalty for our case in the table 17 is (90) and allocate to the lowest cost
value which is (10) as in the table 18

Table 18
Sugar Transportation cost in shs to Total Penalties
supermarkets available

Deports 1 2 3 4 P1 P2
A 100 200 50 0 900 50 50
B 20 100 4001 80 0 400 0 20
C 10 3002 200 70 0 800 500 10 10
Total 300 500 600 700
Required 0 100

P1 10 1001 20 0
P2 902 20 0
Penalties
CHAPTER 19 Transportation model 354

Since the column with allocation have been exhausted we have to cross the remaining cell A1 as
in the table 19

Table 19
Sugar Transportation cost in shs to Total Penalties
supermarkets available

Deports 1 2 3 4 P1 P2
A 100 200 50 0 900 50 50
B 20 100 4001 80 0 400 0 20
C 10 3002 200 70 0 800 500 10 10
Total 300 500 600 700
Required 0 100

P1 10 1001 20 0
P2 902 20 0
Penalties

Repeat the same process of generating the penalties (P3) by getting the difference of between
the two least values in each row and column as in table 20

Table 20
Sugar Transportation cost in shs to Total Penalties
supermarkets available

Deports 1 2 3 4 P1 P2 P3
A 100 200 50 0 900 50 50 50
B 20 100 4001 80 0 400 0 20
C 10 3002 200 70 0 800 500 10 10 70
Total 300 500 600 700
Required 0 100

P1 10 1001 20 0
P2 902 20 0
Penalties

P3 0 20 0
CHAPTER 19 Transportation model 355

From the penalties we pick the highest penalty for our case we have (70) and allocate that as
penalty (703) which is in the column of penalties and allocate to the lowest cost value which is
zero as in table 21

Table 21
Sugar Transportation cost in shs to Total Penalties
supermarkets available

Deports 1 2 3 4 P1 P2 P3
A 100 200 50 0 900 50 50 50
B 20 100 4001 80 0 400 0 20
C 10 3002 200 70 0 800 500 10 10 703
Total 300 500 600 700
Required 0 100

P1 10 1001 20 0
P2 902 20 0
Penalties

P3 0 20 0

We need to allocate 700 bags but the deport C is able to supply 500 bags allocate what is
available and leave the remaining for another allocation as in table 22

Table 22
Sugar Transportation cost in shs to Total Penalties
supermarkets available

Deports 1 2 3 4 P1 P2 P3
A 100 200 50 0 900 50 50 50
B 20 100 4001 80 0 400 0 20
C 10 3002 200 70 0 5003 800 500 10 10 703
Total 300 500 600 700
Required 0 100 200

P1 10 1001 20 0
P2 902 20 0
Penalties

P3 0 20 0
CHAPTER 19 Transportation model 356

In Deport C we have exhausted the supply and we have to cross the remaining cells since there
are no longer available as in table 23

Table 23
Sugar Transportation cost in shs to Total Penalties
supermarkets available

Deports 1 2 3 4 P1 P2 P3
A 100 200 50 0 900 50 50 50
B 20 100 4001 80 0 400 0 20
C 10 3002 200 70 0 5003 800 500 10 10 703
Total 300 500 600 700
Required 0 100 200

P1 10 1001 20 0
P2 902 20 0
Penalties

P3 0 20 0

The rest remaining cells can be allocated without allot of mechanics since we can assume they
are penalties and allocate from the highest penalty of 200, 50, and then last figure is zero as in
table 24.

Table 24
Sugar Transportation cost in shs to Total Penalties
supermarkets available

Deports 1 2 3 4 P1 P2 P3
A 100 200 1004 50 6005 02006 900 800 600 50 50 50
B 20 100 4001 80 0 400 0 20
C 10 3002 200 70 0 5003 800 500 10 10 703
Total 300 500 600 700
Required 0 100 200
0 0
P1 10 1001 20 0
P2 902 20 0
Penalties

P3 0 20 0
CHAPTER 19 Transportation model 357

The above table 24 is fully allocated and the following are the summary of the allocation.

From deport A allocate 100 bags to supermarket 2 200× 100 = 20,000


From deport A allocate 600 bags to supermarket 3 600× 50 = 30,000
From deport A , 200 bags are remaining 200 × 0 = 0
From deport B allocate 400 bags to supermarket 2 400 × 100 = 40,000
From deport C allocate 300 bags to supermarket 1 300× 10 = 3,000
From deport C allocate 500 bags to supermarket 4 500× 0 = 0
93,000

Testing for optimality


Stepping stone is a technique that is used in trying to find an optimal solution for s problem
which is not optimal. This technique has two unique parts; the first involves testing the current
solution to determine if improvement is possible and the second involves making changes to the
current solution in order to make it optimal.
Modified distribution module, in both if the allocation is not optimal a stepping stone procedure
is used to move to the next basic allocation.

Testing for optimality using Module

In general the row component is added ui and the column component v j .


In our case let start by assigning component v1 , v2 , v3 and v4 to supermarket 1, 2 , 3 and
respectively and assign u1 , u2 and u3 respectively.

Table 25
Sugar Transportation cost in KES to Total
supermarkets available
v1 v2 v3 v4
Deports 1 2 3 4
u1 A 100 200 1004 50 6005 02006 900
u2 B 20 100 4001 80 0 400
u3 C 10 30 v 02 200 70 0 5003 800
4

Total 300 500 600 700


Required
CHAPTER 19 Transportation model 358

Test whether the allocation is basic we use m + n − 1 m + n − 1 where m is the number of columns
and n the number of rows which is supposed to give the number of allocation.
For our example 4 + 3 − 1 =6 and our allocation were six which means it is basic.
If the allocation is not basic then allocate zero to any of the allocated cell.
After allocating the component the cells we are going to calculate in our case are
A2 , A3 , A4 , B2 , C1 , and C4 they give the following simultaneous equations.

A2 = u1 + v2 = 200
A3 = u1 + v3 = 50
A4 = u1 + v4 = 0
B2 = u2 + v2 = 100
C1 = u3 + v1 = 10
C4 = u3 + v4 = 0

By convention the first deport (U) is assigned the value of zero that u1 is equal to zero. Thus
given that u1 = 0 then we can calculate v2 as:

u1 = 0
A2 = u1 + v2 = 200
0 + v2 = 200
v2 = 200

Also we can calculate v3 given that u1 = 0 by using A3 = u1 + v3 = 50

A3 = u1 + v3 = 50
u1 = 0
0 + v3 =
50
v3 = 50

Given that u1 = 0 we can calculate the value of v4 using A4 = u1 + v4 = 0

A4 = u1 + v4 = 0
u1 = 0
0 + v4 =
0
v4 = 0

We can be able to calculate u2 since we have v2 using equation B2 = u2 + v2 = 100


CHAPTER 19 Transportation model 359

B2 = u2 + v2 = 100
u2 = 200
200 + v2 =
100
v2 100 − 200
=

v2 = −100

Given that v4 = 0 we can use C4 = u3 + v4 = 0 to find u3

C4 = u3 + v4 = 0
v4 = 0
u3 = 0

Using u3 = 0 we can be able to calculate v1 taking C1 = u3 + v1 = 10

C1 = u3 + v1 = 10
u3 = 0
0 + v1 =
10
v1 = 10

Using the values below we can be able to calculate the shadow cost of an occupying cells and
can be calculated using the following values.

v1 = 10
u1 = 0
v2 = 200
and u2 = −100
v3 = 50
u3 = 0
v4 = 0

Shadow cost
A1 = u1 + v1 = ?
B1 = u2 + v1 = ?
B3 = u2 + v3 = ?
B4 = u3 + v4 = ?
C2 = u3 + v2 = ?
C3 = u3 + v3 = ?

After substituting the shadow cost using the value given above we obtain.
CHAPTER 19 Transportation model 360

A1 =0 + 10 =10
−100 + 10 =
B1 = −90
−100 + 50 =
B3 = −50
B4 =−100 + 0 =−100
0 200 =
C2 =+ 200
C3 =0 + 50 =50

We are supposed to get the difference between the actual and the shadow cost.

Table 26
Actual cost Shadow Difference
cost
A1 100 10 +90
B1 20 -90 +110
B3 80 -50 +130
B4 0 -100 +100
C2 200 200 0
C3 70 50 +20

It is a basic allocation since all the values are positive and any unit you take in given cell A1 will
increase by +90 and allocation is optimum.

From deport A allocate 100 bags to supermarket 2 200× 100 = 20,000


From deport A allocate 600 bags to supermarket 3 600 × 50 = 30,000
From deport A, 200 bags are remaining 200 × 0 = 0
From deport B allocate 400 bags to supermarket 2 400 × 100 = 40,000
From deport C allocate 300 bags to supermarket 1 300 × 10 = 3,000
From deport C allocate 500 bags to supermarket 4 500 × 0 = 0
93,000

Example 19.3

A firm of office equipment suppliers has three deports located in various in various terms. It
receives orders for a total of 600 laptops from 4 customers in total in the three deports there
are 600 laptops available and management wishes to minimize delivery by dispatching the
laptops from the appropriate deports for each customers.
Details of availability requirement and transport cost per laptop are given below.
CHAPTER 19 Transportation model 361

Table 27
A B C D Total
available
X 130 110 200 200 80
Y 170 140 130 130 240
Z 180 180 120 120 280
Total 120 120 160 200
required

Required
Make a feasible allocation of deliveries.
Check the solution to see if it represents the minimum cost possible and hence calculate the
minimum cost.

Solution

Now we are already familiar with making allocation using either the minimum cost method or
the Vogel method. In our case we are going to use the Vogel method because it gives the
minimum cost possible.
We are going to use the final table bearing in mind that you already know to follow the steps to
make the allocation as in table 28.

Table 28
A B C D Total Penalties
available

P1 P2 P3
X 130 110 200 200 80 0 0
801
Y 170 140 130 130 240 120 0
1202 1203 10 10 10
Z 180 180 120 120 280 200 80
405 404 1206 30 30 30
Total 120 120 160 200
required 40 0 40 0
0 0

P1 401 30 30 10
Penalties

P2 10 402 30 10
P3 10 0 303 10
CHAPTER 19 Transportation model 362

We test whether the allocation is basic using the formulae m + n − 1 where 4 + 3 − 1 =6 and the
number of the allocation that we have made are six thus our problem is basic. Now let test for
optimality and introduce our row component ui and column component v j .
In our case let start by assigning component v1 , v2 , v3 and v4 to supermarket 1, 2 , 3 and
respectively and assign u1 , u2 and u3 respectively.

Table 29
A B C D Total
available

v1 v2 v3 v4
u1 X 130 110 200 200 80
801
u2 Y 170 140 130 130 240
1202 1203
u3 Z 180 180 120 120 280
405 404 1206
Total 120 120 160 200
required

After allocating the component the cells lets calculate following simultaneous equations.

X A = u1 + v1 = 130
YB = u2 + v2 = 140
YC = u2 + v3 = 120
Z A = u3 + v1 = 180
Z C = u3 + v3 = 150
Z D = u3 + v4 = 120

By convention u1 = 0 we can use X A = u1 + v1 = 130 can be solved as follows.

X A = u1 + v1 = 130
u=0
0 + v1 =
130
v1 = 130
CHAPTER 19 Transportation model 363

Using Z A = u3 + v1 = 180 given that v1 = 130 we can calculate as follows

Z A = u3 + v1 = 180
v1 = 130
u3 + 130 = 180 − 130
u3 = 50

With u3 = 50 we can be able to calculate v3 and v4 using the following two equation

Z C = u3 + v3 = 150 and Z D = u3 + v4 = 120

Z C = u3 + v3 = 150
u3 = 50
50 + v3 = 150 − 50
v3 = 100

Z D = u3 + v4 = 120
u3 = 50
50 + v4 =
120
v4 = 70

Given v3 = 100 using YC = u2 + v3 = 120 we can be able to solve u2

YC = u2 + v3 = 120
v3 = 100
u2 + 100 =
120
u2 = 20

Finally given that u2 = 20 we can be able to solve v2 using Y2 = u2 + v2 = 140

Y2 = u2 + v2 = 140
u2 = 20
20 + v2 =
140
v2 = 120

Using the values that we have solved for let us now calculate the shadow cost of the un
occupied cells given the values of ;
CHAPTER 19 Transportation model 364
v1 = 130
u1 = 0
v2 = 120
u2 = 20 and
v3 = 100
u3 = 50
v4 = 70

Let find the value of the following simultaneous equation

X B = u1 + v2 = 120
X C = u1 + v3 = 100
X D = u1 + v4 = 70
YA = u2 + v1 = 150
YD = u2 + v4 = 90
Z B = u3 + v2 = 170

Get the difference between the actual cost and the shadow cost

Table 30
Actual cost Shadow Difference
cost
XB 110 120 -10
XC 150 100 +50
XD 200 70 +130
YA 170 150 +20
YD 130 90 +40
ZB 180 170 +10

This means that the total cost could be reduced by KES 10 for every unit and that can be
transferred to X B .
Since there is a cost reduction that can be made then the above solution is not optimal.
To make the solution optimal we follow the following steps
If there is more than one empty cell negative shadow costs choose the cell with the largest
negative value.
Find the stepping circuit for this empty cell.
The table 31 on the next page shows the stepping circuit
CHAPTER 19 Transportation model 365

Table 31
A B C D Total
available

v1 v2 v3 v4
u1 X 80
801 +
u2 Y 1202 1203+ 240

u3 Z + 405 404 1206 280

Total 120 120 160 200


required

Note that when moving the stepping circuit always move horizontally and vertically but not
diagonal. When there more than two values on the rows or column use only two points
according to which cell is your focus to avoid two same signs that is why we are advised to use
even cell not odd.
Using our largest negative which is -40 now any cell that is negative subtract the highest value
that is (-40) and add (+40) to every cell where there is a positive sign.
Again note that the move should not interfere with the allocation.

Table 32
A B C D Total
available

v1 v2 v3 v4
u1 X 80
40 801 +40
u2 Y 40 1202 1203+40 240

u3 Z 40 + 405 404 40 1206 280

Total 120 120 160 200


required
CHAPTER 19 Transportation model 366

The new allocation will appear as the one on table 33 and it will not interfere with the previous
allocation

Table 33
A B C D Total
available

v1 v2 v3 v4
u1 X 80
40 40
u2 Y 80 160 240

u3 Z 80 200 280

Total 120 120 160 200


required

The new allocation cells are as given below

X A = u1 + v1 = 130..............................(i )
X B = u1 + v2 = 110.............................(ii )
YB = u2 + v2 = 140..............................(iii )
YC = u2 + v3 = 120...............................(iv)
Z A = u3 + v1 = 180...............................(v)
Z D = u3 + v4 = 120...............................(vi )

By convention u1 = 0 we solve for v1 using the first equation X A = u1 + v1 = 130

X A = u1 + v1 = 130.........................(i )
u1 = 0
0 + v1 =
130
v1 = 130

Using the same u1 = 0 we can be able to solve v2 using the second equation X B = u1 + v2 = 110
CHAPTER 19 Transportation model 367

X B = u1 + v2 = 110...............................(ii )
u1 = 0
0 + v2 =
110
v2 = 110

Let take v2 = 110 , we can get the value of u2 using the third equation YB = u2 + v2 = 140

YB = u2 + v2 = 140..............................(iii )
v2 = 110
u2 + 110 = 140 − 110
u2 = 30

Using the value of u2 = 30 to solve equation four YC = u2 + v3 = 120

YC = u2 + v3 = 120............................(iv)
u2 = 30
30 + v3 = 120 − 30
v3 = 90

Previously in our first equation we had solve for the value v1 = 130 lets solve for u3 using
Z A = u3 + v1 = 180
Z A = u3 + v1 = 180..............................(v)
v1 = 130
u3 + 130 = 180 − 130
u3 = 50

Finally with the value of u3 = 50 we can use the sixth equation to calculate the value of v4 using
Z D = u3 + v4 = 120
Z D = u3 + v4 = 120...................................(vi )
u3 = 50
50 + v4 = 120 − 50
v4 = 70

The summary of the values that we have solve are given below
CHAPTER 19 Transportation model 368
v1 = 130
u1 = 0
v2 = 110
u2 = 30 and
v3 = 90
u3 = 50
v4 = 70

We use the values to solve the shadow cost of the unoccupied cells:

X C = u1 + v3 = 90
X D = u1 + v4 = 70
YA = u2 + v1 = 160
YD = u2 + v4 = 100
Z B = u3 + v2 = 160
Z C = u3 + v3 = 140

Get the difference between the actual cost and the shadow cost

Table 34
Actual Cost Shadow cost Difference

XC 150 90 +60
XD 200 70 +130
YA 170 160 +10
YD 130 100 +30
ZB 180 160 +20
ZC 150 140 +10

The cost before optimal allocation would have been

X A = 130 × 80 = 10400
YB = 140 ×120 = 16800
YC = 120 ×120 = 14400
Z A = 180 × 40 = 7200
Z C = 150 × 40 = 6000
Z D =120 × 200 = 24000
78, 800

After the optimum allocation the cost will be as follows


CHAPTER 19 Transportation model 369

X A = 130 × 40 = 5200
X B = 110 × 40 = 4400
YB = 140 × 80 = 11200
YC = 120 ×160 = 19200
Z A = 180 × 80 = 14400
Z D =120 × 200 = 24000
78, 400
The difference is (78,800-78,400) = 400
This means for every item moved to X B we will save KES 10 and have 400 that means we need
40 units times 10 gives 400.

Degeracy

A solution is degerate when there are fewer than m + n − 1 allocation in the final table that is if
not basic it is degerate.
It can be overcome by allocating a very small amount essential zero to an independent cell

Example 19.4

Three warehouses XYZ can supply 600, 300 and 400 items to three retailers I, 2 and 3. The
retailer requires 400, 500 and 100 items respectively. Calculate the optimum allocation and
hence optimum transportation cost using the table below.

Table 35
From To Retailers
warehouse 1 2 3
X 60 40 90
Y 50 30 20
Z 20 30 60
CHAPTER 19 Transportation model 370

Solution

Include a row for the demand that is what is required by the retailers and a column for the
supply that is what is available in the warehouse.

Table 36
From To Retailers Total
warehouse available
1 2 3
X 60 40 90 600
Y 50 30 20 300
Z 20 30 60 400
Total 400 500 100
required

Let check whether the what is required is equal to what is available

Table 37
From To Retailers Total
warehouse available
1 2 3
X 60 40 90 600
Y 50 30 20 300 A total of 1300
Z 20 30 60 400
Total 400 500 100
required

A total of 1000

There is a difference between the total required and the total available, a difference of 300
items thus we need to create a dummy retailer number 4 so as to make the demand and supply
equal. Lets introduce a new column for a forth retailer.
CHAPTER 19 Transportation model 371

Table 38
From To Retailers Total
warehouse available
1 2 3 4
X 60 40 90 0 600
Y 50 30 20 0 300
Z 20 30 60 0 400
Total 400 500 100 300
required

We can now start the allocation by using Vogel method where we will introduce the penalties
row and columns

Table 39
From To Retailers Total Penalties
warehous available
e 1 2 3 4
X 60 40 90 0 600
Y 50 30 20 0 300
Z 20 30 60 0 400
Total 400 500 100 300
required
Penalties

By getting the difference between the two least numbers we get our penalty one that is
between the rows and column and the peck the highest as given below.
CHAPTER 19 Transportation model 372

Table 40
From To Retailers Total Penalties
warehouse available
P1
1 2 3 4

X 60 40 90 0 600 40
Y 50 30 20 0 300 200 20
1001
Z 20 30 60 0 400 20
Total 400 500 100 300
required 0

P1 30 0 401 0
Penalties

Let’s move to the next penalty disregarding the already column we have allocated and make the
second allocation using the highest penalty

Table 41
From To Retailers Total Penalties
warehouse available
P1 P1
1 2 3 4

X 60 40 90 0 600 300 40 401


3002
Y 50 30 20 0 300 200 20 20
1001
Z 20 30 60 0 400 20 20
Total 400 500 100 300
required 0 0

P1 30 0 401 0
P1 30 0 0
Penalties

Using the third penalty to make our forth allocation as given below by table 42
CHAPTER 19 Transportation model 373

Table 42
From To Retailers Total Penalties
warehouse available
P1 P2 P3
1 2 3 4

X 60 40 90 0 600 300 40 401 20


3002
Y 50 30 20 0 300 200 20 20 20
1001
Z 20 30 60 0 400 0 20 20 10
4003
Total 400 500 100 300
required 0 0 0

P1 30 0 401 0
P2 30 0 0
Penalties

P3 303 0

Make the final two allocations starting with the highest (40) and followed by (30)

Table 43
From To Retailers Total Penalties
warehouse available
P1 P2 P3
1 2 3 4

X 60 40 90 0 600 300 40 401 20


3004 3002
Y 50 30 20 0 300 200 20 20 20
2005 1001
Z 20 30 60 0 400 0 20 20 10
4003
Total 400 500 100 300
required 0 0 0

P1 30 0 401 0
P2 30 0 0
Penalties

P3 303 0
CHAPTER 19 Transportation model 374

When you examine the final table 43 to see whether the problem is basic m + n − 1 that is
4+3-1 = 6 but the allocation that we have just made are 5 and thus mean the solution is optimal.

Practice Problem

1. Naive supermarkets have four super markets in different towns at present the four
super markets are supplied from two go downs X and Y each with a capacity of 80, 000
tons per day. It is planned to expand the supermarket so that they will require 54, 000,
50, 000, 60, 000, and 70,000 per day respectively from the go down system.
To meet this and future demand Naive of planning to third go down capable of supplying
120, 000 tons per day. Two locations are being considered the transport costs in
KES/tons from each go down to each super market are given below.

Go Transport cost per tone to supermarket


downs (i) (ii) (iii) (iv)
X 7000 8500 5500 12000
Y 11000 9000 7500 11000
1 11500 11500 7000 9000
2 18500 9500 8000 7500

Required
Evaluate the two transportation model and decide which is the better location of new go
down. Assume all other cost are the same

2. Explain the terms


a. Degeneracy
b. Non-unique optimal solution, in transportation problems

3. A company has been given a tender to supply maize to four drought zones. The distance
between each grains depot and each drought zone is given below.

Grain Distance in miles Zones


deport (i) (ii) (iii) (iv)
A 168 172 175 183
B 156 160 158 163
C 138 140 135 145
D 147 142 140 145
CHAPTER 19 Transportation model 375
Required
a. How the maize should be distributed to the zones in order to minimize the total
distance traveled. Use Hungarian algorithm.
b. Calculate the minimum distance to be covered.

4. There are four areas that urgently need emergency food aid due to the effect of draught.
Government has food storage in the following towns. The amount available, the amount
needed per station together with the associated transportation costs are in the table
below.

Transport costs/ bag in KES to emergency


From go area Total
down Turkana Kitui Kwale Marsabit available
Eldoret 180 110 150 200 200 000
Nakuru 170 140 120 130 600 000
Kitale 180 180 150 120 700 000
Total required 300 000 300 000 400 000 500 000

a) Determine the optimal way of distributing the food.


b) Calculate the associated total transportation cost

5. Write brief notes on the meaning of the following words as used in Transportation
Problem
a) Basic Feasible Solution
b) Optimal Solution
c) Non-Degenerate Basic Feasible Solution

6. Find the initial feasible solution of the following transportation problem by Vogel’s
Approximation Method.
Test for optimality and advice the management.

i ii iii Availability

A 16 19 12 14

B 22 13 19 16

C 14 28 8 12

Requirement 10 15 17
CHAPTER 19 Transportation model 376

7. Two warehouses supply four stores. The transportation costs from the warehouses to
the stores, the availabilities at the warehouse and requirements at the stores are given
in the table below.

From Transport Costs in KES ‘000’ per

warehouse item

G H I J Available

1 4 3 5 6 100

2 8 2 4 7 200

Required 50 100 75 75

a) How should the goods be allocated to minimize transport costs?


b) Calculate the minimum cost of transfer

8. Expel Transporters has four large stores in different towns: Kisumu, Eldoret, Nakuru, and
Nairobi. At present the four are supplied from the warehouse A and B, each with a
capacity of 40 tons per day.
It is planned to expand the stores so that they will 27, 25, 30 and 35 tons per day
respectively from the warehouse system. To meet this future increases, it is planned to
build a third warehouse, capable to supplying 60 tons per day. Two locations are being
considered. The transport costs in thousands of shillings per tons from each warehouse
to each store are given below

Distance in kilometers

From Customers

Warehouse

Kisumu Eldoret Nakuru Nairobi

A 70 85 55 120

B 110 90 75 110

New 1 115 115 70 90

New 2 135 95 80 75
CHAPTER 19 Transportation model 377

Evaluate the two transportation model and decide which is the better location the new
warehouse assuming all other costs are the same
CHAPTER 20 ASSIGNMENT MODEL

Objectives

At the end of this chapter the student should be able to do the following:
• Know and understand when to use the assignment model
• Know how to set up the initial assignment model table
• Be able move to the next improved table.
• Know how to recognize the optimum solution
• Deal with origin and destination
• To maximize and minimize a given a problem

Introduction

The assignment model is a special case of the transportation model in which the number of
origin must equal the number of the destination that is the table is a square that is the number
of rows and columns must be equal.
At the destination the demand is equal to one at the origin that is the supply is equal to one.
The commonly used such structure to solve such problem is specially designed solution called
Hungarian Algorithm

Hungarian Algorithm
This is a method that provides us with an efficient means of finding optimal solution without
having to make a direct comparison of every option.
The algorithm has three stages which are:

Stage I
Set out the problem in table format.
For each row in the table find the smallest row element and subtract it from every element in
the row.
Repeat the same for column.
CHAPTER 20 Assignment model 379

Stage II
For a feasible solution there must be exactly one assignment to every row and every column.
Find rows with only one zero in it and make an assignment to this zero. If no such row exists
begin with any zero.
Close out all the zeros in the same column.
Repeat number (i) and (ii) until no further progress can be made.

Stage III
If the solution in stage II is not feasible go on to stage III and follow the following steps;
Draw the minimum number of straight line through the rows and columns so that all zeros in
the table are covered and make sure that there are no diagonals.
Find the smallest element without a line through it.
Subtract this number from every element without a line through it.
Add the chosen number to every element with two lines through it.
Leave alone all the elements with one line through them.

This procedure has created at least one new zero return to the stage II and repeat the
procedure until the optimum solution is reached.

Example 20.1

A ferries manufacturer has four distribution ports and four order to deliver to separate country.
Each port has one ship available which is large enough to carry one of these ferries. The distance
between each port and each country are given below.

Tableau 1

Distance from port to country


(in miles)
u v w x
Ports

i) 680 720 750 830


ii) 560 600 580 630
Iii) 380 420 350 450
iv) 470 450 400 450
Required
How should be the orders be assigned to ports in order to minimize the total distance travelled.
CHAPTER 20 Assignment model 380

Solution

Identify the smallest element in each row to set up the initial as tableau 2

Tableau 2

Distance from port to country


(in miles)
Smallest element
u v w x In each row

i) 680 720 750 830 680


ii) 560 600 580 630 560
Iii) 380 420 350 450 350
iv) 470 450 400 450 400

Subtract the smallest row element from the entire element in each corresponding row as given
in the tableau 3

Tableau 3

Distance from port to country


(in miles)

u v w x
i) 0 40 70 150
ii) 0 40 20 70
Iii) 30 70 0 100
iv) 70 50 0 50

Identify the smallest element in each column as in tableau 4


CHAPTER 20 Assignment model 381

Tableau 4

Distance from port to country


(in miles)

u v w x
i) 0 40 70 150
ii) 0 40 20 70
Iii) 30 70 0 100
iv) 70 50 0 50
0 40 0 50
Smallest element in each column

Subtract the smallest element from the entire element in each corresponding column as given
below by the tableau 5

Tableau 5

Distance from port to country


(in miles)

u v w x
i) 0 0 70 100
ii) 0 0 20 20
Iii) 30 30 0 50
iv) 70 10 0 0

Assign to any of the row or column with one zero and to that zero that is column (x) as shown
below use square to show the assignment
CHAPTER 20 Assignment model 382

Tableau 6

Distance from port to country


(in miles)

u v w x
i) 0 0 70 100
ii) 0 0 20 20
Iii) 30 30 0 50
iv) 70 10 0 0
Move to the next row or column with one zero if there is none move to the column of your
choice if it has more than one assign one and close the others in that column as shown by the
tableau 7

Tableau 7

Distance from port to country


(in miles)

u v w x
i) 0 0 70 100
ii) 0 0 20 20
Iii) 30 30 0 50
iv) 70 10 0 0

Lets move to the next column (v) and assign to one zero and close the other zero as shown in
the tableau 8

Tableau 8

Distance from port to country


(in miles)

u v w x
i) 0 0 70 100
ii) 0 0 20 20
Iii) 30 30 0 50
iv) 70 10 0 0
CHAPTER 20 Assignment model 383

Finally we can assign the last column (u) which have two zeros, thus we need to assign to one
zero and close the other zero as given below

Tableau 9

Distance from port to country


(in miles)

u v w x
i) 0 0 70 100
ii) 0 0 20 20
Iii) 30 30 0 50
iv) 70 10 0 0

Looking at the tableau 9 we can that every row and every column has one assignment that
means that the solution is feasible and we can assign as follows
Pick the corresponding amount of mile which are in the first table tableau 2 to everywhere
there is assignment.
Assign port (i) to country (u) = 680
Assign port ii) to country (v) = 600
Assign port iii) to country (w) = 350
Assign port iv) to country (x) = 450
2080 miles

Alternative assignment

We need to assign column (u) need to assign to cell ii)u as the alternative assignment as shown
below by tableau 10
CHAPTER 20 Assignment model 384

Tableau 10

Distance from port to country


(in miles)

u v w x
i) 0 0 70 100
ii) 0 0 20 20
Iii) 30 30 0 50
iv) 70 10 0 0

Assign the column (v) that is cell i)v as alternative assignment as shown below in the tableau 11

Tableau 11

Distance from port to country


(in miles)

u v w x
i) 0 0 70 100
ii) 0 0 20 20
Iii) 30 30 0 50
iv) 70 10 0 0
We have assigned every row and every column and the solution is feasible and we can make the
following assignments.

Assign port i) to country (u) = 560


Assign port ii) to country (v) = 720
Assign port iii) to country (w) = 350
Assign port iv) to country (u) = 450
2080 miles
CHAPTER 20 Assignment model 385

Example 20.2

A company has six sales region and six sales ladies from pat experience it is known that sales
lady perform differently in different region. The company sales director has established sales for
each region. She has approached you for advice in who should take which region.

Tableau 12

Sales Nyanza Rift Nairobi Eastern Mt. Costal


lady Valley Kenya
A 68 72 75 83 75 69
B 56 60 58 63 61 59
C 35 38 40 45 25 27
D 40 42 47 45 53 36
E 62 70 68 66 69 70
F 65 63 69 70 72 68

Required
How should the sale director assign the sales ladies to the region to maximize total sales?

Solution
The problem which involves maximization require we multiply the whole table by negative as
given below by Tableau 13

Tableau 13

Sales Nyanza Rift Nairobi Eastern Mt. Costal


lady Valley Kenya
A -68 -72 -75 -83 -75 -69
B -56 -60 -58 -63 -61 -59
C -35 -38 -40 -45 -25 -27
D -40 -42 -47 -45 -53 -36
E -62 -70 -68 -66 -69 -70
F -65 -63 -69 -70 -72 -68
CHAPTER 20 Assignment model 386

After multiplying every element by negative lets identify the smallest element in each row as
given by the tableau 14

Tableau 14

Sales Nyanza Rift Nairobi Eastern Mt. Costal


Smallest element
lady Valley Kenya
In each row
A -68 -72 -75 -83 -75 -69 -83
B -56 -60 -58 -63 -61 -59 -63
C -35 -38 -40 -45 -25 -27 -45
D -40 -42 -47 -45 -53 -36 -53
E -62 -70 -68 -66 -69 -70 -70
F -65 -63 -69 -70 -72 -68 -72

Subtract the smallest element from the corresponding rows as given in the tableau 15

Tableau 15

Sales Nyanza Rift Nairobi Eastern Mt. Costal


lady Valley Kenya
A 15 11 8 0 8 14
B 7 3 5 0 2 4
C 10 7 5 0 20 18
D 13 11 6 8 0 17
E 12 0 2 4 1 0
F 7 9 3 2 0 4

Identify the smallest element in each column as given below by the tableau 16
CHAPTER 20 Assignment model 387

Tableau 16

Sales Nyanza Rift Nairobi Eastern Mt. Costal


lady Valley Kenya
A 15 11 8 0 8 14
B 7 3 5 0 2 4
C 10 7 5 0 20 18
D 13 11 6 8 0 17
E 12 0 2 4 1 0
F 7 9 3 2 0 4
7 0 2 0 0 0
Smallest element in each column

Subtract the identified smallest element in each corresponding column to obtain the table
below as given by the table 17

Tableau 17
Sales Nyanza Rift Nairobi Eastern Mt. Costal
lady Valley Kenya
A 8 11 6 0 8 14
B 0 3 3 0 2 4
C 3 7 3 0 20 18
D 6 11 4 8 0 17
E 5 0 0 4 1 0
F 0 9 1 2 0 4

From the above tableau 17 we can try to assign starting with the column with one zero and see
whether we can attain a feasible region. The tableau 18 below shows the assigned elements.
CHAPTER 20 Assignment model 388

Tableau 18
Sales Nyanza Rift Nairobi Eastern Mt. Costal
lady Valley Kenya
A 8 11 6 0 8 14
B 0 3 3 0 2 4
C 3 7 3 0 20 18
D 6 11 4 8 0 17
E 5 0 0 4 1 0
F 0 9 1 2 0 4

In the above Tableau 18 we need to draw line passing through rows and columns that have
zeros and draw as minimal lines as possible as in the tableau 19

Tableau 19
Sales Nyanza Rift Nairobi Eastern Mt. Costal
lady Valley Kenya
A 8 11 6 0 8 14
B 0 3 3 0 2 4
C 3 7 3 0 20 18
D 6 11 4 8 0 17
E 5 0 0 4 1 0
F 0 9 1 2 0 4

Identify the smallest uncovered element which is 1; add 1 to every element covered by two
lines
That is where two are intersecting each other.
CHAPTER 20 Assignment model 389

Tableau 20
Sales Nyanza Rift Nairobi Eastern Mt. Costal
lady Valley Kenya
A 8 11 6 0 8 14
B 0 3 3 0 2 4
C 3 7 3 0 20 18
D 6 11 4 8 0 17
E 5+1 0 0 4+1 1+1 0
F 0 9 1 2 0 4

Subtract 1 to every element that is not covered and the other elements which lies in the line are
supposed to remain untouched as given by the tableau 21

Tableau 21
Sales Nyanza Rift Nairobi Eastern Mt. Costal
lady Valley Kenya
A 8 11-1 6-1 0 8 14-1
B 0 3-1 3-1 0 2 4-1
C 3 7-1 3-1 0 20 18-1
D 6 11-1 4-1 8 0 17-1
E 6 0 0 5 2 0
F 0 9-1 1-1 2 0 4-1

Remember that the elements that lie on the lines where there is no intersection are supposed
to remain untouched and the table on the next page shows the final table after performing the
three operations.

We try to assign and see whether we get an assignment on each row and each column as given
below by the tableau 22, starting with the row or column that has one zero.
CHAPTER 20 Assignment model 390

Tableau 22
Sales Nyanza Rift Nairobi Eastern Mt. Costal
lady Valley Kenya
A 8 10 5 0 8 13
B 0 2 2 0 2 3
C 3 6 2 0 20 17
D 6 10 3 8 0 16
E 6 0 0 5 2 0
F 0 8 0 2 0 3

From the above allocation we can see that when we try to assign zeros in the rows and columns
the solution is no feasible because one row and one column are un assigned and thus we need
to repeat the process of drawing the lines as shown in the tableau 23

Tableau 23
Sales Nyanza Rift Nairobi Eastern Mt. Costal
lady Valley Kenya
A 8 10 5 0 8 13
B 0 2 2 0 2 3
C 3 6 2 0 20 17
D 6 10 3 8 0 16
E 6 0 0 5 2 0
F 0 8 0 2 0 3

By drawing the minimum lines possible identify the smallest element which is uncovered that is
2
Repeating the above procedure add 2 to every element in the intersections of the lines as
shown in the tableau 24
CHAPTER 20 Assignment model 391

Tableau 24
Sales Nyanza Rift Nairobi Eastern Mt. Costal
lady Valley Kenya
A 8 10 5 0 8 13
B 0 2 2 0 2 3
C 3 6 2 0 20 17
D 6 10 3 8 0 16
E 6+2 0 0 5+2 2+2 0
F 0+2 8 0 2+2 0+2 3

Subtract 2 to every element that is not covered to obtain a table as the one given below in the
tableau 25

Tableau 25
Sales Nyanza Rift Nairobi Eastern Mt. Costal
lady Valley Kenya
A 8 10-2 5-2 0 8 13-2
B 0 2-2 2-2 0 2 3-2
C 3 6-2 2-2 0 20 17-2
D 6 10-2 3-2 8 0 16-2
E 8 0 0 7 4 0
F 2 8 0 4 2 3

Now after performing the three operations of adding 2, subtracting 2 and leaving untouched the
element in the lines, we can try to assign where there are zeros and see whether the solution is
feasible. Let examine the tableau 26
CHAPTER 20 Assignment model 392

Start assigning the row or column with one zero and repeat the procedure until every row and
column are assigned.

Tableau 26
Sales Nyanza Rift Nairobi Eastern Mt. Costal
lady Valley Kenya
A 8 8 3 0 8 11
B 0 0 0 0 2 1
C 3 4 0 0 20 15
D 6 8 1 8 0 14
E 8 0 0 7 4 0
F 2 8 0 4 2 3

The above solution is not feasible and we need to redraw the lines again and make sure that the
lines are as minimal as possible that are covering all the zeros that are in the tableau 26. The
tableau 27 below shows the number of lines.

Tableau 27
Sales Nyanza Rift Nairobi Eastern Mt. Costal
lady Valley Kenya
A 8 8 3 0 8 11
B 0 0 0 0 2 1
C 3 4 0 0 20 15
D 6 8 1 8 0 14
E 8 0 0 7 4 0
F 2 8 0 4 2 3

Identify the smallest uncovered element which is 2 and add 2 to where our lines are intersecting
As in the tableau 28 below
CHAPTER 20 Assignment model 393

Tableau 28
Sales Nyanza Rift Nairobi Eastern Mt. Costal
lady Valley Kenya
A 8 8 3 0 8 11
B 0 0 0+2 0+2 2+2 1
C 3 4 0 0 20 15
D 6 8 1 8 0 14
E 8 0 0+2 7+2 4+2 0
F 2 8 0 4 2 3

Subtract 2 from the uncovered elements and elements that are covered except those that are in
the intersection will remain untouched see tableau 29

Tableau 29
Sales Nyanza Rift Nairobi Eastern Mt. Costal
lady Valley Kenya
A 8-2 8-2 3 0 8 11-2
B 0 0 2 2 4 1
C 3-2 4-2 0 0 20 15-2
D 6-2 8-2 1 8 0 14-2
E 8 0 2 9 6 0
F 2-2 8-2 0 4 2 3-2

A summary for the operation is as in the tableau 30 and let try to assign and see whether the
solution is feasible
CHAPTER 20 Assignment model 394

Tableau 30
Sales Nyanza Rift Nairobi Eastern Mt. Costal
lady Valley Kenya
A 6 6 3 0 8 9
B 0 0 2 2 4 1
C 1 2 0 0 20 13
D 6 6 1 8 0 12
E 8 0 2 9 6 0
F 0 6 0 4 2 1

The solution is feasible since when you look at every column and row it is assigned and we need
to allocate as follows

Assign A to Eastern region 83


Assign B to Rift Valley region 60
Assign C to Nairobi region 40
Assign D to Mt Kenya region 53
Assign E to Coast region 70
Assign F to Nyanza region 65
371

Example 20.3
A wood working firm employs six joiners, each joiner has different ability and skills and takes a
different amount of time to do each job at present. There are five joiners to be allocated the
time as given below.

Tableau 31

Job 1 Job 2 Job 3 Job 4 Job 5


Joiner1 25 16 15 14 13
Joiner2 25 17 18 23 15
Joiner3 30 15 20 19 14
Joiner4 27 20 22 25 12
Joiner5 29 19 17 32 10
Joiner6 34 19 24 32 15
CHAPTER 20 Assignment model 395

Required
The joiner has to be assigned one job. How this should be done in order to minimize the total
man time needed to finish all the jobs.

S
Solution

We must establish a dummy job and get the smallest element on each row as given below by
tableau 32

Tableau 32
Job 1 Job 2 Job 3 Job 4 Job 5 Job 6

Joiner1 25 16 15 14 13 0
Joiner2 25 17 18 23 15 0
Joiner3 30 15 20 19 14 0
Joiner4 27 20 22 25 12 0
Joiner5 29 19 17 32 10 0
Joiner6 34 19 24 32 15 0

The smallest values in each row are zero and that will not change our table very much .

We get the smallest value in each column by just picking from the column and write them below
the table as shown by tableau 33

Tableau 33
Job 1 Job 2 Job 3 Job 4 Job 5 Job 6

Joiner1 25 16 15 14 13 0
Joiner2 25 17 18 23 15 0
Joiner3 30 15 20 19 14 0
Joiner4 27 20 22 25 12 0
Joiner5 29 19 17 32 10 0
Joiner6 34 19 24 32 15 0
25 15 15 14 10 0
CHAPTER 20 Assignment model 396

Get the differences from each element in the column by subtracting the difference with the
element corresponding with the columns as in tableau 34

Tableau 34
Job 1 Job 2 Job 3 Job 4 Job 5 Job 6

Joiner1 0 1 0 0 3 0
Joiner2 0 2 3 9 5 0
Joiner3 5 0 5 5 4 0
Joiner4 2 5 7 11 2 0
Joiner5 4 4 2 18 0 0
Joiner6 9 4 9 18 5 0

We can try to assign and see whether we can get a feasible solution as given in the tableau 35

Tableau 35
Job 1 Job 2 Job 3 Job 4 Job 5 Job 6

Joiner1 0 1 0 0 3 0
Joiner2 0 2 3 9 5 0
Joiner3 5 0 5 5 4 0
Joiner4 2 5 7 11 2 0
Joiner5 4 4 2 18 0 0
Joiner6 9 4 9 18 5 0

The solution above is not feasible because one row and column has not been assigned and we
need to introduce lines as given by tableau 36
CHAPTER 20 Assignment model 397

Tableau 36
Job 1 Job 2 Job 3 Job 4 Job 5 Job 6

Joiner1 0 1 0 0 3 0
Joiner2 0 2 3 9 5 0
Joiner3 5 0 5 5 4 0
Joiner4 2 5 7 11 2 0
Joiner5 4 4 2 18 0 0
Joiner6 9 4 9 18 5 0

This has been accomplished by drawing 5 lines making sure that all the zeros are covered as
given in the tableau 36 above

Identify the smallest value which is not covered which in our case is 2, add 2 to where the lines
are intersecting and subtract 2 to the elements which are not covered and let the other
elements which are covered remain the way they are except the operation we have done in the
intersection. See a summary in tableau 37

Tableau 37
Job 1 Job 2 Job 3 Job 4 Job 5 Job 6

Joiner1 0 1 0 0 3 2
Joiner2 0 2 3 9 5 2
Joiner3 5 0 5 5 4 2
Joiner4 0 3 5 9 0 0
Joiner5 4 4 2 18 0 2
Joiner6 7 2 7 16 3 0

Let try assign and see whether we can achieve a feasible solution as given by the tableau 38
CHAPTER 20 Assignment model 398

Tableau 38
Job 1 Job 2 Job 3 Job 4 Job 5 Job 6

Joiner1 0 1 0 0 3 2
Joiner2 0 2 3 9 5 2
Joiner3 5 0 5 5 4 2
Joiner4 0 3 5 9 0 0
Joiner5 4 4 2 18 0 2
Joiner6 7 2 7 16 3 0

The solution is not feasible and we need to repeat the same procedure of drawing as minimal as
possible number of lines as given in the tableau 39 below

Tableau 39
Job 1 Job 2 Job 3 Job 4 Job 5 Job 6

Joiner1 0 1 0 0 3 2
Joiner2 0 2 3 9 5 2
Joiner3 5 0 5 5 4 2
Joiner4 0 3 5 9 0 0
Joiner5 4 4 2 18 0 2
Joiner6 7 2 7 16 3 0

By identifying the smallest element value in our case we have 3 as the smallest element value
from the uncovered elements.
Lets add 3 to where the lines are intersecting, subtract 3 to the elements that are uncovered
and the rest that lie along the lines remain untouched. See tableau 40
CHAPTER 20 Assignment model 399

Tableau 40
Job 1 Job 2 Job 3 Job 4 Job 5 Job 6

Joiner1 3 4 0 0 3 5
Joiner2 0 2 1 6 2 2
Joiner3 5 0 2 2 1 2
Joiner4 3 6 5 9 0 3
Joiner5 7 7 2 18 0 5
Joiner6 7 2 4 13 0 0

Let’s try to assign and see whether we can get a feasible solution as shown in the tableau 41

Tableau 41
Job 1 Job 2 Job 3 Job 4 Job 5 Job 6

Joiner1 3 4 0 0 3 5
Joiner2 0 2 1 6 2 2
Joiner3 5 0 2 2 1 2
Joiner4 3 6 5 9 0 3
Joiner5 7 7 2 18 0 5
Joiner6 7 2 4 13 0 0

The allocation is not feasible and we required again repeating the same process until we can
achieve an optimal solution and we begin by drawing lines as minimal as possible as given by
the tableau 42
CHAPTER 20 Assignment model 400

Tableau 42
Job 1 Job 2 Job 3 Job 4 Job 5 Job 6

Joiner1 3 4 0 0 3 5
Joiner2 0 2 1 6 2 2
Joiner3 5 0 2 2 1 2
Joiner4 3 6 5 9 0 3
Joiner5 7 7 2 18 0 5
Joiner6 7 2 4 13 0 0

Identify the smallest element value that is not covered that is 1, add 1 to where the lines are
intersecting and subtract 1 to the elements that are not covered and leave the rest that are
lying along the lines untouched.
See the tableau 43

Tableau43
Job 1 Job 2 Job 3 Job 4 Job 5 Job 6

Joiner1 4 5 0 0 4 5
Joiner2 0 2 0 5 2 1
Joiner3 5 0 1 1 1 1
Joiner4 3 6 4 8 0 2
Joiner5 7 7 1 17 0 4
Joiner6 8 3 4 13 1 0

Assign the to see whether the solution is feasible and remember to assign only one zero at
every column and row and when there are more than one zero in the column close the rest
remaining zeros to avoiding assign more than time in each column.
See tableau 44
CHAPTER 20 Assignment model 401

Tableau 44
Job 1 Job 2 Job 3 Job 4 Job 5 Job 6

Joiner1 4 5 0 0 4 5
Joiner2 0 2 0 5 2 1
Joiner3 5 0 1 1 1 1
Joiner4 3 6 4 8 0 2
Joiner5 7 7 1 17 0 4
Joiner6 8 3 4 13 1 0

Again our allocation is no feasible and we need to identify the number of lines that covers our
zeros as shown in the tableau 45 below

Tableau 44
Job 1 Job 2 Job 3 Job 4 Job 5 Job 6

Joiner1 4 5 0 0 4 5
Joiner2 0 2 0 5 2 1
Joiner3 5 0 1 1 1 1
Joiner4 3 6 4 8 0 2
Joiner5 7 7 1 17 0 4
Joiner6 8 3 4 13 1 0

The smallest element value is 1 and where there is intersection we need to add 1 and subtract 1
where there are elements that are no covered and leave the rest an touched as we are familiar
with these procedure as summary is given below.
See tableau 45
CHAPTER 20 Assignment model 402

Tableau 45
Job 1 Job 2 Job 3 Job 4 Job 5 Job 6

Joiner1 4 6 0 0 5 6
Joiner2 0 3 0 5 3 2
Joiner3 4 0 0 0 1 1
Joiner4 2 6 3 7 0 2
Joiner5 6 7 0 16 0 4
Joiner6 7 3 3 12 1 0

The solution above in tableau 45 is a feasible one because every row and column have been
assigned and thus we need make the assignment to joiners to see which job they are going to
take.

Assign as shown below


Assign to joiner 1 25
Assign to joiner 2 15
Assign to joiner 3 17
Assign to joiner 4 14
Assign to joiner 5 12
Assign to joiner 6 0
83

Practice Problem

1. A company has four distribution depots and four orders to be delivered to separate
customers. Each depot has one lorry available which is large enough to carry one of
these orders. The distance between each depot and each customer is given below.
CHAPTER 20 Assignment model 403

Distance in kilometers

Depot Customers

Alvin Faith Brain Jane

A 68 72 75 83

B 56 60 58 63

C 38 40 35 45

D 47 42 40 45

a) How should be the order assigned to the depot in order to minimize the total
distance traveled, use Hungarian algorithm.
b) Calculate the minimum distance to be covered.

2. An electricity company has four sub-stations that need to be conducted. Five


construction companies have quoted for the jobs. The quoted costs in millions of
shillings are in table below

Firms Sub-stations

A B C D

DTL 5 6 8 10

KDL 18 14 12 16

EXL 10 13 14 12

ZAP 13 10 10 16

MINT 14 11 11 10

a) What assignment should be made to minimize construction cost?


b) Determine the minimum cost of constructing the sub-stations
c) State the construction firm that won’t be assigned any job
CHAPTER 20 Assignment model 404

3. Duport Ltd has 6 sales region and 6 sales representatives. From past experience it is
known that the sales representatives perform differently in the different areas. The
company’s. Sales Director has estimated from each sales person in each area. These are
given in the table below.

Sales Sales in KES ‘000’


Representatives I II III IV V VI
A 680 720 750 830 750 690
B 580 600 580 630 610 590
C 350 380 400 450 250 270
D 400 420 470 450 530 360
E 620 700 680 670 690 700
F 650 630 690 690 720 680

How should the Sales Director assign the salesmen to the areas to maximize total sales?

4. Explain the difference between transportation and assignment models.

5. The wood working firm, Crestwood Ltd, employs six joiners. Each man has different
abilities and skills and takes a different amount of time to do each job. At present,
there are five jobs to be allocated. The times are given below:

Time per job (hours)

Job 1 Job 2 Job3 Job 4 Job 5

Man 1 25 16 15 14 13

Man 2 25 17 18 23 15

Man 3 30 15 20 19 14

Man 4 27 20 22 25 12

Man 5 29 19 17 32 10

a) The jobs have to be assigned one job to one man. How should this be done in
order to minimize the total man-time needed to finish all of the jobs?

b) Calculate the total time that they will take.


CHAPTER 20 Assignment model 401

6. ANU Computer Information System (CIS) has five expert programmers. The CIS needs
five application programs to be developed. The head of the computer information
system, after studying the programs to be developed, estimates the computer time in
minutes by the experts for the application programs as follows.

Programmers

Abel Ben Calf Dave Evan

A 160 130 175 190 200

B 135 120 130 160 175

Programs C 140 110 155 170 185

D 50 50 80 80 150

E 55 35 70 80 105

How would you assign the programs to the programmers in order to attain minimum
total computer time?
21
INVENTORY PLANNING &
CHAPTER CONTROL

Objectives

At the end of this chapter the student should be able to do the following:

 Understand the importance of inventory control in modern business


world
 Use the economic order quantity(EOQ) to determine how much to
order
 Understand the basic stock model

Introduction

One of the consequences of changes in the economic climate is that manufacturing companies
have hard to review their policies to wants the holding and control of stocks that is both raw
materials and finished goods.
When a company that is holding stock of goods capital is tied up in the goods. This unusable
capital represents a cost to the company in the form of lost interest or investment
opportunities.
In addition the stock held will incur cost inform of storage chart personnel employed to manage
it, insurance etc.
A stock model is therefore necessary for any for any organization to help the decision makers to
determine how much to order and in production how much to produce.
The objective of the decision as to minimize the total cost associated with the holding of the
stock.
CHAPTER 21 Inventory planning and control 407

Basic stock model

Figure 1

Order size
Re-order
Stock level
level

Lead time
Stock out

In any stock system the stock level will vary over time in cyclical patterns. The pattern of the
demand will dictate how the stock level drop at same point an order will be passed to replenish
the stock after an erupt of time known as the lead time the order will arrive and stock level will
instantly increase and now stock circle will begin.

Assumptions of EOQ

Economic order quantity (EOQ) being one of the oldest and the most commonly known
inventory techniques is still in use to today because it is easy to use but however it has several
assumptions.
1) The demand for the item is constant or approximately constant if the rate of usage is
constant.
2) The lead time also known and is constant
3) Receipt of inventory is instantaneous.
4) Cost per unit is constant throughout the year
5) The only variable cost is the cost placing an order that is ordering cost.
6) Orders are placed so that there are no shortages that are no stock outs permitted
CHAPTER 21 Inventory planning and control 408

Figure 2
Order size

Re-order
Stock level
level

Time
Lead time

Annual cost of ordering

D
If D items are required per year and they are ordered in batches of (q) then orders will
q
needed each year.
Annual cost of ordering = Cost of placing order × number of orders placed
The cost of placing order is given by Co
Thus the annual cost of ordering will be expressed as follows

= Co × D
Annual cost of ordering
q

Annual cost of holding the stock

This cost is based on the average amount held in stock over a single stock cycle. In simple
situation the stock level varies linearly from (q) to zero and therefore the average stock is
expressed as follows.

q+0 q
Average stock
= =
2 2
Hence the annual cost of hold the stock = Cost of holding unit for a year × Average stock held
CHAPTER 21 Inventory planning and control 409

= Ch × q
Annual cost of holding the stock
2

Total cost in EOQ Model

The total cost is given by adding all the costs and the costs that we have are the annual cost
ordering and annual cost of holding the stock.

Total cost = annual cost of ordering + annual cost of holding the stock
= Co × D
Given that annual cost of ordering
q
and
= Ch × q
annual cost of holding the stock
2
We can express the total cost as follows

TC = Co × D + Ch × q
q 2
CoD Chq
TC
= +
q 2

Example 21.1

The optimum order quantity


Let’s take D = annual demand
O = order quantity
Co = cost of ordering for one order
Ch = cost of holding one item per year
q
Hence Average stock =
2
Chq
Annual cost of holding stock =
2
D
Number of order per year =
q
CoD
Annual ordering cost =
q
CHAPTER 21 Inventory planning and control 410

CoD Chq
Total cost =
TC +
q 2

The order quantity at minimum can bbe obtained by getting the derivative of the total cost in
respect to the stock (q)
CoD Chq
=TC +
q 2
dTC 1
= CoDq −1 + Chq
dq 2
1
= CoDq −1−1 + Chq1−1
2
1
= CoDq −2 + Chq 0
2
CoD Ch
= +
q2 2

dTC
When TC takes its minimum value when = 0 or when the second derivative is negative that
dq
d 2TC
is less than zero => 0
dq 2
Let us equate our derivative to its minimum that is zero.
CoD Ch
+ = 0
q2 2
And using the above we can the value of (q) or in other words making (q) the subject of the
above equation.
CoD Ch
=
q2 2
2
Cross multiply q by Cho and 2 by CoD

2CoD = q 2Ch
CHAPTER 21 Inventory planning and control 411

2CoD
= q2
Ch
2CoD
= q2
Ch
2.Co.D
q=
Ch

This optimum order quantity is known as the economic order quantity (EOQ). If this quantity is
ordered at regularly intervals through the year then the total cost of holding stocks will take its
minimum value.

2.Co.D
EOQ or (qo ) =
Ch

Example 21.2

A supermarket sells 500, 000 bars of soap annually the demand is spread evenly over the year.
The purchase for each bar is KES 40 .It cost the proprietor KES 20, 000 to place an order .the
supplier lead time is 12 working days.
The holding cost is to be 20% per year of the average stock value.

a) How many bars should be ordered at time if he wishes to minimize his total stocking
cost.
b) Determining how often the order should be placed and re-order level.
c) Assume that the shop is open for 300 days a year and a week is 6 days.

Solution
Let D = 500, 000

Co = 20, 000

= 20
Ch × 40
= 8
100
CHAPTER 21 Inventory planning and control 412

2CoD
EOQ =
Ch
2 × 20, 000 × 50, 000
=
8
= 2,500, 000, 000
= 50, 000

Taking D = 500,000 and since we have already gotten our q = 50, 000 we can get how often he
can order in a given year.

D 500, 000
= = 10 Times
q 50, 000

Within a year he should order 10 times and the shop operates for 300 days a year and thus we
can calculate the duration that the shops before it orders.

300
= 30 Days
10

Economic Batch Quantity

Company’s which produce a number of different lines may organize their production on batch
bases rather than questions one e.g. coke may decide to make coke in the morning, sprite in the
middle morning and Fanta in the afternoon.
When the batch production is used the company has to decide how large a batch to make at
time and how often to make a batch of particular product the problem is similar to the
economic order quantity.
The ordering of a fixed quantity from an outside supplier is replaced by the [production of a
fixed amount the ordering cost is therefore replaced by the set up costs (Cs).

D q
TC Cs
= + Ch
q 2

Where (q) is the batch size


CHAPTER 21 Inventory planning and control 413

2.Cs.D
EBQ or (qo ) =
Ch

Example 21.3

XYZ produces several brands of soft drinks these soft drinks are made on a batch production
bases at rate of 50, 000 bottle per day the demand for the most popular soft drink coke is 30,
000, 000 per year spread evenly offer the year.
Whenever a batch of coke is to be made a set up cost of KES 1, 000, 000 are incurred the
company estimate that the annual cost of holding this soft drink is KES 7.50 per bottle.
Assuming that the information given concern 300ml bottle only
How many bottles should they produce in batch if they wish to minimize the total annual cost of
production and holding?
How often should there be a production to run and how long will it last.
Assume the year has 300 days

Solution
Considering the following given information D = 30, 000, 000

Cs = 1, 000, 000

Ch = 7.50

2.Cs.D
EBQ or (qo ) =
Ch

2 ×1,.000, 000 × 30, 000, 0000


=
7.5

= 2,828, 428

D q
TC Cs
= + Ch
q 2
CHAPTER 21 Inventory planning and control 414
1, 000, 000 × 30, 000, 000 7.5 × 2,828, 428
= +
2,828, 428 2

= 106, 066.02 + 106, 07

= 21, 213, 793

D 30, 000, 000


= = 11 Times
q 2,828, 428

q
How long will it last =
Daily demand

2,828, 428
= = 56.7
50, 000

In the situation examined so far it has assumed that the stock count is not permitted. There may
be occasion in which it is cheaper to go out of stock than to carry the size of stock needed to
avoid stock count.
Suppose a machine produces a batch of component some of which pass directly to slower
machine for immediate use the rest on stock until needed by second machine.
Instead of arriving all together and stock holding jumping firm (0 – q) the stock increase steady
during the time for which the first machines is producing and then decreases as the second
machine uses up the stock let the rate of production be P and usage be D P ≥ D

The stock build up rate is (P –D)

If the production cycle last for a year then the total quantity produced during a cycle is given by
Pt
q = Pt
Batch produced

The maximum stock level we can achieve is (P – D)t


q
t=
P
D (P - D)q
TC = Cs + Ch
q 2P
CHAPTER 21 Inventory planning and control 415

When we differentiate the above equation at its minimum we will get the following equation

D (P - D)q
TC = Cs + Ch
q 2P

dTC −1−1 (P - D)q1−1


−2CsDq + Ch
=
dq 2P
dTC
=0
dq
2CsD (P - D)
− 2 + Ch
= 0
=
q 2P
(P - D) 2CsD
= Ch
=
2P q2

2CsD. P
q=
Ch (P - D)

Example 21.4

A machine manufactures spare parts at the rate of 20, 000 per month. A second machine uses
those spare parts at the rate 50, 000 per month and remainder put into stock.
It cost KES100, 000 to set up the machine the company establish their stock hold costs 20% per
annum of the average stock value each parts costs KES 250 to make.
Required
What batch size should be produced on the first machine and what frequency.
Calculate the total variable cost of production
If the set up cost could be reduced to KES 25, 000 how mould changes affects answer (i) and (ii)

Solution
Taking Cs = 100, 000

D = 5 000 × 12 = 60, 000

P = 20, 000 × 12 = 240, 000


CHAPTER 21 Inventory planning and control 416

Ch = 250 × 20% = 50

2(100, 000)(60, 000 × 24, 000


=
50

20, 000, 000, 000, 000


=
750, 000

= 26, 666, 666.7

= 17,889

Frequency

17,889 1
= 3 months
60, 000 2

D (P - D)q
TC = Cs + Ch
q 2P
60, 000 50(240, 000 − 60, 0000
100, 000 × +
17,889 2 × 240, 000
335, 402 + 18.75

= 335, 420.75
CHAPTER 21 Inventory planning and control 417

The planned shortage model

2CoD  Ch + Cb 
q=  
Ch  Cb 

Cb is the cost of being out of stock for one year

Example 21.5
Supper electronic ltd an independently tailor of electronic and audio equipment One of their
most popular iPod the demand is for 2000 a year spreading evenly over the year. These items
cost the company KES 25, 000 to buy directly from the manufacturer the cost of placing an
order is KES 125, 000 and the cost of holding the iPods in stock is charge at 15% per year of the
average stock value.
The manager is considering the stocks held of these items in order to help improve customer
cash flow.
He has established that the cost of administering an out of stock ordering system together with
a charge for many lost sales and loss of good will amount KES 2000 per iPod per year.

a) Determine the value of total variable cost of stock the iPod if stock out are not
permitted.
b) How much could be saved if a system of planned shortages was introduced

Solution

Given that
CHAPTER 21 Inventory planning and control 418
D = 2000
Co = 125, 000

Ch = 15 ×125, 000 =
3750
100

2CoD
EOQ = q =
Ch

2 ×125, 000 × 2000


=
3750
= 365.1

D q
TC = Co + Ch
q 2

 2000   365.1 
= 125, 000 ×  +  3750 × 
 365.1   2 
= 684743.9 + 684562.5

= 1,369,306.4

Optimum order quantity

2CoD  Ch+Cb 
q =  
Ch  Cb 

2 ×125,000 × 2000  3750+2000 


q =  
3750  2000 

= 133,333.333 × 2.875

= 383,332
CHAPTER 21 Inventory planning and control 419
= 619

22
D  q − s  CbS
TC =
Co + Ch   +
Ch  Cb  2q

2CoD  Ch 
S is given by:  
Ch  Ch + Cb 
2 2
D  q - s  CbS
TC = Co + Ch   +
Ch  Cb  2q

2CoD  Ch 
S is given by  
Ch  Ch + Cb 

2 ×125, 000 × 2000  3750 


=   294.88
3750  3750 + 200 
therefore

( 619 − 295) + 2000 ( 295)


2 2
125, 000 × 2000
TC = + 3750
619 2 × 619 2 × 619
= 862, 447.50
= 1,369,306.4 − 862, 442.50 = 506,858.90
CHAPTER 21 Inventory planning and control 420

Practice Problem

1. Telfix Ltd is a mobile phone company. The demand for the phone is
12,500 per annum. Telfix buy the phones for KES 900 each. The ordering
costs are KES 15,000 per order and the stock costs are KES 30 per phone
per annum.
a) Calculate the optimum order quantity.
b) Currently the company is ordering in batches of 300 phones.
How much will they save each year if they change to the order
quantity found in part (a)

2. Chan electronic ltd an independently tailor of electronic and audio


equipment. One of their most popular iPod the demand is for 4000 a
year spreading evenly over the year. These items cost the company KES
50, 000 to buy directly from the manufacturer the cost of placing an
order is KES 250, 000 and the cost of holding the iPods in stock is charge
at 15% per year of the average stock value. The manager is considering
the stocks held of these items in order to help improve customer cash
flow.
He has established that the cost of administering an out of stock
ordering system together with a charge for many lost sales and loss of
good will amount KES 4000 per iPod per year.

a) Determine the value of total variable cost of stock the iPod if


stock out are not permitted.
b) How much could be saved if a system of planned shortages was
introduced.

3. Megafix Ltd retails Kodax camera in Kenya. On its line is Kodax 10.0
special. The demand for the camera is 6 per week. Megafix buys the
camera for KES 20,000 each. The ordering costs are KES 1.2 million per
order and the stockholding costs are KES 1000 per camera of average
stock per year plus 15% per annum of the average stock value. Assume
50 weeks per year.
a) What is the optimum order quantity
b) Currently Megafix Ltd is ordering in batches of 500 cameras.
How much money will they save each year if they change to
order quantity found in (a) above.
APPENDICES 421

Standard Normal Cumulative Probability Table


Z
Cumulative probabilities for NEGATIVE z-values are shown in the following table:

z 0.00 0.01 0.02 0.03 0.04 0.05 0.06 0.07 0.08 0.09
-3.4 0.0003 0.0003 0.0003 0.0003 0.0003 0.0003 0.0003 0.0003 0.0003 0.0002
-3.3 0.0005 0.0005 0.0005 0.0004 0.0004 0.0004 0.0004 0.0004 0.0004 0.0003
-3.2 0.0007 0.0007 0.0006 0.0006 0.0006 0.0006 0.0006 0.0005 0.0005 0.0005
-3.1 0.0010 0.0009 0.0009 0.0009 0.0008 0.0008 0.0008 0.0008 0.0007 0.0007
-3.0 0.0013 0.0013 0.0013 0.0012 0.0012 0.0011 0.0011 0.0011 0.0010 0.0010
-2.9 0.0019 0.0018 0.0018 0.0017 0.0016 0.0016 0.0015 0.0015 0.0014 0.0014
-2.8 0.0026 0.0025 0.0024 0.0023 0.0023 0.0022 0.0021 0.0021 0.0020 0.0019
-2.7 0.0035 0.0034 0.0033 0.0032 0.0031 0.0030 0.0029 0.0028 0.0027 0.0026
-2.6 0.0047 0.0045 0.0044 0.0043 0.0041 0.0040 0.0039 0.0038 0.0037 0.0036
-2.5 0.0062 0.0060 0.0059 0.0057 0.0055 0.0054 0.0052 0.0051 0.0049 0.0048
-2.4 0.0082 0.0080 0.0078 0.0075 0.0073 0.0071 0.0069 0.0068 0.0066 0.0064
-2.3 0.0107 0.0104 0.0102 0.0099 0.0096 0.0094 0.0091 0.0089 0.0087 0.0084
-2.2 0.0139 0.0136 0.0132 0.0129 0.0125 0.0122 0.0119 0.0116 0.0113 0.0110
-2.1 0.0179 0.0174 0.0170 0.0166 0.0162 0.0158 0.0154 0.0150 0.0146 0.0143
-2.0 0.0228 0.0222 0.0217 0.0212 0.0207 0.0202 0.0197 0.0192 0.0188 0.0183
-1.9 0.0287 0.0281 0.0274 0.0268 0.0262 0.0256 0.0250 0.0244 0.0239 0.0233
-1.8 0.0359 0.0351 0.0344 0.0336 0.0329 0.0322 0.0314 0.0307 0.0301 0.0294
-1.7 0.0446 0.0436 0.0427 0.0418 0.0409 0.0401 0.0392 0.0384 0.0375 0.0367
-1.6 0.0548 0.0537 0.0526 0.0516 0.0505 0.0495 0.0485 0.0475 0.0465 0.0455
-1.5 0.0668 0.0655 0.0643 0.0630 0.0618 0.0606 0.0594 0.0582 0.0571 0.0559
-1.4 0.0808 0.0793 0.0778 0.0764 0.0749 0.0735 0.0721 0.0708 0.0694 0.0681
-1.3 0.0968 0.0951 0.0934 0.0918 0.0901 0.0885 0.0869 0.0853 0.0838 0.0823
-1.2 0.1151 0.1131 0.1112 0.1093 0.1075 0.1056 0.1038 0.1020 0.1003 0.0985
-1.1 0.1357 0.1335 0.1314 0.1292 0.1271 0.1251 0.1230 0.1210 0.1190 0.1170
-1.0 0.1587 0.1562 0.1539 0.1515 0.1492 0.1469 0.1446 0.1423 0.1401 0.1379
-0.9 0.1841 0.1814 0.1788 0.1762 0.1736 0.1711 0.1685 0.1660 0.1635 0.1611
-0.8 0.2119 0.2090 0.2061 0.2033 0.2005 0.1977 0.1949 0.1922 0.1894 0.1867
-0.7 0.2420 0.2389 0.2358 0.2327 0.2296 0.2266 0.2236 0.2206 0.2177 0.2148
-0.6 0.2743 0.2709 0.2676 0.2643 0.2611 0.2578 0.2546 0.2514 0.2483 0.2451
-0.5 0.3085 0.3050 0.3015 0.2981 0.2946 0.2912 0.2877 0.2843 0.2810 0.2776
-0.4 0.3446 0.3409 0.3372 0.3336 0.3300 0.3264 0.3228 0.3192 0.3156 0.3121
-0.3 0.3821 0.3783 0.3745 0.3707 0.3669 0.3632 0.3594 0.3557 0.3520 0.3483
-0.2 0.4207 0.4168 0.4129 0.4090 0.4052 0.4013 0.3974 0.3936 0.3897 0.3859
-0.1 0.4602 0.4562 0.4522 0.4483 0.4443 0.4404 0.4364 0.4325 0.4286 0.4247
0.0 0.5000 0.4960 0.4920 0.4880 0.4840 0.4801 0.4761 0.4721 0.4681 0.4641
APPENDICES 422
Standard Normal Cumulative Probability Table
Cumulative probabilities for POSITIVE z-values are shown in the following
table: Z
z 0.00 0.01 0.02 0.03 0.04 0.05 0.06 0.07 0.08 0.09
0.0 0.5000 0.5040 0.5080 0.5120 0.5160 0.5199 0.5239 0.5279 0. 5319 0.5359
0.1 0.5398 0.5438 0.5478 0.5517 0.5557 0.5596 0.5636 0.5675 0.5714 0.5753
0.2 0.5793 0.5832 0.5871 0.5910 0.5948 0.5987 0.6026 0.6064 0.6103 0.6141
0.3 0.6179 0.6217 0.6255 0.6293 0.6331 0.6368 0.6406 0.6443 0.6480 0.6517
0.4 0.6554 0.6591 0.6628 0.6664 0.6700 0.6736 0.6772 0.6808 0.6844 0.6879
0.5 0.6915 0.6950 0.6985 0.7019 0.7054 0.7088 0.7123 0.7157 0.7190 0.7224
0.6 0.7257 0.7291 0.7324 0.7357 0.7389 0.7422 0.7454 0.7486 0.7517 0.7549
0.7 0.7580 0.7611 0.7642 0.7673 0.7704 0.7734 0.7764 0.7794 0.7823 0.7852
0.8 0.7881 0.7910 0.7939 0.7967 0.7995 0.8023 0.8051 0.8078 0.8106 0.8133
0.9 0.8159 0.8186 0.8212 0.8238 0.8264 0.8289 0.8315 0.8340 0.8365 0.8389
1.0 0.8413 0.8438 0.8461 0.8485 0.8508 0.8531 0.8554 0.8577 0.8599 0.8621
1.1 0.8643 0.8665 0.8686 0.8708 0.8729 0.8749 0.8770 0.8790 0.8810 0.8830
1.2 0.8849 0.8869 0.8888 0.8907 0.8925 0.8944 0.8962 0.8980 0.8997 0.9015
1.3 0.9032 0.9049 0.9066 0.9082 0.9099 0.9115 0.9131 0.9147 0.9162 0.9177
1.4 0.9192 0.9207 0.9222 0.9236 0.9251 0.9265 0.9279 0.9292 0.9306 0.9319
1.5 0.9332 0.9345 0.9357 0.9370 0.9382 0.9394 0.9406 0.9418 0.9429 0.9441
1.6 0.9452 0.9463 0.9474 0.9484 0.9495 0.9505 0.9515 0.9525 0.9535 0.9545
1.7 0.9554 0.9564 0.9573 0.9582 0.9591 0.9599 0.9608 0.9616 0.9625 0.9633
1.8 0.9641 0.9649 0.9656 0.9664 0.9671 0.9678 0.9686 0.9693 0.9699 0.9706
1.9 0.9713 0.9719 0.9726 0.9732 0.9738 0.9744 0.9750 0.9756 0.9761 0.9767
2.0 0.9772 0.9778 0.9783 0.9788 0.9793 0.9798 0.9803 0.9808 0.9812 0.9817
2.1 0.9821 0.9826 0.9830 0.9834 0.9838 0.9842 0.9846 0.9850 0.9854 0.9857
2.2 0.9861 0.9864 0.9868 0.9871 0.9875 0.9878 0.9881 0.9884 0.9887 0.9890
2.3 0.9893 0.9896 0.9898 0.9901 0.9904 0.9906 0.9909 0.9911 0.9913 0.9916
2.4 0.9918 0.9920 0.9922 0.9925 0.9927 0.9929 0.9931 0.9932 0.9934 0.9936
2.5 0.9938 0.9940 0.9941 0.9943 0.9945 0.9946 0.9948 0.9949 0.9951 0.9952
2.6 0.9953 0.9955 0.9956 0.9957 0.9959 0.9960 0.9961 0.9962 0.9963 0.9964
2.7 0.9965 0.9966 0.9967 0.9968 0.9969 0.9970 0.9971 0.9972 0.9973 0.9974
2.8 0.9974 0.9975 0.9976 0.9977 0.9977 0.9978 0.9979 0.9979 0.9980 0.9981
2.9 0.9981 0.9982 0.9982 0.9983 0.9984 0.9984 0.9985 0.9985 0.9986 0.9986
3.0 0.9987 0.9987 0.9987 0.9988 0.9988 0.9989 0.9989 0.9989 0.9990 0.9990
3.1 0.9990 0.9991 0.9991 0.9991 0.9992 0.9992 0.9992 0.9992 0.9993 0.9993
3.2 0.9993 0.9993 0.9994 0.9994 0.9994 0.9994 0.9994 0.9995 0.9995 0.9995
3.3 0.9995 0.9995 0.9995 0.9996 0.9996 0.9996 0.9996 0.9996 0.9996 0.9997
3.4 0.9997 0.9997 0.9997 0.9997 0.9997 0.9997 0.9997 0.9997 0.9997 0.9998
APPENDICES 423

Standard Normal Distribution: Table Values Represent AREA to the LEFT of the Z score.

Z .00 .01 .02 .03 .04 .05 .06 .07 .08 .09
-3.9 .00005 .00005 .00004 .00004 .00004 .00004 .00004 .00004 .00003 .00003
-3.8 .00007 .00007 .00007 .00006 .00006 .00006 .00006 .00005 .00005 .00005
-3.7 .00011 .00010 .00010 .00010 .00009 .00009 .00008 .00008 .00008 .00008
-3.6 .00016 .00015 .00015 .00014 .00014 .00013 .00013 .00012 .00012 .00011
-3.5 .00023 .00022 .00022 .00021 .00020 .00019 .00019 .00018 .00017 .00017
-3.4 .00034 .00032 .00031 .00030 .00029 .00028 .00027 .00026 .00025 .00024
-3.3 .00048 .00047 .00045 .00043 .00042 .00040 .00039 .00038 .00036 .00035
-3.2 .00069 .00066 .00064 .00062 .00060 .00058 .00056 .00054 .00052 .00050
-3.1 .00097 .00094 .00090 .00087 .00084 .00082 .00079 .00076 .00074 .00071
-3.0 .00135 .00131 .00126 .00122 .00118 .00114 .00111 .00107 .00104 .00100
-2.9 .00187 .00181 .00175 .00169 .00164 .00159 .00154 .00149 .00144 .00139
-2.8 .00256 .00248 .00240 .00233 .00226 .00219 .00212 .00205 .00199 .00193
-2.7 .00347 .00336 .00326 .00317 .00307 .00298 .00289 .00280 .00272 .00264
-2.6 .00466 .00453 .00440 .00427 .00415 .00402 .00391 .00379 .00368 .00357
-2.5 .00621 .00604 .00587 .00570 .00554 .00539 .00523 .00508 .00494 .00480
-2.4 .00820 .00798 .00776 .00755 .00734 .00714 .00695 .00676 .00657 .00639
-2.3 .01072 .01044 .01017 .00990 .00964 .00939 .00914 .00889 .00866 .00842
-2.2 .01390 .01355 .01321 .01287 .01255 .01222 .01191 .01160 .01130 .01101
-2.1 .01786 .01743 .01700 .01659 .01618 .01578 .01539 .01500 .01463 .01426
-2.0 .02275 .02222 .02169 .02118 .02068 .02018 .01970 .01923 .01876 .01831
-1.9 .02872 .02807 .02743 .02680 .02619 .02559 .02500 .02442 .02385 .02330
-1.8 .03593 .03515 .03438 .03362 .03288 .03216 .03144 .03074 .03005 .02938
-1.7 .04457 .04363 .04272 .04182 .04093 .04006 .03920 .03836 .03754 .03673
-1.6 .05480 .05370 .05262 .05155 .05050 .04947 .04846 .04746 .04648 .04551
-1.5 .06681 .06552 .06426 .06301 .06178 .06057 .05938 .05821 .05705 .05592
-1.4 .08076 .07927 .07780 .07636 .07493 .07353 .07215 .07078 .06944 .06811
-1.3 .09680 .09510 .09342 .09176 .09012 .08851 .08691 .08534 .08379 .08226
-1.2 .11507 .11314 .11123 .10935 .10749 .10565 .10383 .10204 .10027 .09853
-1.1 .13567 .13350 .13136 .12924 .12714 .12507 .12302 .12100 .11900 .11702
-1.0 .15866 .15625 .15386 .15151 .14917 .14686 .14457 .14231 .14007 .13786
-0.9 .18406 .18141 .17879 .17619 .17361 .17106 .16853 .16602 .16354 .16109
-0.8 .21186 .20897 .20611 .20327 .20045 .19766 .19489 .19215 .18943 .18673
-0.7 .24196 .23885 .23576 .23270 .22965 .22663 .22363 .22065 .21770 .21476
-0.6 .27425 .27093 .26763 .26435 .26109 .25785 .25463 .25143 .24825 .24510
-0.5 .30854 .30503 .30153 .29806 .29460 .29116 .28774 .28434 .28096 .27760
-0.4 .34458 .34090 .33724 .33360 .32997 .32636 .32276 .31918 .31561 .31207
-0.3 .38209 .37828 .37448 .37070 .36693 .36317 .35942 .35569 .35197 .34827
-0.2 .42074 .41683 .41294 .40905 .40517 .40129 .39743 .39358 .38974 .38591
-0.1 .46017 .45620 .45224 .44828 .44433 .44038 .43644 .43251 .42858 .42465
-0.0 .50000 .49601 .49202 .48803 .48405 .48006 .47608 .47210 .46812 .46414
APPENDICES 424
Standard Normal Distribution: Table Values Represent AREA to the LEFT of the Z score.
Z .00 .01 .02 .03 .04 .05 .06 .07 .08 .09
0.0 .50000 .50399 .50798 .51197 .51595 .51994 .52392 .52790 .53188 .53586
0.1 .53983 .54380 .54776 .55172 .55567 .55962 .56356 .56749 .57142 .57535
0.2 .57926 .58317 .58706 .59095 .59483 .59871 .60257 .60642 .61026 .61409
0.3 .61791 .62172 .62552 .62930 .63307 .63683 .64058 .64431 .64803 .65173
0.4 .65542 .65910 .66276 .66640 .67003 .67364 .67724 .68082 .68439 .68793
0.5 .69146 .69497 .69847 .70194 .70540 .70884 .71226 .71566 .71904 .72240
0.6 .72575 .72907 .73237 .73565 .73891 .74215 .74537 .74857 .75175 .75490
0.7 .75804 .76115 .76424 .76730 .77035 .77337 .77637 .77935 .78230 .78524
0.8 .78814 .79103 .79389 .79673 .79955 .80234 .80511 .80785 .81057 .81327
0.9 .81594 .81859 .82121 .82381 .82639 .82894 .83147 .83398 .83646 .83891
1.0 .84134 .84375 .84614 .84849 .85083 .85314 .85543 .85769 .85993 .86214
1.1 .86433 .86650 .86864 .87076 .87286 .87493 .87698 .87900 .88100 .88298
1.2 .88493 .88686 .88877 .89065 .89251 .89435 .89617 .89796 .89973 .90147
1.3 .90320 .90490 .90658 .90824 .90988 .91149 .91309 .91466 .91621 .91774
1.4 .91924 .92073 .92220 .92364 .92507 .92647 .92785 .92922 .93056 .93189
1.5 .93319 .93448 .93574 .93699 .93822 .93943 .94062 .94179 .94295 .94408
1.6 .94520 .94630 .94738 .94845 .94950 .95053 .95154 .95254 .95352 .95449
1.7 .95543 .95637 .95728 .95818 .95907 .95994 .96080 .96164 .96246 .96327
1.8 .96407 .96485 .96562 .96638 .96712 .96784 .96856 .96926 .96995 .97062
1.9 .97128 .97193 .97257 .97320 .97381 .97441 .97500 .97558 .97615 .97670
2.0 .97725 .97778 .97831 .97882 .97932 .97982 .98030 .98077 .98124 .98169
2.1 .98214 .98257 .98300 .98341 .98382 .98422 .98461 .98500 .98537 .98574
2.2 .98610 .98645 .98679 .98713 .98745 .98778 .98809 .98840 .98870 .98899
2.3 .98928 .98956 .98983 .99010 .99036 .99061 .99086 .99111 .99134 .99158
2.4 .99180 .99202 .99224 .99245 .99266 .99286 .99305 .99324 .99343 .99361
2.5 .99379 .99396 .99413 .99430 .99446 .99461 .99477 .99492 .99506 .99520
2.6 .99534 .99547 .99560 .99573 .99585 .99598 .99609 .99621 .99632 .99643
2.7 .99653 .99664 .99674 .99683 .99693 .99702 .99711 .99720 .99728 .99736
2.8 .99744 .99752 .99760 .99767 .99774 .99781 .99788 .99795 .99801 .99807
2.9 .99813 .99819 .99825 .99831 .99836 .99841 .99846 .99851 .99856 .99861
3.0 .99865 .99869 .99874 .99878 .99882 .99886 .99889 .99893 .99896 .99900
3.1 .99903 .99906 .99910 .99913 .99916 .99918 .99921 .99924 .99926 .99929
3.2 .99931 .99934 .99936 .99938 .99940 .99942 .99944 .99946 .99948 .99950
3.3 .99952 .99953 .99955 .99957 .99958 .99960 .99961 .99962 .99964 .99965
3.4 .99966 .99968 .99969 .99970 .99971 .99972 .99973 .99974 .99975 .99976
3.5 .99977 .99978 .99978 .99979 .99980 .99981 .99981 .99982 .99983 .99983
3.6 .99984 .99985 .99985 .99986 .99986 .99987 .99987 .99988 .99988 .99989
3.7 .99989 .99990 .99990 .99990 .99991 .99991 .99992 .99992 .99992 .99992
3.8 .99993 .99993 .99993 .99994 .99994 .99994 .99994 .99995 .99995 .99995
3.9 .99995 .99995 .99996 .99996 .99996 .99996 .99996 .99996 .99997 .99997
APPENDICES 425
Time Value of Money
Present Value of Annuity Factors
n/r 1% 2% 3% 4% 5% 6% 7% 8% 9% 10%
1 .9901 .9804 .9709 .9615 .9524 .9434 .9346 .9259 .9174 .9091
2 1.9704 1.9416 1.9135 1.8861 1.8594 1.8334 1.8080 1.7833 1.7591 1.7355
3 2.9410 2.8839 2.8286 2.7751 2.7232 2.6730 2.6243 2.5771 2.5313 2.4869
4 3.9020 3.8077 3.7171 3.6299 3.5460 3.4651 3.3872 3.3121 3.2397 3.1699
5 4.8534 4.7135 4.5797 4.4518 4.3295 4.2124 4.1002 3.9927 3.8897 3.7908
6 5.7955 5.6014 5.4172 5.2421 5.0757 4.9173 4.7665 4.6229 4.4859 4.3553
7 6.7282 6.4720 6.2303 6.0021 5.7864 5.5824 5.3893 5.2064 5.0330 4.8684
8 7.6517 7.3255 7.0197 6.7327 6.4632 6.2098 5.9713 5.7466 5.5348 5.3349
9 8.5660 8.1622 7.7861 7.4353 7.1078 6.8017 6.5152 6.2469 5.9952 5.7590
10 9.4713 8.9826 8.5302 8.1109 7.7217 7.3601 7.0236 6.7101 6.4177 6.1446
11 10.3676 9.7868 9.2526 8.7605 8.3064 7.8869 7.4987 7.1390 6.8052 6.4951
12 11.2551 10.5753 9.9540 9.3851 8.8633 8.3838 7.9427 7.5361 7.1607 6.8137
13 12.1337 11.3484 10.6350 9.9856 9.3936 8.8527 8.3577 7.9038 7.4869 7.1034
14 13.0037 12.1062 11.2961 10.5631 9.8986 9.2950 8.7455 8.2442 7.7862 7.3667
15 13.8651 12.8493 11.9379 11.1184 10.3797 9.7122 9.1079 8.5595 8.0607 7.6061
16 14.7179 13.5777 12.5611 11.6523 10.8378 10.1059 9.4466 8.8514 8.3126 7.8237
17 15.5623 14.2919 13.1661 12.1657 11.2741 10.4773 9.7632 9.1216 8.5436 8.0216
18 16.3983 14.9920 13.7535 12.6593 11.6896 10.8276 10.0591 9.3719 8.7556 8.2014
19 17.2260 15.6785 14.3238 13.1339 12.0853 11.1581 10.3356 9.6036 8.9501 8.3649
20 18.0456 16.3514 14.8775 13.5903 12.4622 11.4699 10.5940 9.8181 9.1285 8.5136
21 18.8570 17.0112 15.4150 14.0292 12.8212 11.7641 10.8355 10.0168 9.2922 8.6487
22 19.6604 17.6580 15.9369 14.4511 13.1630 12.0416 11.0612 10.2007 9.4424 8.7715
23 20.4558 18.2922 16.4436 14.8568 13.4886 12.3034 11.2722 10.3711 9.5802 8.8832
24 21.2434 18.9139 16.9355 15.2470 13.7986 12.5504 11.4693 10.5288 9.7066 8.9847
25 22.0232 19.5235 17.4131 15.6221 14.0939 12.7834 11.6536 10.6748 9.8226 8.0770
26 22.7952 20.1210 17.8768 15.9828 14.3752 13.0032 11.8258 10.8100 9.9290 9.1609
27 23.5596 20.7069 18.3270 16.3296 14.6430 13.2105 11.9867 10.9352 10.0266 9.2372
28 24.3164 21.2813 18.7641 16.6631 14.8981 13.4062 12.1371 11.0511 10.1161 9.3066
29 25.0658 21.8444 19.1885 16.9837 15.1411 13.5907 12.2777 11.1584 10.1983 9.3696
30 25.8077 22.3965 19.6004 17.2920 15.3725 13.7648 12.4090 11.2578 10.2737 9.4269
31 26.5423 22.9377 20.0004 17.5885 15.5928 13.9291 12.5318 11.3498 10.3428 9.4790
32 27.2696 23.4683 20.3888 17.8736 15.8027 14.0840 12.6466 11.4350 10.4062 9.5264
33 27.9897 23.9886 20.7658 18.1476 16.0025 14.2302 12.7538 11.5139 10.4644 9.5694
34 28.7027 24.4986 21.1318 18.4112 16.1929 14.3681 12.8540 11.5869 10.5178 9.6086
35 29.4086 24.9986 21.4872 18.6646 16.3742 14.4982 12.9477 11.6546 10.5668 9.6442
36 30.1075 25.4888 21.8323 18.9083 16.5469 14.6210 13.0352 11.7172 10.6118 9.6765
37 30.7995 25.9695 22.1672 19.1426 16.7113 14.7368 13.1170 11.7752 10.6530 9.7059
38 31.4847 26.4406 22.4925 19.3679 16.8679 14.8460 13.1935 11.8289 10.6908 9.7327
39 32.1630 26.9026 22.8082 19.5845 17.0170 14.9491 13.2649 11.8786 10.7255 9.7570
40 32.8347 27.3555 23.1148 19.7928 17.1591 15.0463 13.3317 11.9246 10.7574 9.7791
41 33.4997 27.7995 23.4124 19.9931 17.2944 15.1380 13.3941 11.9672 10.7866 9.7991
42 34.1581 28.2348 23.7014 20.1856 17.4232 15.2245 13.4524 12.0067 10.8134 9.8174
43 34.8100 28.6616 23.9819 20.3708 17.5459 15.3062 13.5070 12.0432 10.8380 9.8340
44 35.4555 29.0800 24.2543 20.5488 17.6628 15.3832 13.5579 12.0771 10.8605 9.8491
45 36.0945 29.4902 24.5187 20.7200 17.7741 15.4558 13.6055 12.1084 10.8812 9.8628
46 36.7272 29.8923 24.7754 20.8847 17.8801 15.5244 13.6500 12.1374 10.9002 9.8753
47 37.3537 30.2866 25.0247 21.0429 17.9810 15.5890 13.6916 12.1643 10.9176 9.8&66
48 37.9740 30.6731 25.2667 21.1951 18.0772 15.6500 13.7305 12.1891 10.9336 9.8969
49 38.5881 31.0521 25.5017 21.3415 18.1687 15.7076 13.7668 12.2122 10.9482 9.9063
50 39.1961 31.4236 25.7298 21.4822 18.2559 15.7619 13.8007 12.2335 10.9617 9.9148
APPENDICES 426
Time Value of Money
Present Value of Annuity Factors

n/r 11% 12% 13% 14% 15% 16% 17% 18% 19%
1 .9009 .8929 .8850 .8772 .8696 .8621 .8547 .8475 .8403
2 1.7125 1.6901 1.6681 1.6467 1.6257 1.6052 1.5852 1.5656 1.5465
3 2.4437 2.4018 2.3612 2.3216 2.2832 2.2459 2.2096 2.1743 2.1399
4 3.1024 3.0373 2.9745 2.9137 2.8550 2.7982 2.7432 2.6901 2.6386
5 3.6959 3.6048 3.5172 3.4331 3.3522 3.2743 3.1993 3.1272 3.0576
6 4.2305 4.1114 3.9976 3.8887 3.7845 3.6847 3.5892 3.4976 3.4098
7 4.7122 4.5638 4.4226 4.2883 4.1604 4.0386 3.9224 3.8115 3.7057
8 5.1461 4.9676 4.7988 4.6389 4.4873 4.3436 4.2072 4.0776 3.9544
9 5.5370 5.3283 5.1317 4.9464 4.7716 4.6065 4.4506 4.3030 4.1633
10 5.8892 5.6502 5.4262 5.2161 5.0188 4.8332 4.6586 4.4941 4.3389
11 6.2065 5.9377 5.6869 5.4527 5.2337 5.0286 4.8364 4.6560 4.4865
12 6.4924 6.1944 5.9176 5.6603 5.4206 5.1971 4.9884 4.7932 4.6105
13 6.7499 6.4235 6.1218 5.8424 5.5831 5.3423 5.1183 4.9095 4.7147
14 6.9819 6.6282 6.3025 6.0021 5.7245 5.4675 5.2293 5.0081 4.8023
15 7.1909 6.8109 6.4624 6.1422 5.8474 5.5755 5.3242 5.0916 4.8759
16 7.3792 6.9740 6.6039 6.2651 5.9542 5.6685 5.4053 5.1624 4.9377
17 7.5488 7.1196 6.7291 6.3729 6.0472 5.7487 5.4746 5.2223 4.9897
18 7.7016 7.2497 6.8399 6.4674 6.1280 5.8178 5.5339 5.2732 5.0333
19 7.8393 7.3658 6.9380 6.5504 6.1982 5.8775 5.5845 5.3162 5.0700
20 7.9633 7.4694 7.0248 6.6231 6.2593 5.9288 5.6278 5.3527 5.1009
21 8.0751 7.5620 7.1016 6.6870 6.3125 5.9731 5.6648 5.3837 5.1268
22 8.1757 7.6446 7.1695 6.7429 6.3587 6.0113 5.6964 5.4099 5.1486
23 8.2664 7.7184 7.2297 6.7921 6.3988 6.0442 5.7234 5.4321 5.1668
24 8.3481 7.7843 7.2829 6.8351 6.4338 6.0726 5.7465 5.4509 5.1822
25 8.4217 7.8431 7.3300 6.8729 6.4641 6.0971 5.7662 5.4669 5.1951
26 8.4881 7.8957 7.3717 6.9061 6.4906 6.1182 5.7831 5.4804 5.2060
27 8.5478 7.9426 7.4086 6.9352 6.5135 6.1364 5.7975 5.4919 5.2151
28 8.6016 7.9844 7.4412 6.9607 6.5335 6.1520 5.8099 5.5016 5.2228
29 8.6501 8.0218 7.4701 6.9830 6.5509 6.1656 5.8204 5.5098 5.2292
30 8.6938 8.0552 7.4957 7.0027 6.5660 6.1772 5.8294 5.5168 5.2347
31 8.7331 8.0850 7.5183 7.0199 6.5791 6.1872 5.8371 5.5227 5.2392
32 8.7686 8.1116 7.5383 7.0350 6.5905 6.1959 5.8437 5.5277 5.2430
33 8.8005 8.1354 7.5560 7.0482 6.6005 6.2034 5.8493 5.5320 5.2462
34 8.8293 8.1566 7.5717 7.0599 6.6091 6.2098 5.8541 5.5356 5.2489
35 8.8552 8.1755 7.5856 7.0700 6.6166 6.2153 5.8582 5.5386 5.2512
36 8.8786 8.1924 7.5979 7.0790 6.6231 6.2201 5.8617 5.5412 5.2531
37 8.8996 8.2075 7.6087 7.0868 6.6288 6.2242 5.8647 5.5434 5.2547
38 8.9186 8.2210 7.6183 7.0937 6.6338 6.2278 5.8673 5.5452 5.2561
39 8.9357 8.2330 7.6268 7.0997 6.6380 6.2309 5.8695 5.5468 5.2572
40 8.9511 8.2438 7.6344 7.1050 6.6418 6.2335 5.8713 5.5482 5.2582
41 8.9649 8.2534 7.6410 7.1097 6.6450 6.2358 5.8729 5.5493 5.2590
42 8.9774 8.2619 7.6469 7.1138 6.6478 6.2377 5.8743 5.5502 5.2596
43 8.9886 8.2696 7.6522 7.1173 6.6503 6.2394 5.8755 5.5510 5.2602
44 8.9988 8.2764 7.6568 7.1205 6.6524 6.2409 5.8765 5.5517 5.2607
45 9.0079 8.2825 7.6609 7.1232 6.6543 6.2421 5.8773 5.5523 5.2611
46 9.0161 8.2880 7.6645 7.1256 6.6559 6.2432 5.8781 5.5528 5.2614
47 9.0235 8.2928 7.6677 7.1277 6.6573 6.2442 5.8787 5.5532 5.2617
48 9.0302 8.2972 7.6705 7.1296 6.6585 6.2450 5.8792 5.5536 5.2619
49 9.0362 8.3010 7.6730 7.1312 6.6596 6.2457 5.8797 5.5539 5.2621
50 9.0417 8.3045 7.6752 7.1327 6.6605 6.2463 5.8801 5.5541 5.2623

Discount Factor =1 / (1 + r)n Where r = Discount rate and n = length of time


APPENDICES 427
Present Value Factor for a Single Future Amount
(Interest rate = r, Number of periods = n)
n/r 1% 2% 3% 4% 5% 6% 7% 8% 9% 10%
1 .9901 .9804 .9709 .9615 .9524 .9434 .9346 .9259 .9174 .9091
2 .9803 .9612 .9426 .9246 .9070 .8900 .8734 .8573 .8417 .8264
3 .9707 .9423 .9151 .8890 .8638 .8396 .8163 .7938 .7722 .7513
4 .9610 .9238 .8885 .8548 .8227 .7921 .7629 .7350 .7084 .6830
5 .9515 .9057 .8626 .8219 .7835 .7473 .7130 .6806 .6499 .6209
6 .9420 .8880 .8375 .7903 .7462 .7050 .6663 .6302 .5963 .5645
7 .9327 .8706 .8131 .7599 .7107 .6651 .6228 .5835 .5470 .5132
8 .9235 .8535 .7894 .7307 .6768 .6274 .5820 .5403 .5019 .4665
9 .9143 .8368 .7664 .7026 .6446 .5919 .5439 .5002 .4604 .4241
10 .9053 .8203 .7441 .6756 .6139 .5584 .5083 .4632 .4224 .3855
11 .8963 .8043 .7224 .6496 .5847 .5268 .4751 .4289 .3875 .3505
12 .8874 .7885 .7014 .6246 .5568 .4970 .4440 .3971 .3555 .3186
13 .8787 .7730 .6810 .6006 .5303 .4688 .4150 .3677 .3262 .2897
14 .8700 .7579 .6611 .5775 .5051 .4423 .3878 .3405 .2992 .2633
15 .8613 .7430 .6419 .5553 .4810 .4173 .3624 .3152 .2745 .2394
16 .8528 .7284 .6232 .5339 .4581 .3936 .3387 .2919 .2519 .2176
17 .8444 .7142 .6050 .5134 .4363 .3714 .3166 .2703 .2311 .1978
18 .8360 .7002 .5874 .4936 .4155 .3503 .2959 .2502 .2120 .1799
19 .8277 .6864 .5703 .4746 .3957 .3305 .2765 .2317 .1945 .1635
20 .8195 .6730 .5537 .4564 .3769 .3118 .2584 .2145 .1784 .1486
21 .8114 .6598 .5375 .4388 .3589 .2942 .2415 .1987 .1637 .1351
22 .8034 .6468 .5219 .4220 .3419 .2775 .2257 .1839 .1502 .1228
23 .7954 .6342 .5067 .4057 .3256 .2618 .2109 .1703 .1378 .1117
24 .7876 .6217 .4919 .3901 .3101 .2470 .1971 .1577 .1264 .1015
25 .7798 .6095 .4776 .3751 .2953 .2330 .1842 .1460 .1160 .0923
26 .7720 .5976 .4637 .3607 .2812 .2198 .1722 .1352 .1064 .0839
27 .7644 .5859 .4502 .3468 .2678 .2074 .1609 .1252 .0976 .0763
28 .7568 .5744 .4371 .3335 .2551 .1956 .1504 .1159 .0895 .0693
29 .7493 .5631 .4243 .3207 .2429 .1846 .1406 .1073 .0822 .0630
30 .7419 .5521 .4120 .3083 .2314 .1741 .1314 .0994 .0754 .0573
31 .7346 .5412 .4000 .2965 .2204 .1643 .1228 .0920 .0691 .0521
32 .7273 .5306 .3883 .2851 .2099 .1550 .1147 .0852 .0634 .0474
33 .7201 .5202 .3770 .2741 .1999 .1462 .1072 .0789 .0582 .0431
34 .7130 .5100 .3660 .2636 .1904 .1379 .1002 .0730 .0534 .0391
35 .7059 .5000 .3554 .2534 .1813 .1301 .0937 .0676 .0490 .0356
36 .6989 .4902 .3450 .2437 .1727 .1227 .0875 .0626 .0449 .0323
37 .6920 .4806 .3350 .2343 .1644 .1158 .0818 .0580 .0412 .0294
38 .6858 .4712 .3252 .2253 .1566 .1092 .0765 .0537 .0378 .0267
39 .6784 .4619 .3158 .2166 .1491 .1031 .0715 .0497 .0347 .0243
40 .6717 .4529 .3066 .2083 .1420 .0972 .0668 .0460 .0318 .0221
41 .6650 .4440 .2976 .2003 .1353 .0917 .0624 .0426 .0292 .0201
42 .6584 .4353 .2890 .1926 .1288 .0865 .0583 .0395 .0268 .0183
43 .6520 .4268 .2805 .1852 .1227 .0816 .0545 .0365 .0246 .0166
44 .6454 .4184 .2724 .1780 .1169 .0770 .0509 .0338 .0226 .0151
45 .6391 .4102 .2644 .1712 .1113 .0727 .0476 .0313 .0207 .0137
46 .6327 .4022 .2567 .1646 .1060 .0685 .0445 .0290 .0190 .0125
47 .6265 .3943 .2493 .1583 .1009 .0647 .0416 .0269 .0174 .0113
48 .6203 .3865 .2420 .1522 .0961 .0610 .0389 .0249 .0160 .0103
49 .6141 .3790 .2350 .1463 .0916 .0575 .0363 .0230 .0147 .0094
50 .6080 .3715 .2281 .1407 .0872 .0543 .0339 .0213 .0134 .0085

Discount Factor =1 / (1 + r)n Where r = Discount rate and n = length of time


APPENDICES 428
Present Value Factor for a Single Future Amount
(Interest rate = r, Number of periods = n)
n/r 11% 12% 13% 14% 15% 16% 17% 18% 19%
1 .9009 .8929 .8850 .8772 .8696 .8621 .8547 .8475 .8403
2 .8116 .7972 .7831 .7695 .7561 .7432 .7305 .7182 .7062
3 .7312 .7118 .6931 .6750 .6575 .6407 .6244 .6086 .5934
4 .6587 .6355 .6133 .5921 .5718 .5523 .5337 .5158 .4987
5 .5935 .5674 .5428 .5194 .4972 .4761 .4561 .4371 .4190
6 .5346 .5066 .4803 .4556 .4323 .4104 .3898 .3704 .3521
7 .4817 .4523 .4251 .3996 .3759 .3538 .3332 .3139 .2959
8 .4339 .4039 .3762 .3506 .3269 .3050 .2848 .2660 .2487
9 .3909 .3606 .3329 .3075 .2843 .2630 .2434 .2255 .2090
10 .3522 .3220 .2946 .2697 .2472 .2267 .2080 .1911 .1756
11 .3173 .2875 .2607 .2366 .2149 .1954 .1778 .1619 .1476
12 .2858 .2567 .2307 .2076 .1869 .1685 .1520 .1372 .1240
13 .2575 .2292 .2042 .1821 .1625 .1452 .1299 .1163 .1042
14 .2320 .2046 .1807 .1597 .1413 .1252 .1110 .0985 .0876
15 .2090 .1827 .1599 .1401 .1229 .1079 .0949 .0835 .0736
16 .1883 .1631 .1415 .1229 .1069 .0930 .0811 .0708 .0618
17 .1696 .1456 .1252 .1078 .0929 .0802 .0693 .0600 .0520
18 .1528 .1300 .1108 .0946 .0808 .0691 .0592 .0508 .0437
19 .1377 .1161 .0981 .0829 .0703 .0596 .0506 .0431 .0367
20 .1240 .1037 .0868 .0728 .0611 .0514 .0433 .0365 .0308
21 .1117 .0926 .0768 .0638 .0531 .0443 .0370 .0309 .0259
22 .1007 .0826 .0680 .0560 .0462 .0382 .0316 .0262 .0218
23 .0907 .0738 .0601 .0491 .0402 .0329 .0270 .0222 .0183
24 .0817 .0659 .0532 .0431 .0349 .0284 .0231 .0188 .0154
25 .0736 .0588 .0471 .0378 .0304 .0245 .0197 .0160 .0129
26 .0663 .0525 .0417 .0331 .0264 .0211 .0169 .0135 .0109
27 .0597 .0469 .0369 .0291 .0230 .0182 .0144 .0115 .0091
28 .0538 .0419 .0326 .0255 .0200 .0157 .0123 .0097 .0077
29 .0485 .0374 .0289 .0224 .0174 .0135 .0105 .0082 .0064
30 .0437 .0334 .0256 .0196 .0151 .0116 .0090 .0070 .0054
31 .0394 .0298 .0226 .0172 .0131 .0100 .0077 .0059 .0046
32 .0355 .0266 .0200 .0151 .0114 .0087 .0066 .0050 .0038
33 .0319 .0238 .0177 .0132 .0099 .0075 .0056 .0042 .0032
34 .0288 .0212 .0157 .0116 .0086 .0064 .0048 .0036 .0027
35 .0259 .0189 .0139 .0102 .0075 .0055 .0041 .0030 .0023
36 .0234 .0169 .0123 .0089 .0065 .0048 .0035 .0026 .0019
37 .0210 .0151 .0109 .0078 .0057 .0041 .0030 .0022 .0016
38 .0190 .0135 .0096 .0069 .0049 .0036 .0026 .0019 .0013
39 .0171 .0120 .0085 .0060 .0043 .0031 .0022 .0016 .0011
40 .0154 .0107 .0075 .0053 .0037 .0026 .0019 .0013 .0010
41 .0139 .0096 .0067 .0046 .0032 .0023 .0016 .0011 .0008
42 .0125 .0086 .0059 .0041 .0028 .0020 .0014 .0010 .0007
43 .0112 .0076 .0052 .0036 .0025 .0017 .0012 .0008 .0006
44 .0101 .0068 .0046 .0031 .0021 .0015 .0010 .0007 .0005
45 .0091 .0061 .0041 .0028 .0019 .0013 .0009 .0006 .0004
46 .0082 .0054 .0036 .0024 .0016 .0011 .0007 .0005 .0003
47 .0074 .0059 .0032 .0021 .0014 .0009 .0006 .0004 .0003
48 .0067 .0043 .0028 .0019 .0012 .0008 .0005 .0004 .0002
49 .0060 .0039 .0025 .0016 .0011 .0007 .0005 .0003 .0002
50 .0054 .0035 .0022 .0014 .0009 .0006 .0004 .0003 .0002
Discount Factor =1 / (1 + r)n Where r = Discount rate and n = length of time
INDEX 429

Absorbing in nature, 140 Commutative law for Descriptive model, 293


Accounts receivables, 140 intersection, 7 Determinant, 96
Activity Time, 314 Commutative law for Union, Difference rule, 181
Differentiation, 168
Activity, 313 7
Discounted cash flow
Additional cost, 336 Complement set, 3 methods, 261
Adjoint matrix, 115 Completing the square Discounted payback period,
Adjugate matrix, 114 method, 230 261
Analagol Model, 294 Compliment, 3 Discriminant, 229
Annual cost of ordering, 408 Compound inequalities, 64 Disjoined sets, 5
Annual demand, 409 Compound Interest, 260 Distributive law for
intersection, 7
Annuity, 266 Conditional economic
Distributive law for union, 7
Anti – derivative, 218 consequences, 155 Division Rule, 187
Arc, 25 Conditional profits values, Division, 19
Assets, 261 155 Dual Program, 311
Assignment model, 378 Conservative, 158 Dummy activity, 313
Associative law for Constant Rule, 179 Earliest starting time, 314
intersection, 7 Constant, 25 Economic Batch Quantity,
Constrained Optimization, 412
Associative law for union, 7
215 Economic life, 261
Assumptions of Markov, 140 Element, 1
Constraints, 275, 284
Average cost, 243 Coordinates, 20 Elimination method38
Average rate of return, 261 Cost of holding, 409 Elimination, 38
Basic Variables, 297-299 Cost of ordering, 408 Empty set, 3
Basic, 358 Cost of ordering, 409 Equilibrium price, 55
Crash cost, 335 Equilibrium quantity, 55
Batch size, 412
Critical path, 314 Equilibrium state, 143
Bayes approach, 161 Equilibrium, 55
Curve, 25
Bayes, 161 Event number, 314
Data collection, 294
Binomial Theorem, 76 De Morgan Laws, 7 Event, 313
Brand royalty, 140 Decision analysis, 155 Expected Time, 325
Break even point, 245 Decision making problem, Expected value approach,
Break even point, 250 155 160
Decision node, 157 Expected value, 160
Cartesian plane, 20, 21
Decision tree, 157 External demand, 121
Cause of action, 155 Factor method, 230
Decision, 155
Chain rule, 190 Factorial, 81
Definite Integral, 220
Characteristics of Markov, Degeracy, 379 Feasible production set, 285
140 Delta, 21 Feasible region, 280, 287,
Chebychev’s theorem, 327 Demand analysis, 46 296
Chord, 168 Demand Function, 47 Feasible Solution, 296
Demand Functions, 46 Final demand, 121
Co-efficient, 59, 60
Demand, 46 Finite number, 140
Cofactors, 100 Fixed cost, 243
Denominator, 187
Coincident line, 36 Dependent variable, 46 Float time, 336
Combination Sum rule, 183 Derivative, 170 Free time, 337
Combinations, 83
INDEX 430

Gaussian elimination Marginal cost, 199, 223-227 Optimistic approach, 161


process, 45 Marginal revenue, 199, 223,- Optimizer, 158
Gradient of a curve, 168 227 Optimum order quantity, 409
Gradient, 19, 21, 168 Market share, 140 Order quantity, 409
Graph, 31 Marketing, 140 Order size, 407
Graphical method, 230 Markov model, 140 Origin, 20
Hessian determinant is Markov Process model, 140 Parabola, 228
negative, 212 Mathematical Model, 294 Partial differential, 209
Hessian determinant is Matrix notation, 126 Pascal's triangle, 75
positive, 212 Matrix, 84 Pay off, 155
Hessian determinant, 209 Max regret, 159 Payback period method, 261,
Heuristic Model, 294 Maxima, 193 262
Horizontal axis, 20 Maxmin Minimax Approach, Permutation, 81
Horizontal gradient, 29 158 Pessimistic approach, 161
Hungarian Algorithm, 378 Minima, 193 Points of intersection, 33
Iconic Model, 294 Minimax approach, 158 Points, 20
Identity matrix, 96 Minimax Regret Approach, Positive co-efficient, 59
Implementation of final 158 Positive gradient, 26
solution, 294 Minimizing Simplex Method, Positive integer, 76
Independent Float, 337 311 Posterior probabilities, 163
Independent variable, 46 Minimum cost method, 343 Preceding Activity, 325
Inequalities, 63 Minimum return, 261 Preceding state, 140
Initial cash outflow, 261 Model building, 293 Present Value, 267
Input-output, 121 Multiplication of matrices, 90 Principle diagonal, 84
Insurance policies, 140 Multiplication rule, 185 Principle of Inclusion-
Integration, 218 Multiplication, 19 Exclusion, 6
Intermediate output, 121 Natural numbers, 2, 16 Principle, 85
Internal consumption, 125 Negative coefficient, 60 Problem Recognition, 293
Internal rate of return Negative gradient, 28 Problem solving, 294
method, 261, 265 Negative integer, 2, 16 Product rule, 185
Interpretation of the Negative integer, 77 Production possibility set,
solution, 294 Net cash inflow, 261 285
Intersection of sets, 3 Net present value, 261 Quadratic formula, 230
Inventory Planning & Net Present Value, 264 Quantitative techniques, 293
Control, 406 Network analysis, 313 Quotient Rule, 187
Investment, 261 Network, 313 Range, 31
Investments Analysis, 261 New demand, 138 Rational numbers, 2, 16
Lagrange multipliers, 215 Non-singular matrix, 97 Real numbers, 2, 16
Lambda, 215 Normal cost, 335 Reciprocal, 96
Laplace, 158 Normative model, 293 Regret opportunity cost, 158
Latest starting time, 314 Notation, 4 Regret table, 159
Lead time, 407 Number line, 16 Re-order level, 407
Leontief inverse, Numerator, 187 Returns, 261
126,129,132,137 Numeric, 218 Review and Maintenance,
Leontief Model, 126 Objectives function, 275, 284 294
Linear program, 276 Opportunity cost, 158 Scalar multiplication, 88
Linear Programming, 274 Opportunity set, 285 Secant, 168
Long-run, 143 Optimal strategy, 163
INDEX 431

Second derivative is positive, Technical coefficient matrix,


196 122
Second derivative is positive, Technology matrix, 122
196 Total cost, 199
Second derivative, 195 Total Cost, 409
Segment, 21 Total Float, 336
Sentivity analysis, 274 Total internal consumption,
Set up costs, 412 125
Set, 1 Total normal cost, 335
Shadow cost, 359 Total revenue, 199, 241
Simple inequalities, 64 Traditional method, 261
Simple Interest, 258 Transition probabilities, 140,
Simplex algorithm, 295 141
Simplex method, 295 Transition probability matrix,
Simulation Model, 294 144
Simultaneous equation, 35 Transportation problem, 343
Singular, 97, 98 Transpose, 89
Sketch, 31 Transposing, 115
Slack variable, 295 Turning points, 193
Slope of line, 25, 168 Umbers1
Slope, 21, 168 Uncertainty, 155
Standard equation, 146 Union of set, 3
Standard form, 25 Universal set, 3
Standard Normal Variable cost, 243
Distribution, 319, 321,322 Variable Cost, 335
Standardization, 295 Venn diagram, 4
Starting event, 314 Vertical axis, 20
State of nature, 155 Vogel method, 344, 351
State on nature node, 157 x-axis, 20
State transition relationship, x-intercepts, 32
140 y- axis, 20
Stationary Points, 193 y-intercept, 25,32
Steady state, 143 Zero gradient, 29
Steepness, 21 Z score, 319
Stepping circuit, 364
Stochastic process, 140
Stock level, 407
Stock out, 407
Straight line, 23
Subset, 4
Substitution method, 39
Substitution, 39
Sum Rule, 180
Supply analysis, 46
Supply Function, 49
Supply, 46
Surplus variable, 295
Tangent, 168

View publication stats

Anda mungkin juga menyukai